Questions 2020

अब Quizwiz के साथ अपने होमवर्क और परीक्षाओं को एस करें!

A 5-year-old girl is seen for evaluation because her older brother has severe hemophilia A. She has frequent epistaxis from both nares lasting up to 30 minutes per episode. No other family members have bleeding symptoms. Of the following, the MOST appropriate next step in the evaluation of this patient is A.activated partial thromboplastin time B.factor VIII level C.factor VIII genetic analysis D.reassurance

The child in this vignette has a 50% chance of being a carrier of hemophilia A, or factor VIII deficiency. Although up to a third of patients with severe hemophilia A have no known family history, the majority of these de novo cases are caused by new germline mutations in the mother of the affected child; thus, their siblings are also at risk of inheriting the causative mutation. The activated partial thromboplastin time will be abnormal in all patients with severe hemophilia A, but this test may miss mild factor deficiencies as seen in female carriers, depending on the sensitivity of the test used. Genetic analysis of the factor VIII gene (F8) will in almost all cases identify the carrier state. However, genetic testing requires genetic information from the proband, may not be covered by insurance, and may not be readily available. Women and girls who are heterozygous, or carriers, for hemophilia have one normal X chromosome and one with a causative mutation. They can present with factor VIII levels ranging from normal (asymptomatic) to those diagnostic of severe hemophilia, depending upon the pattern of X chromosome inactivation. For an individual female, the factor level will remain fairly stable throughout her life, although it can vary with stress and pregnancy. Girls may be diagnosed with hemophilia A by obtaining factor VIII levels. However, carrier status cannot be completely ruled out without genetic analysis of the affected gene. Girls and women who are known carriers with factor levels in the normal range report increased bleeding symptoms compared to unaffected women. Pregnant women who are asymptomatic or symptomatic carriers of hemophilia may wish to explore prenatal testing for diagnosis of the fetus and help in planning their delivery. Chorionic villus sampling can be done as early as 13 weeks' gestation, and amniocentesis can be performed at 18 weeks' gestation. Both of these methods use genetic analysis. Less invasive methods of prenatal diagnosis are being investigated. Pre-implantation genetic diagnosis is also available. Umbilical cord blood sampling of factor activity is the preferred method for diagnosis of a potentially affected neonate at the time of delivery because it is faster and less invasive than venipuncture. Factor VIII levels at birth are similar to those of an adult, so the diagnosis of factor VIII deficiency is straightforward. In contrast, the normal range of factor IX is lower in newborns than in adults, and the results must be compared to age-adjusted normal values if hemophilia B is suspected. In some cases, definitive diagnosis of mild factor IX deficiency must be delayed until around 6 months of age when infants without hemophilia will typically achieve adult values. PREP Pearls The activated partial thromboplastin time will be abnormal in all patients with severe hemophilia, but it may miss mild factor deficiencies depending on the sensitivity of the test used at the laboratory. Genetic analysis in almost all cases of hemophilia will identify the asymptomatic and symptomatic carrier state. Girls who are heterozygous or carriers for hemophilia can present with factor levels ranging from normal (asymptomatic) to those diagnostic of severe hemophilia, depending upon the pattern of X chromosome inactivation.

A previously healthy 14-year-old adolescent boy has severe anemia, leukopenia, and neutropenia. Laboratory data are shown: Laboratory Test Result Hemoglobin 6.8 g/dL (68 g/L) Mean corpuscular volume 84 µm3 (84 fL) White blood cell count 1,500/µL (1.5 × 109/L) Absolute neutrophil count 280/µL (0.28 × 109/L) Platelet count 150 × 103/µL (150 × 109/L) Peripheral blood smear Mild anisocytosis He consumes a regular diet and is receiving no prescribed medications; however, he has been ingesting many zinc lozenges on a daily basis during the past year to prevent colds. His physical examination findings are notable for pallor, mild tachycardia, and a subtle sensory ataxia. Of the following, the MOST likely bone marrow finding is A.abnormal megakaryocytes B.erythroid hypoplasia C.granulocytic hyperplasia D.increased iron staining

The described scenario is highly suggestive of copper deficiency due to zinc toxicity. Copper deficiency classically manifests with anemia (which usually is normocytic or macrocytic but can also be microcytic), leukopenia, and neutropenia (often severe). The platelet count is usually normal or slightly decreased. Because copper is a key trace element and cofactor to several critical oxidative enzymes, its deficiency can also cause significant neurologic symptoms (eg, sensory ataxia, peripheral neuropathy, paresthesias, spasticity), often mimicking the subacute combined degeneration seen in vitamin B12 deficiency. However, the anemia of vitamin B12 deficiency typically is macrocytic, and although leukopenia can occur, neutrophils are classically hypersegmented. Excess zinc intake leads to upregulation of the chelating protein metallothionein within enterocytes. Because copper has a higher affinity for this protein than does zinc, over time, excess zinc exposure leads to chronically impaired copper absorption and eventual copper deficiency. Besides zinc toxicity, other conditions that predispose to copper deficiency include malabsorption syndromes (eg, cystic fibrosis, celiac disease, protein-losing enteropathies, short bowel syndrome), a history of gastrointestinal surgery (eg, gastric bypass) and total parenteral nutrition without adequate copper supplementation. Copper is absorbed primarily in the stomach and proximal duodenum. A 2018 case report described its occurrence in a child on a ketogenic diet. A significant proportion of cases of diagnosed copper deficiency lack any apparent predisposing risk factors. Although copper deficiency can be easily confirmed by checking serum copper and ceruloplasmin levels, additional studies (including bone marrow examination) are often performed because copper deficiency may be overlooked as a diagnostic consideration. Typical bone marrow findings in copper deficiency include erythroid hyperplasia and left-shifted granulocytic hypoplasia, resembling myeloid maturation arrest. Cytoplasmic vacuolization of erythroid and myeloid precursors is often present, but megakaryopoiesis (and megakaryocyte morphology) is usually normal. Increased iron staining in the bone marrow is a hallmark of copper deficiency, with classic findings being iron inclusions in plasma cells, increased macrophage iron, and ringed sideroblasts. Hematogone hyperplasia has also been described. Bone marrow cellularity may be increased, normal, or decreased. Overall, the bone marrow findings can resemble those found in myelodysplastic syndrome, a frequent misdiagnosis. Copper deficiency is rarely encountered in clinical practice, contributing to the challenges of recognizing it when it does occur. Median time to its diagnosis has been reported as 1 year, despite the severe anemia and neutropenia that can result. The hematologic abnormalities typically normalize within weeks of implementing copper replacement, but neurologic features may take longer to resolve or may even linger. A high index of suspicion for copper deficiency is thus required to minimize long-term sequelae, especially when anemia, neutropenia, or both are accompanied by neurologic symptoms. PREP Pearls Copper deficiency is rarely encountered in clinical practice but is considered in the differential diagnosis of anemia and neutropenia, especially when accompanied by neurologic symptoms. Risk factors for copper deficiency include gastrointestinal malabsorption syndromes, history of gastrointestinal surgery, and excess zinc exposure. Marrow findings suggestive of copper deficiency include erythroid hyperplasia, granulocytic hypoplasia, vacuolization of erythroid and myeloid precursors, and increased iron staining, which are similar to findings in myelodysplastic syndrome. ABP Content Specifications(s)/Content Area Hematologic findings and epidemiology of copper deficiency-induced cytopenias

A 5-year-old boy with acute myeloid leukemia and pulmonary aspergillosis has been hospitalized with fever and neutropenia for more than 1 month. He was initially treated with voriconazole but now receives liposomal amphotericin because of persistent fever. Despite daily injections of granulocyte colony-stimulating factor since hospital admission, his absolute neutrophil count remains zero. He is intubated in the intensive care unit and receiving inotropic agents. Computed tomography shows worsening pulmonary infiltrates and new liver lesions. He was given a granulocyte transfusion today. Of the following, the transfusion-associated complication that is MOST likely to arise within the next 48 hours is A.encephalitis B.hepatitis C.nephritis D.pneumonitis

The most significant immediate adverse event to granulocyte transfusions is pneumonitis or pulmonary inflammation resulting in respiratory distress within 48 hours of transfusion. The inflammation may be caused by factors including worsening of the existing pneumonia, transfusion-related acute lung injury, inadvertent viral transmission (hepatitis B or C, HIV), fluid/volume overload, or allergic reactions. Encephalitis, hepatitis, and nephritis are possible complications of overwhelming infection in an immunocompromised patient with fever and neutropenia, but they are not common adverse effects of granulocyte transfusions. Another possible complication of granulocyte transfusions is the development of anti-HLA antibodies, antineutrophil antibodies, or both. Anti-HLA antibodies may lead to refractoriness to the infused neutrophils or rejection of an HLA-mismatched hematopoietic stem cell graft. Because cytomegalovirus (CMV) antigen may be stored in neutrophils previously exposed to CMV, transmission of CMV infection is another possible complication of granulocyte transfusions. For this reason, most centers use CMV-negative donors. Two of the more recent randomized clinical trials studying the efficacy of granulocyte transfusions have not shown any clinical benefits. Therefore, the decision to transfuse granulocytes is based on retrospective reviews and case reports. More studies are needed to better define the indications for granulocyte transfusion. Common uses include prolonged neutropenia with concurrent, overwhelming infection unresponsive to appropriate antimicrobial agents. In light of the potential complications, the risk-benefit ratio of granulocyte transfusions needs to be carefully considered. Currently, granulocyte colony-stimulating factor, dexamethasone, or both are used to mobilize the neutrophils of donors for granulocyte donation to significantly increase the number of granulocytes per leukapheresis collection. At least 1.5 to 100 × 108 granulocytes/kg of recipient body weight is the recommended daily infusion dose to expect efficacy. Some experts recommend higher granulocyte concentrations, such as 0.6 to 40 × 109 granulocytes/kg. After collection, granulocytes are irradiated to prevent transfusion-associated graft-vs-host disease and then are transfused into the recipient within 6 hours of collection. Contamination by donor red blood cells also mandates that the granulocyte product be ABO blood group-compatible. PREP Pearls At least 1.5 to 100 × 108 granulocytes/kg of body weight of the recipient is the recommended dose to be transfused for efficacy. Granulocytes should be irradiated prior to infusion to prevent transfusion-related graft-vs-host disease. Pneumonitis or pulmonary inflammation resulting in respiratory distress is the most common immediate adverse reaction to granulocyte transfusions. Potential adverse effects of granulocyte transfusions include HLA alloimmunization and cytomegalovirus transmission. ABP Content Specifications(s)/Content Area

A 5-year-old boy develops leg pain, abdominal cramping, and dark urine a few hours after participating in a snowball fight with his brothers. He is generally healthy but his family reports that he had "walking pneumonia" a few weeks ago. A laboratory evaluation shows reticulocytosis and the following results: Laboratory Test Result White blood cell count 5,400/µL (5.4 × 109/L) Hemoglobin 6.2 g/dL (62 g/L) Platelet count 189 × 103/µL (189 × 109/L) Lactate dehydrogenase Elevated Direct antiglobulin test Positive for C3, negative for IgG No agglutination is present on the peripheral blood smear. Urine is positive for heme but not red blood cells. Additional testing, as shown, is performed: Tube contents: Incubation parameters: A1, B1, C1 = patient serum only A1, A2, A3 = 0°C × 30 min, then 37°C × 60 min A2, B2, C2 = patient serum and control serum B1, B2, B3 = 0°C × 90 min A3, B3, C3 = control serum only C1, C2, C3 = 37°C × 90 min Of the following, which diagnosis is MOST likely? A.cold agglutinin disease B.cryoglobulinemia C.paroxysmal cold hemoglobinuria D.warm autoimmune hemolytic anemia

The patient in this vignette has intravascular hemolysis as evidenced by elevated lactate dehydrogenase, reticulocytosis, and hemoglobinuria following exposure to cold. Moreover, his blood bank testing is consistent with a biphasic antibody that binds at cold temperatures and causes hemolysis when warmed to 37°C (ie, Donath-Landsteiner antibody), making paroxysmal cold hemoglobinuria (PCH), the most likely diagnosis. The Donath-Landsteiner testing shows hemolysis only in the tubes with the patient's serum that were first held at 0°C for 30 minutes and then warmed to 37°C and incubated for 60 minutes. Paroxysmal cold hemoglobinuria is a generally transient autoimmune hemolytic anemia first recognized in 1872. The majority of the initial patients identified had advanced syphilis infections, as the antibody responsible for PCH cross-reacts with antigens on Treponema pallidum. It was later determined by Donath and Landsteiner that the antibody responsible for triggering PCH is against the P antigens on red blood cells. The antibody is of the IgG subtype, and while it does not agglutinate red blood cells (as seen with the IgM antibody in cold agglutinin disease) it can fix complement. The polyclonal anti-P antibodies require the blood to be chilled to facilitate binding of the antibody to the red blood cell and fixation of complement. Subsequent warming of the blood results in complement activation and intravascular hemolysis. This sequence of events with chilling and then rewarming the blood is the laboratory approach to detecting the Donath-Landsteiner antibody. The majority of modern PCH diagnoses in children stem from recent viral infections, particularly with varicella, although measles, mumps, Epstein-Barr virus, cytomegalovirus, adenovirus, respiratory syncytial virus, and influenza A have been reported. Associated bacterial infections include Mycoplasma pneumoniae. Far less frequently, PCH is associated with autoimmune conditions or malignancy in children. Transfusions can be given to patients with life-threatening hemolysis, ideally from a donor who is P-antigen negative, although this is not always feasible. Plasma exchange has been used successfully in acutely ill patients, while immunosuppression with prednisone and/or cyclophosphamide has been used for persistent/refractory cases. In general, the majority of children do well with minimal intervention consisting of cold avoidance, adequate hydration, and monitoring. Splenectomy has no role in the treatment of PCH because the hemolysis is intravascular. Cold agglutinin disease is a result of IgM antibodies that can usually fix complement on the direct antiglobulin test but also agglutinate red blood cells. Patients with cryoglobulinemia generally present with symptoms following exposure to the cold, but their direct antiglobulin test result is negative and hemolysis is not significant. Warm autoimmune hemolytic anemia is characterized by an IgG antibody that can often fix complement; however, the antibodies bind at body temperature and do not require cooling/rewarming to activate complement. PREP Pearls The most common infection associated with paroxysmal cold hemoglobinuria is varicella, although most common childhood viruses have been implicated. The presence of the Donath-Landsteiner antibody can be used to diagnose paroxysmal cold hemoglobinuria. The Donath-Landsteiner antibody is directed towards the P antigen on red blood cells. Treatment for paroxysmal cold hemoglobinuria primarily involves avoidance of cold, with prednisone and other immunosuppressants reserved for persistent cases.

A 9-year-old girl developed cough, rhinorrhea, nasal congestion, and left eye swelling 1 week ago. Magnetic resonance imaging with contrast reveals a 6.3 cm x 5.6 cm x 4.6 cm heterogeneous enhancing mass centered at the left medial orbital wall with extension into the left ethmoid and maxillary sinuses and destruction of the cribriform plate. Biopsy reveals small round blue cells that are positive for desmin, Myo-D1, and CD56. Cytogenetic analysis reveals hyperdiploidy but no chromosomal translocations. A metastatic evaluation including lumbar puncture does not reveal any detectable disease apart from the incompletely resected primary tumor. Of the following, the stage that should be assigned to this patient's disease is A.stage 1, T2b, Nx, M0, group III B.stage 2, T1b, Nx, M1, group IV C.stage 3, T2b, Nx, M0, group III D.stage 4, T1a, Nx, M1, group IV

The patient in this vignette with progressive eye swelling, nasal congestion, and tumor histopathology positive for desmin, Myo-D1, and CD56, has findings consistent with rhabdomyosarcoma (RMS) located in the orbit and ethmoid and maxillary sinuses. Rhabdomyosarcoma risk groups (low, intermediate, and high) for treatment purposes are based on the combination of the stage assignment (stages 1-4), the clinical group assignment (groups I-IV), and the PAX/FOXO1 fusion gene status. A summary of RMS stages and risk groups is provided by the American Cancer Society at https://www.cancer.org/cancer/rhabdomyosarcoma/detection-diagnosis-staging/staging.html. Stage assignment is determined by tumor confinement to single site (T1 = yes, T2 = no) and tumor diameter size (a = < 5 cm, b = ≥ 5 cm), regional nodal involvement with tumor (N0 = no, N1 = yes, Nx = unknown), and extent of metastatic spread (M0 = no, M1 = yes). Stage 1 is a tumor of any size (a or b), T1 or T2, N0 or N1, without metastases (M0), with a favorable site defined as orbit, head, and neck non-parameningeal sites, genitourinary primaries (excluding the kidney, bladder, and prostate), and the biliary tract. Stage 2 is a tumor smaller than 5 cm (a), T1, N0, M0, and includes unfavorable sites such as bladder, prostate, arm, leg, cranial sites, parameningeal sites, and any body part not listed in stage 1. Parameningeal RMS encompasses tumors arising in the middle ear/mastoid, nasopharynx/nasal cavity, paranasal sinus, parapharyngeal region, and pterygopalatine/infratemporal fossa. Stage 3 includes unfavorable sites and if smaller than 5 cm has nodal involvement (T1a or T2a, N1 or Nx) or if greater than 5 cm may or may not have nodal involvement (T1b or T2b, N0 or N1 or Nx). Stage 4 is a tumor of any size, of any site (a or b, T1 or T2, N0 or N1 or Nx), with metastases (M1) to lung, bone marrow, bone, liver, distant lymph nodes, or muscles, and rarely, meningeal/cerebral spinal fluid spread from parameningeal tumors. A clinical group assignment (groups I-IV) is based on surgical resection or biopsy and subsequent pathologic margins. Group I is a localized RMS that has been completely resected. Group II is a localized RMS with microscopic tumor left at the margins after resection attempt and/or RMS that has spread to regional nodes. Group III is RMS that has not been completely resected with macroscopic tumor remaining and/or RMS that has spread to regional lymph nodes. Group IV is evidence of RMS metastases. Rhabdomyosarcoma is the most common soft-tissue malignancy in children and adolescents. It arises from primitive mesenchymal cells that tend toward myogenesis, but the precise cell of origin remains unknown. Rhabdomyosarcoma is characterized into embryonal and alveolar subtypes. The alveolar form is characterized by the t(2;13) or t(1;13) translocations (PAX3-FOXO1 and PAX7-FOXO1, respectively). Alveolar RMS (ARMS) is a more aggressive variant, and thus is not classified as low risk in the RMS risk classification. However, there is a trend towards treating translocation-negative ARMS like embryonal RMS because they have similar clinical behavior (Williamson D, et al). Embryonal RMS can exhibit loss of heterozygosity at chromosome 11p and frequently is associated with hyperdiploidy of chromosome 2, 7, 8, 11, 12, 13, and 20. In this vignette, the patient has embryonal RMS as evidenced by hyperdiploidy and lack of PAX3-FOXO1 or PAX7-FOXO1 translocations. The head and neck region is the most common primary site for RMS, with 25% of cases occurring in this region. Although this patient's tumor has orbital involvement, which is considered a favorable site, the tumor has regional extension into the paranasal sinuses or a parameningeal site, which is unfavorable and consistent with stage 3 RMS. The tumor size is greater than 5 cm with regional extension consistent with T2b. Nodal status is unknown (Nx). There are no metastases (M0). There is incomplete tumor resection consistent with group III. Thus, the classification of this patient's disease is stage 3, T2b, Nx, M0, group III. The tumor in this vignette is not stage 1 because stage 1 excludes parameningeal sites. Although stage 2 RMS includes cranial/parameningeal sites, the tumors must be smaller than 5 cm, unlike in this vignette. Stage 4 RMS is a tumor of any size or site but with metastatic spread, unlike in this vignette. ​ PREP Pearls Rhabdomyosarcoma risk group assignment (low, intermediate, and high) for treatment purposes consists of a stage assignment, which is determined by tumor site and size (T), nodal involvement (N), and extent of metastatic spread (M), and a clinical group assignment (groups I-IV), which is based on surgical resection or biopsy and subsequent pathologic margins. Parameningeal rhabdomyosarcoma is considered an unfavorable site and includes tumors arising in the middle ear/mastoid, nasopharynx/nasal cavity, paranasal sinus, parapharyngeal region, and pterygopalatine/infratemporal fossa.

A 10-day-old male neonate is brought to medical attention because of persistent oozing from his umbilical cord stump. He had also experienced prolonged oozing from his circumcision and heel stick sites. A small hematoma is present on his right thigh, where he received a vitamin K injection at birth. His prothrombin, activated partial thromboplastin, and thrombin times are markedly prolonged. Of the following, the component MOST likely to have an abnormal activity level in this patient is A.anti-Xa B.factor VIII C.factor XIII D.fibrinogen

The patient's presentation and laboratory findings are classic for congenital afibrinogenemia, a rare, recessively inherited bleeding disorder with an estimated prevalence of one per million. Lack of fibrin clot formation leads to marked prolongation of all coagulation screening tests that are based on fibrin formation, including prothrombin (PT), activated partial thromboplastin time (aPTT), and thrombin time. In addition, in afibrinogenemia, fibrinogen activity and antigen are absent and reptilase time is prolonged. The presence of heparin in the plasma specimen can lead to prolongation of the aPTT and thrombin time but not of the PT. In addition, the patient in this vignette has not been exposed to heparin; therefore, anti-Xa activity would not be abnormal. Although factor VIII deficiency or hemophilia A is a diagnostic consideration in a male patient with a history of the bleeding symptoms described in this vignette, umbilical cord bleeding is not commonly described in hemophilia. Furthermore, factor VIII deficiency does not prolong the PT or thrombin time. Severe factor XIII (FXIII) deficiency is classically associated with bleeding from the umbilical cord or stump. However, FXIII deficiency does not prolong PT, aPTT, or thrombin time. Severe deficiencies of factors V, X, combined V and VIII, and vitamin K-dependent factors (factors II, VII, IX, and X) are other congenital disorders in which umbilical cord bleeding can be a presenting feature. Other less clinically severe inherited fibrinogen disorders include hypofibrinogenemia and dysfibrinogenemia, both of which would result in prolonged thrombin and reptilase times. Significant hypofibrinogenemia, a quantitative defect, and dysfibrinogenemia, a qualitative defect, will typically prolong the PT and/or aPTT, but milder defects may not prolong these times. Thus, direct measurement of fibrinogen activity is the most sensitive test for mild to severe defects while thrombin and reptilase times are also useful screening tests. Because heparin prolongs the thrombin time but not the reptilase time, a prolonged reptilase time can rule out heparin exposure as the cause of a prolonged thrombin time. Fibrinogen or factor I is a 340-kDa glycoprotein synthesized in hepatocytes. It occupies a central role in primary and secondary hemostasis, fibrinolysis, and multiple biological functions, including wound healing and placental implantation. With a typical concentration of 200 to 450 mg/dL (5.9-13.2 µmol/L), it is the most abundant coagulation protein circulating in the plasma. This wide normal range does not differ significantly among neonates, infants, children, adolescents, and adults, but fibrinogen levels do increase gradually with age. Much of the variability in plasma concentration stems from the role of fibrinogen as an acute phase reactant. Stress, tissue injury, infection, and inflammation can lead to increased transcription of the three homologous genes comprising the fibrinogen gene cluster on the long arm of chromosome 4, FGA, FGB, and FGG, which is regulated by proinflammatory cytokines such as interleukin 6. Together, these three genes encode the hexameric fibrinogen molecule, formed from two chains each of A⍺, Bβ, and Ɣ. These chains are linked together via disulfide bonds to form a symmetric, linear molecule with a central E domain, consisting of the amino terminals of the six chains and two end D domains, consisting of the carboxy terminals of three chains from one trimeric half of the molecule. Acute phase responses generate higher circulating fibrinogen levels, which increase adhesive forces between erythrocytes leading to rouleaux formation and increased erythrocyte sedimentation rate. Fibrinogen occupies a central role in primary hemostasis by serving as the major ligand bridging activated platelets via the platelet fibrinogen receptor, glycoprotein IIb-IIIa. This leads to formation of the initial platelet plug (platelet aggregation) at sites of vascular injury. In secondary hemostasis, fibrinogen is the building block for the fibrin clot formed by the coagulation cascade. Activated factor II (thrombin) cleaves fibrinopeptides A and B from the central E domains of fibrinogen molecules to form fibrin monomers. These fibrin monomers polymerize centrally and laterally to form the complex meshwork of the fibrin clot, which is subsequently stabilized via cross-linking by the transglutaminase function of FXIII. Moreover, fibrin regulates coagulation by binding excess circulating thrombin. The attenuation or absence of this thrombin-sequestering function is thought to contribute to the paradoxical observation of increased thrombotic risk in some patients with quantitative or qualitative fibrinogen defects. Fibrin also contains binding sites for plasminogen and tissue plasminogen activator, which is central to the regulation of fibrinolytic activity. Acquired causes of fibrinogen deficiency include liver disease, asparaginase therapy, hemophagocytic lymphohistiocytosis, and consumptive states associated with major trauma, surgery, disseminated intravascular coagulation, and sepsis. Bleeding due to fibrinogen deficiency can be treated with either fibrinogen-containing blood products, such as fresh frozen plasma and cryoprecipitate, or plasma-derived fibrinogen concentrates. The circulating half-life of fibrinogen is typically about 3 to 4 days, but may be shorter in high consumptive states such as disseminated intravascular coagulation. PREP Pearls Fibrinogen (factor I), a hepatically synthesized hexameric glycoprotein with a typical circulating half-life of 3 to 4 days, plays a fundamental role in platelet aggregation, fibrin formation, fibrinolysis, wound healing, placental implantation, and acute phase responses. Thrombin cleaves the short polypeptides fibrinopeptides A and B from the amino-terminal ends of fibrinogen to form fibrin monomers, which polymerize to generate fibrin, the end product of the coagulation cascade. Fibrin formation is the endpoint for the hemostatic functional assays of prothrombin, activated partial thromboplastin, thrombin, and reptilase times. These assay clotting times are prolonged with quantitative defects such as afibrinogenemia, some cases of hypofibrinogenemia, and qualitative defects known as dysfibrinogenemia.

A 16-year-old adolescent boy diagnosed with polycythemia vera 2 years ago is seen in clinic for follow-up. His erythrocytosis had been managed with serial phlebotomies. He takes aspirin daily. He now has worsening splenomegaly and inability to maintain a hematocrit less than 45% despite monthly phlebotomy. Of the following, the BEST therapy to add to his current regimen is A.anagrelide B.busulfan C.hydroxyurea D.ruxolitinib

The severity of polycythemia vera (PV) in this young patient without a history of thrombosis is classified as low risk. Despite appropriate primary treatment with phlebotomy, he has now developed progressive splenomegaly and a hematocrit persistently greater than 45% which necessitates additional treatment. A hematocrit goal of < 45% is associated with a reduction in symptoms including thrombosis. Hydroxyurea is the preferred agent for cytoreduction in symptomatic, progressive PV. Anagrelide is used in the treatment of essential thrombocythemia to reduce the platelet count. Since anagrelide is associated with an increased risk of marrow fibrosis, it is not recommended in the treatment of PV. Busulfan is used in the treatment of PV as a second- or third-line agent when treatment with other agents has failed. Ruxolitinib, an inhibitor of Janus-associated kinases, is approved for the treatment of PV in patients after hydroxyurea treatment fails or is not tolerated. Hydroxyurea is an oral antineoplastic agent or antimetabolite. It is a selective ribonucleoside diphosphate reductase inhibitor, preventing the conversion of ribonucleotides to deoxynucleotides, which interrupts the cell cycle at the G1/S phase. It maintains cells in the G1 phase by interfering with DNA repair, so it is considered a radiation sensitizer. It is metabolized by the liver and renally excreted. Patients with significant renal dysfunction or end-stage renal disease require hydroxyurea dose reductions based on creatinine clearance to avoid toxicities. Gastrointestinal adverse effects of hydroxyurea, such as nausea, vomiting, and diarrhea, often improve with continued drug administration. Myelosuppression, the most common adverse effect, is rapidly reversible with treatment interruption and resumption at a lower dose. Another hematologic side effect is macrocytosis. Melanonychia, black or brown discoloration under the nails, can also be observed during hydroxyurea administration. Hydroxyurea has a long history of use in myeloproliferative neoplasms, with a starting dose of 15 mg/kg/day. Since myeloproliferative neoplasms are rare in children, treatment approaches are based on adult-based guidelines. Polycythemia vera is primarily managed with phlebotomy or erythrocytapheresis, but a subset of patients may require hydroxyurea for progressive splenomegaly. Hydroxyurea can also be used for cytoreduction in essential thrombocythemia and, more rarely, in primary myelofibrosis. Hydroxyurea has been used to treat a variety of other malignant and hematologic diseases. It can be used for leukoreduction in patients with acute leukemia who present with hyperleukocytosis (50-100 mg/kg/day). It is unclear whether this approach or leukapheresis provides any survival benefit over starting conventional chemotherapy. Hydroxyurea has been used to treat hyperleukocytosis that may precede development of differentiation syndrome during treatment of acute promyelocytic leukemia with all-trans retinoic acid. More recently, hydroxyurea has shown efficacy in the treatment of desmoid-type fibromatosis. In children with sickle cell anemia, hydroxyurea reduces the number of vaso-occlusive pain events, the risk for development of acute chest syndrome, and the number of red blood cell transfusions while increasing the fetal hemoglobin level and subsequent hemoglobin level. The BABY HUG study (The Pediatric Hydroxyurea Phase 3 Clinical Trial), which randomized infants (9-18 months of age) with sickle cell anemia to fixed-dose hydroxyurea (20 mg/kg/day) or placebo, found these same beneficial results without an increased risk of serious infection. Based on this information, the National Heart, Lung, and Blood Institute recommends that hydroxyurea therapy be offered to all children with hemoglobin SS or sickle/beta zero thalassemia after 9 months of age. Additionally, the TWiTCH (TCD With Transfusions Changing to Hydroxyurea) study found that a subset of patients with sickle cell anemia who are receiving chronic transfusion therapy as primary stroke prophylaxis based on abnormal transcranial Doppler velocities can be transitioned to hydroxyurea therapy with doses up to 30 mg/kg/day. PREP Pearls Hydroxyurea can be used as a cytoreductive agent in the treatment of myeloproliferative neoplasms such as polycythemia vera, essential thrombocythemia, and primary myelofibrosis. Hydroxyurea causes myelosuppression, which is reversible with treatment interruption and resumption at a lower dose. In children with sickle cell anemia, hydroxyurea reduces the number of vaso-occlusive pain events, the risk for development of acute chest syndrome, and the number of red blood cell transfusions while increasing the fetal hemoglobin level and subsequent hemoglobin level.

A 19-year old male patient with a history of acute lymphoblastic leukemia, currently 13 years from completion of therapy, presents for a fertility consultation. He is interested in his risk for infertility. Which of the following statements is true? a. A semen analysis at this point would provide accurate information about future fertility. b. Males can maintain gonadal function at higher cumulative alkylator dosages compared with females. c. He should have been offered sperm cryopreservation at diagnosis. d. His risk for testosterone deficiency is greater than his risk for infertility. e. Prepubertal status at diagnosis is protective from gonadal injury in males.

A Adolescents and young adults are often concerned about their risk for future infertility. Risk for gonadotoxicity and fertility preservation options differ for males and females. Males are more sensitive to gonadotoxic exposures and have a higher risk of infertility compared to females with equivalent treatment. The Leydig cells that secrete testosterone are fairly resistant to gonadotoxic injury; therefore, males are often able to produce normal levels of testosterone even if they have Sertoli/germ cell damage leading to infertility. In females, the stromal and germ cells are equally affected by therapy; therefore, after girls receive highly gonadotoxic therapy, they are more likely to need hormone replacement to proceed through puberty or maintain menstrual cycles. Prepubertal status is protective from gonadal injury in females; however, this is not true in males. In males, gonadal recovery after therapy can take up to 5 years. After recovery, gonadal function is stable with aging. On the other hand, females may have recovery after treatment leading to a reproductive window prior to premature menopause. A semen analysis is the best method for fertility evaluation in males and would be appropriate to pursue at this time if the patient is interested. Semen cryopreservation is only possible in postpubertal males. Prepubertal males can undergo testicular tissue cryopreservation; however, it is still considered experimental.

A 6-year-old boy presents with rapidly increasing abdominal girth, abdominal pain, and bilious emesis. On examination, his abdomen is distended, and he has mild, diffuse tenderness to palpation without rebound or guarding. Laboratory studies reveal pancytopenia and a markedly elevated LDH. Review of the peripheral smear reveals circulating blasts with oval nucleus, small but distinct nucleoli, and a modest amount of deep blue cytoplasm with prominent vacuoles. CT reveals diffuse abdominal lymphadenopathy with tumor involving the mesentery, retroperitoneum, and kidneys. The patient has not yet received chemotherapy or steroids. Based on your suspected diagnosis, which of the following is an immediate risk for this patient? a. Tumor lysis syndrome b. Superior vena cava syndrome c. Superior mediastinal syndrome d. Spinal cord compression e. Intestinal perforation

A Based on presentation, disease location, blast histology, and cytopenias, the patient likely has Burkitt leukemia. The most common site of disease in sporadic cases of Burkitt lymphoma is the abdomen. Patients are most often boys aged 5 to 10 years who present with nausea, vomiting, abdominal pain or distension, and GI bleeding. Intestinal perforation can occur but is rare. Even before the initiation of chemotherapy, patients with high-grade Burkitt leukemia or lymphoma are at high risk of spontaneous tumor lysis syndrome. Risk factors include large tumor burden, elevated LDH, and renal involvement of disease. Although prophylaxis with recombinant xanthine oxidase reduces this risk, it remains greater than 10% of cases. Although Burkitt lymphoma can involve the mediastinum, presentation in this location is very rare.

A 15-year-old girl presents with stage IIIB nodular sclerosing Hodgkin lymphoma involving thoracic and abdominal lymph nodes. PET imaging shows no other sites of disease. After two cycles of chemotherapy, her lymph nodes have all decreased in size, with the largest nodal aggregate deceasing from 13 cm in its longest axis to 6 cm. Her mediastinal mass has reduced in diameter by half. Her tumor remains PET-avid with maximal standard uptake values in the nodal aggregate of 2.1 compared with 2.8 in the mediastinum. Which of the following most accurately describes her response to therapy? a. Complete metabolic response b. Partial response c. Stable disease d. Refractory disease

A The Deauville score is a 5-point scale used to assess fluorodeoxyglucose (FDG) avidity in both Hodgkin and non-Hodgkin lymphoma. It is internationally accepted as the standard of care for evaluation of response to therapy in Hodgkin lymphoma. Deauville Score 1 = FDG-PET Result: No uptake above background Deauville Score 2 = FDG-PET Result: Uptake ≤ mediastinum Deauville Score 3 = FDG-PET Result: Uptake > mediastinum but ≤ liver Deauville Score 4 = FDG-PET Result: Uptake moderately increased compared with the liver at any site Deauville Score 5 = FDG-PET Result: Uptake markedly increased compared with the liver at any site Deauville Score X = FDG-PET Result: New areas of uptake that are considered unlikely to be related to lymphoma A Deauville score of 1 to 3 is generally accepted as metabolic complete remission. However, to prevent undertreatment, some clinical trials testing reduction of therapy consider a Deauville score of 3 as an inadequate response. A Deauville score of 1 or 2 is always considered a metabolic complete remission, and when it occurs during an interim analysis, it is usually associated with good prognosis with standard care. In patients with the nodular sclerosing subtype of Hodgkin lymphoma, it is common to have a complete metabolic response despite residual mass.

5-year-old boy presents with 1 week of fevers, weight loss, and swollen abdomen. Complete blood counts are normal, but serum chemistries show hyperkalemia, hyperuricemia, hyperphosphatemia, and LDH of 3,900 IU/L (four times the upper limit of normal). CT shows diffuse mesenteric lymphadenopathy. Biopsy shows malignant cells that express CD10, CD19, CD20, CD22, and surface IgM. Ninety-nine percent of cells are positive for Ki-67+. Cytogenetics reveal a t(8;14) translocation. CNS and bone marrow are negative for malignancy. For this disease, which of the following factors influence prognosis? a. LDH b. Uric acid c. B symptoms (fevers, night sweats, weight loss) d. t(8;14) translocation e. CD20 expression

A This patient has stage III Burkitt lymphoma, as indicated by diffuse abdominal involvement. Non-Hodgkin lymphomas (NHLs) are staged according to the St. Jude (Murphy) system. High LDH is a negative prognostic feature in Burkitt lymphoma in both Berlin-Frankfurt-Munster and French-American-British Society of Pediatric Oncology trials. Elevated uric acid indicates spontaneous tumor lysis but is not predictive of response to therapy, per se. The Ann Arbor stages of Hodgkin lymphoma are subclassified into "A" or "B" based on the absence or presence of systemic systems. Although B symptoms are prognostic in Hodgkin lymphoma, they are not used for risk stratification of NHL. cMYC is a transcription factor on chromosome 8 that acts like an oncogene and drives proliferation. Common translocations in Burkitt lymphoma are t(8;14)(q24;q32) IgM-cMYC in about 80% of cases, t(2,8)(p11;q24) IgK-cMYC in 15% of cases, and t(8;22)(q24;q11) IgL-cMYC in 5% of cases. Although important for diagnosis, the translocations are not used in risk stratification. Finally, although many Burkitt lymphomas express the surface marker CD20, and immunotherapy using antibodies against CD20 has been shown to improve outcomes in adults with CD20-positive lymphomas such as diffuse large B-cell lymphoma, the presence of CD20 on pediatric Burkitt lymphoma is not a prognostic marker.

A 3-year-old boy presents to your office with a 4 cm × 5 cm raised, erythematous, purplish mass of the scalp on the right side of his face. CBC is normal, and imaging shows no other sites of disease. Biopsy reveals small round blue cells that stain for CD10, CD19, CD22, CD24, CD79a, and nuclear TdT. The diagnosis of B-lymphoblastic lymphoma is made. Which clinical feature of this patient confers a significantly increased risk of relapse? a. Age at diagnosis b. WBC count less than 10,000 at diagnosis c. Hyperdiploid cytogenetics d. Extranodal (skin) involvement

A Unlike in acute lymphoblastic leukemia in which patients are older than 10 years at diagnosis, in B lymphoblastic lymphoma, patients younger than 4 years at diagnosis have a significantly higher risk of relapse (about 45%) compared with children 4 to 15 years of age (about 5%). Currently, WBC count and hyperdiploid cytogenetics are not used for risk stratification. Skin is involved in about 37% of cases of B-lymphoblastic lymphoma and is not prognostic.

A 12-year-old boy presents with 4 months of painless swelling in his groin and neck. During the past 6 weeks he has had fevers, fatigue, and a 5-lb weight loss. He has been treated with 2 weeks of clindamycin, but lymphadenopathy has not resolved. Physical examination reveals painless inguinal, femoral, cervical, and axillary lymphadenopathy. Lymph nodes are firm, nontender, and nonmobile. A needle biopsy is performed and reveals a hematolymphoid neoplasm that expresses CD30 and evidence of T-cell receptor rearrangement. What will additional studies most likely reveal? a. t(2;5)(p23;q35) chromosomal translocation resulting in the nucleophosmin (NPM)-ALK fusion gene b. t(8;14)(q24;q32) chromosomal translocation involving the cMYC oncogene and the immunoglobulin heavy chain locus c. Expression of high levels of BCL-6 d. Reed-Sternberg cells

A. This patient has an anaplastic large-cell lymphoma (ALCL), a mature T-cell lymphoma. The majority of ALCLs are characterized by the t(2;5)(p23;q35) chromosomal translocation and NPM-ALK fusion gene. The NPM gene promoter results in overexpression of the ALK kinase in lymphoid cells. Hodgkin lymphoma, Burkitt lymphoma, and diffuse large B-cell lymphoma (DLBCL) are all mature B-cell lymphomas. Burkitt lymphoma cells contain a translocation involving the cMYC oncogene and one of the following: the immunoglobulin heavy chain locus t(8;14)(q24;q32), the kappa immunoglobulin light chain gene locus t(2;8)(p11;q24), or the lambda immunoglobulin light chain gene locus t(8;22)(q24;q11). Although approximately one-third of pediatric DLBCLs have translocations associated with cMYC, DLBCL has no specific, diagnostic cytogenetic abnormalities. Most cases have complex karyotypes with three or more cytogenetic aberrations. Pediatric DLBCL can express high levels of BCL-6 and CD10. Although some DLBCLs express CD30, they do not express T-cell markers. Reed-Sternberg cells are the malignant cell of Hodgkin lymphoma. Although Reed-Sternberg cells can express CD30, they do not express T-cell markers.

A 10-year-old boy with homozygous SS sickle cell anemia was found to have abnormal transcranial Doppler velocity of the right middle cerebral artery (212 cm/s) at age 3 years. He has not had a clinical stroke, transient ischemic attack, or seizure. He has been treated with chronic transfusion therapy for 7 years. His transcranial Doppler velocities are now normal. Findings from brain magnetic resonance imaging/magnetic resonance angiography reveal no vasculopathy, stroke or severe stenosis. Of the following, the MOST accurate statement regarding his subsequent therapy is A.transfusions can be discontinued now based on brain imaging results B.transfusions must be given for a minimum of 10 years C.transfusions are to be continued indefinitely D.transition from transfusions to hydroxyurea therapy can be considered

According to the most recent results from the "Transcranial Doppler (TCD) With Transfusions Changing to Hydroxyurea" (or TWiTCH) randomized trial, a subset of children with severe sickle cell disease (SCD) with a history of abnormal TCD velocity who have been treated with chronic transfusion therapy can be safely transitioned to hydroxyurea therapy when their TCD velocities have normalized. The child in this vignette fulfills the criteria for the subset of individuals who were successfully transitioned from transfusions to hydroxyurea. More details about TWiTCH follow the historical clinical trials that preceded these current recommendations. Children with severe SCD, defined as homozygous SS or sickle-beta zero thalassemia, have an increased risk of stroke. Historically, prior to screening and the appropriate therapy of routine packed red blood cell transfusions, approximately 11% of children with severe SCD experienced a stroke by 20 years of age. Screening with TCD identifies children at highest risk of experiencing stroke. High risk individuals are those with abnormal TCD velocities defined as ≥ 200 cm/s if nonimaging TCD is used, or ≥ 185 cm/s if TCD is used with imaging in the middle cerebral artery or internal carotid artery, on 2 separate readings at least 1-2 weeks apart. Findings in the initial or "Standardized Treatment of Pulmonary Exacerbations I" (or STOP-1) clinical trial showed that chronic transfusion therapy in high-risk patients could reduce the risk of first stroke by 90%. These results led to chronic transfusion therapy becoming the standard of care for children with SCD and abnormal TCD velocities. The following STOP-2 trial examined whether children receiving chronic transfusion therapy whose TCD values normalized could safely discontinue transfusion therapy. Unfortunately, in the discontinuation arm, 2 patients had overt strokes, and a significant number of other patients reverted back to abnormal TCD velocities. Therefore, after STOP-2, children with abnormal TCD velocities faced a future of indefinite chronic transfusion therapy. The TWiTCH randomized trial sought to determine whether children with severe SCD and initially abnormal TCD velocities could safely make a transition from chronic transfusion therapy to hydroxyurea therapy. The TCD velocities were required to have normalized while on transfusion therapy at the time of enrollment in this study. Participants who were randomly assigned to continue transfusion therapy, also continued iron chelation with deferasirox. Participants who were randomly assigned to receive hydroxyurea, started by receiving 20 mg/kg/day of hydroxyurea, with doses being escalated to the maximally tolerated, while concurrently being weaned from transfusion therapy. Once the maximum tolerated hydroxyurea dose had been achieved, and transfusion was discontinued, participants underwent serial phlebotomy to reduce iron overload. No participant in either cohort reverted from normal to abnormal TCD velocities during the study. The study authors concluded that hydroxyurea was "non-inferior for the maintenance of normal TCD velocities" compared with chronic transfusion therapy for primary stroke prevention. Before a child with severe SCD and an abnormal TCD who receives chronic transfusion therapy can be considered an optimal candidate to switch to hydroxyurea therapy, the clinician must carefully address the inclusion and exclusion criteria used in the TWiTCH study. At entry, the average subject was older than 9 years and had a mean TCD velocity of 145 cm/s. Additional eligibility criteria required a minimum of 1 year of transfusions, but on average participants had received transfusions for 4.5 years. Participants with brain magnetic resonance angiography evidence of grade 4 or higher vasculopathy, defined as moderate stenosis in > 2 arterial segments or severe stenosis/occlusion in ≤ 2 segments, were excluded from the study. Based on fulfilling these criteria, the child in this vignette could be considered a candidate to transition from transfusion to hydroxyurea. Limitations of the TWiTCH trial include reproducing the extra vigilance of monthly clinical evaluation and TCD performed every 12 weeks. Additionally, the study only followed participants for 24 months, so longer-term data are still needed to determine long-term safety. Of note, Bernaudin and colleagues reported on 45 patients in France with abnormal TCD velocities who switched from chronic transfusions with a mean duration > 2 years to hydroxyurea therapy. Thirteen of those patients reverted back to abnormal TCD velocities and returned to chronic transfusion therapy for primary stroke prophylaxis. In their analysis, baseline (before initiation of transfusion therapy) absolute reticulocyte count of 400 × 103/µL (400 × 109/L) or greater was highly predictive of reversion to abnormal TCD velocities. The mean duration of hydroxyurea therapy was more than 3 years in this population. PREP Pearls In a subset of children with severe sickle cell disease and initial abnormal transcranial Doppler velocities, hydroxyurea therapy was noninferior to chronic transfusion therapy for primary stroke prevention after initial treatment with chronic transfusions resulted in normalization of transcranial Doppler velocities. For patients who have sickle cell disease and a history of abnormal transcranial Doppler velocities and who are appropriate for transitioning from chronic transfusions to hydroxyurea, the hydroxyurea is escalated to the maximum tolerated doses before transfusion therapy is discontinued. A portion of the population of patients with sickle cell disease transitioned to hydroxyurea appears to be at risk for reversion to abnormal transcranial Doppler (TCD) velocities. Increased TCD monitoring throughout transition and afterward is essential to identify these patients promptly.

A 7-year-old boy with aplastic anemia received an HLA-matched, unrelated donor, hematopoietic stem cell transplant 50 days ago. He has had the acute onset of a maculopapular rash confined to his front torso, back torso, and both legs (Figure 1). He has a total bilirubin level of 2.5 mg/dL (42.8 µmol/L). He has no nausea, vomiting, or anorexia. His stools have been formed with bowel movements occuring twice per day without blood. Of the following, the grade of graft-vs-host disease that BEST categorizes this patient is A.grade 0 B.grade II C.grade III D.grade IV

Acute graft-vs-host disease (GVHD) is a result of donor T lymphocytes infiltrating the recipient's healthy tissues, potentially resulting in skin, liver, and gastrointestinal inflammation and destruction. Traditionally, acute GVHD occurs within 100 days after hematopoietic stem cell transplant (HSCT), and chronic GVHD occurs after 100 days. However, there may be overlap of both conditions around 100 days. If signs and symptoms consistent with acute GVHD are first seen after day 100, it is still labeled acute GVHD. In addition to the timeline of symptom development, the pattern of affected organ systems can help distinguish acute from chronic GVHD. In acute GVHD, typically the skin, liver, and upper and lower gastrointestinal tracts are involved. In chronic GVHD, any organ may be affected. Determination of the GVHD grade allows for standardization of records and appropriate clinical care. To determine the grade (grades 0-IV) of acute GVHD severity, each affected organ (skin, liver, upper and lower gastrointestinal tract) is first staged (stages 0-4), and then these stages are combined to develop the grade (Table). Grade 0 GVHD is defined as no involvement of any organ (all stage 0). Grade I GVHD is defined as stage 1 of skin (maculopapular rash < 25% body surface area [BSA]) or stage 2 of skin (maculopapular rash < 50% BSA) without involvement of liver or gastrointestinal tracts. Grade II GVHD is defined as stage 3 skin GVHD with stage 1 liver (defined as bilirubin 2-3 mg/dL [24-51 µmol/L]) or stage 1 gastrointestinal involvement (defined as stool output of 500-999 mL/day for an adult and 10-19.9 mL/kg/day for a child) or persistent nausea, vomiting, or anorexia, with a positive upper gastrointestinal tract biopsy. Any involvement of liver (stages 1-4) or the upper and/or lower gastrointestinal tract (stages 1-4) automatically is designated grade II GVHD or higher. Grade III GVHD is defined as any skin stages 0 to 4, with stage 2 to 3 liver or stages 2 to 4 gastrointestinal involvement. Grade IV GVHD is defined as stage 4 skin involvement (generalized erythroderma plus bullous formation) or stage 4 liver involvement and any gastrointestinal stages 0 to 4. In this vignette, the patient developed a rash less than 100 days after HSCT, which is consistent with acute skin GVHD. His skin rash is without bullae or desquamation. Using the "rule of 9's" method in calculating BSA affected by GVHD (Figure 2), he has GVHD on his front (18% BSA), back (18% BSA), and both legs (36% BSA). Therefore, he has skin GVHD covering more than 50% BSA, consistent with stage 3 skin disease. The elevated bilirubin level greater than 2 mg/dL (34 µmol/L) but less than 3 mg/dL (51 µmol/L) is consistent with stage 1 liver GVHD. He has no nausea, vomiting, or anorexia, and his stools are normal, formed, and non-bloody, which are consistent with no GVHD of the upper or lower gastrointestinal tracts, respectively. The combined findings from the skin (stage 3), liver (stage 1), and upper/lower gastrointestinal tracts (stage 0) are consistent with grade II GVHD (Table). Grade II or higher GVHD requires systemic therapy to prevent worsening GVHD associated with increasing morbidity and mortality. The differential diagnosis of a maculopapular rash in a patient who has undergone HSCT includes viral infection, drug reaction, and GVHD. Hyperbilirubinemia can have multifactorial causes after HSCT, including sinusoidal obstructive syndrome, infection, adverse effects of medications, and GVHD. Other conditions mimicking gastrointestinal GVHD include infection and medications. Tissue biopsies of affected organs are frequently obtained and require careful interpretation to differentiate these possible etiologies. PREP Pearls To standardize and provide appropriate clinical care, the grade of acute graft-vs-host disease severity (grades 0-IV) is determined by first staging each affected organ (skin, liver, upper/lower gastrointestinal tract) (stages 0-4) and then combining these stages. Findings associated with acute graft-vs-host disease, such as maculopapular rash, loose watery stools, or hyperbilirubinemia, can mimic other conditions (eg, infection or medication adverse effects) observed after hematopoietic stem cell transplant. Although acute graft-vs-host disease is defined when skin, liver, or gastrointestinal symptoms occur less than 100 days after hematopoietic stem cell transplant, similar symptoms that occur greater than 100 days after transplant are still defined and treated as acute graft-vs-host disease. ABP Content Specifications(s)/Content Area Recognize the clinical and laboratory manifestations of acute graft-versus-host disease

A 15-year-old adolescent boy with sickle cell disease has a 2-year history of monthly packed red blood cell transfusions to prevent recurrence of a cerebrovascular accident. He has been appropriately treated for red blood cell alloantibodies and iron overload. He is currently 29 days after an HLA-identical sibling hematopoietic stem cell transplant with a myeloablative conditioning regimen followed by a bone marrow graft infusion of 8.5 × 107 total nucleated cells per kilogram recipient body weight. He requires packed red blood cell transfusions every 10 to 14 days and platelet transfusions every 4 to 5 days. His absolute neutrophil count had reached 400/mm3 but is now 100/mm3 despite daily granulocyte colony-stimulating factor injections. The patient is receiving appropriate antifungal and antiviral prophylaxis. He is afebrile, is in no respiratory distress, and has no rash or diarrhea. Of the following, the MOST likely cause of graft failure is due to the A.bone marrow as the graft source B.HLA-identical sibling donor C.red blood cell alloimmunization D.total nucleated cell dose of the graft

Although allogeneic hematopoietic stem cell transplant (alloHSCT) is a curative option for various malignant and nonmalignant diseases, an associated acute complication is donor graft failure. Donor stem cell engraftment is defined as sustained attainment of an absolute neutrophil count of greater than 0.5 × 109/L in the recipient's peripheral blood, without supportive granulocyte colony-stimulating factor. In the absence of relapse of the primary underlying disorder, graft failure is defined as either the Lack of initial donor stem cell engraftment by Day + 28 after alloHSCT with either peripheral blood or bone marrow progenitor grafts OR Day +42 after alloHSCT with umbilical cord blood graft OR 2) Loss of initial donor stem cell engraftment (also known as secondary graft failure) The child in this vignette with an absolute neutrophil count less than 500/mm3 requiring frequent red blood cell (RBC) and platelet transfusions beyond day +28 after alloHSCT, is experiencing graft failure. Risk factors for graft failure vary. Overall, graft failure occurs as a result of residual recipient T cell response directed against the donor graft or hematopoietic stem cells. In patients who have received large numbers of RBC transfusions prior to alloHSCT, the development of alloimmune sensitization to donor RBC antigens resulting in recipient RBC antibodies can result in graft failure, as in this vignette. Some risk factors for graft failure are related to the manipulation of the graft or the graft itself. Although donor cytotoxic T cells help facilitate donor stem cell engraftment, absence of donor T cells in the infused graft due to in vivo or ex vivo T cell depletion methods, also results in high risk of graft failure. Another risk factor is inadequate total nucleated cell dose of the infused graft. To reduce the risk of graft failure after bone marrow progenitor grafts, a minimum donor stem cell dose of 2 × 107 total nucleated cells per kilogram recipient weight is considered adequate. Higher total nucleated cell doses may be required for different donor source types, conditions treated, and conditioning regimens. Donor graft sources with the greater to lesser risk of graft failure in descending order are cord blood, bone marrow, and peripheral blood derived. Other risk factors for graft failure include the degree of donor and recipient matching as well as the underlying diagnosis. Greater disparity of human leukocyte antigen matching,, using haploidentical donors versus matched unrelated donors or matched sibling donors results in greater risk of graft failure. Baseline RBC antigen and ABO mismatching have also been implicated in graft failure. Reduced-intensity conditioning regimens have been associated with greater risk of graft failure because of residual recipient T cells. Certain primary diagnoses such as severe aplastic anemia, hemoglobinopathies, juvenile myelomonocytic leukemia, and myelodysplastic syndromes have also demonstrated higher rates of graft failure after HSCT. Secondary causes of graft failure include viral infections acquired from the donor graft, reactivated during post-HSCT immunosuppression, or from community exposures. Outcomes of graft failure range between autologous recovery of hematopoiesis, permanent or partial marrow aplasia with varying severities of pancytopenia, and/or transfusion dependence. Some affected individuals require repeat HSCT. PREP Pearls Graft failure/rejection mediated by residual host T cells acting against donor hematopoietic cells after hematopoietic stem cell transplant is considered primary when the absolute neutrophil count never rises above 0.5 × 109/L or secondary when the absolute neutrophil count declines after rising above 0.5 × 109/L. Risk factors for graft failure are human leukocyte antigen disparity (haploidentical > matched unrelated donor > matched sibling donor); ABO mismatching; reduced-intensity conditioning regimens; primary diagnoses of severe aplastic anemia, hemoglobinopathies, juvenile myelomonocytic leukemia, or myelodysplastic syndromes; graft source (cord blood > bone marrow > peripheral blood derived); and T cell depletion (either in vivo or ex vivo).

An 11-year-old boy has recurrent fever, fatigue, bruising, and gum bleeding. He appears tired and has pallor, hepatosplenomegaly, and pancytopenia. A bone marrow evaluation shows the presence of 82% blasts. Immunohistochemistry and flow cytometry are performed, and the results of cytogenetics are pending. The blast cells are positive for CD34, TdT, CD19, PAX5, and myeloperoxidase. Of the following, the MOST likely diagnosis is A.acute B-lymphoblastic leukemia B.acute myeloid leukemia C.acute undifferentiated leukemia D.mixed-phenotype acute leukemia

Although most acute leukemias can be classified as acute lymphoblastic leukemia (ALL) or acute myeloid leukemia (AML), there are rare cases in which both lymphoid and myeloid features are present. These cases of mixed-phenotype acute leukemia (MPAL) comprise 2% to 5% of all acute leukemias. Classification systems from the World Health Organization (WHO) and the European Group for Immunological Characterization of Acute Leukemias (EGIL) are commonly used internationally to define MPAL, and it is controversial as to which system is superior. The WHO system defines MPAL as an acute leukemia containing 2 distinct populations of blasts fulfilling diagnostic criteria for 2 different lineages (bilineal leukemia) or a single blast population expressing mixed markers (biphenotypic leukemia). While it leans heavily on flow cytometry, the WHO system also uses morphological and cytochemical features of the bone marrow in diagnosis. Some key markers used to indicate lineage in the WHO system are as follows: Myeloid: myeloperoxidase positivity T-lineage: cytoplasmic CD3 or surface CD3 B-lineage: CD19, CD79a Most MPAL cases are bilineal, and the approximate frequencies are as follows: B-lymphoid/myeloid, 60% T-lymphoid/myeloid, 33% T-/B-lymphoid, 4% Trilineage, 3% The WHO criteria has a larger category of acute leukemia of ambiguous lineage, which is comprised of MPAL and acute undifferentiated leukemia (AUL). Cases without lineage-specific markers are termed AUL, which is associated with a poor prognosis. These usually express CD34, HLA-DR, CD38, and sometimes TdT but no specific myeloid/lymphoid markers. In the vignette, the patient's blasts are positive for CD19 (suggesting B-lymphoid origin) and myeloperoxidase (suggesting myeloid origin), which is suggestive of B-lymphoid/myeloid MPAL. The lack of CD3 expression excludes T-lineage leukemia. It is helpful in monitoring disease response to know whether the disease is bilineal or biphenotypic by morphology and flow cytometry. The EGIL classification involves a scoring system that incorporates the number and specificity of certain markers for myeloid and lymphoid origin. The system defines positivity by flow cytometry on at least 20% of blasts for surface markers and 10% for cytoplasmic markers. The EGIL system is more complex and less stringent than the WHO system, but it is less reliant on MPO to indicate myeloid origin. Because MPAL is a rare diagnosis, the literature is somewhat limited to case reports and small case series, which makes it difficult to draw conclusions about clinical presentation or best treatment approach. However, a 2018 systematic review and meta-analysis of the literature revealed trends and suggested an initial treatment strategy corroborated by individual series. With regard to clinical presentation, hyperleukocytosis and central nervous system involvement are not particularly common as compared to other acute leukemias. Hyperleukocytosis occurs in about 20% of patients with MPAL, and the median white blood cell count at diagnosis is 12,000/μL to 28,000/μL (12.0-28.0 × 109/L). Central nervous system involvement at diagnosis is seen in less than 20% of patients. There is no defining recurrent cytogenetic abnormality, but 2 genetic subclassifications of MPAL are included in the WHO system: BCR-ABL1 fusion and KMT2A rearrangement. BCR-ABL1 is the most common cytogenetic finding, occurring in 17% to 35% of adult MPAL cases and 3% of pediatric MPAL cases. Most of these patients have the B-lymphoid/myeloid phenotype. KMT2A rearrangements were seen in 10% or less of patients with MPAL and are more common in pediatric patients. KMT2A rearrangements are the most frequent abnormality seen in cases of lineage switch after chemotherapy, although MPAL is not always present at initial diagnosis. Historically, the selection of treatment was often based on which lineage was more dominant. However, recent literature shows that ALL induction therapy is associated with a complete remission rate that is 3-fold higher than AML therapy. Some poor responders to AML therapy may have a complete remission when switched to an ALL regimen. Compared to other acute leukemias, MPAL may be associated with overall worse outcomes. It is postulated that this may be related to chemoresistance and the adaptive nature of the blasts. Combined adult and pediatric data show a 3-year overall survival of 44%, which might be negatively impacted by the inclusion of elderly patients who have a particularly poor prognosis. Highest survival appears to occur in pediatric patients and has been reported upwards of 50%. The literature suggests that the B-lymphoid/myeloid phenotype has a better complete remission rate than the T-lymphoid/myeloid phenotype. Outcomes of MPAL appear to be better than those of AUL. The role of hematopoietic stem cell transplant (HSCT) remains controversial for MPAL. In meta-analyses, HSCT appears to be associated with a survival advantage, but the data are not clear and may be skewed by the fact that only patients who survived long enough could proceed to HSCT. At this time, it is unclear if HSCT has the best role as part of frontline therapy or salvage therapy. PREP Pearls Mixed-phenotype acute leukemia is a rare acute leukemia containing 2 distinct populations of blasts (bilineal leukemia) or a single blast population expressing mixed markers (biphenotypic leukemia). Mixed phenotype acute leukemia likely has a worse prognosis than other acute leukemias, but children have better outcomes than adults. In patients with mixed-phenotype acute leukemia, regimens based on acute lymphoblastic leukemia therapy are associated with better outcomes than regimens based on acute myeloid leukemia therapy. ABP Content Specifications(s)/Content Area Mixed Phenotype Acute Leukemia - diagnosis and treatment

A 17-year-old boy presents with 2 weeks of worsening fatigue, cough, and shortness of breath. He is tachypneic, but oxygen saturation is normal. X ray reveals a large anterior mediastinal mass and a large left-sided pleural effusion. CT imaging shows minor airway compression of the distal trachea and left main stem bronchus and no pericardial effusion. What is the most appropriate next step in this patient's care? a. Intubation for airway protection b. Thoracentesis c. Radiation to the mediastinal mass d. Empiric steroid therapy e. Incisional biopsy of the mediastinal mass f. Bone marrow biopsy followed by cytoreductive therapy with steroids

B Patients with large anterior mediastinal masses are at risk of superior vena cava syndrome and superior mediastinal syndrome. In such circumstances it is important to obtain diagnostic tissue in the least invasive way possible. In this case, although the patient does have a large mediastinal mass, he has only minimal airway compression when recumbent, and so it is most likely that the pleural effusions are the cause of his symptoms. Draining the pleural effusion via thoracentesis likely will alleviate his respiratory symptoms, and the pleural fluid may contain sufficient material for diagnosis. Bone marrow biopsy, although also relatively noninvasive, may or may not yield the diagnosis in the presence of a normal CBC and will not relieve the patient's symptoms. Intubating a patient with a mediastinal mass is inappropriate because it can be difficult to intubate past an airway obstruction, and the anesthesia can increase compression of the airway or superior vena cava. Thus, incisional biopsy of the mediastinal mass, because it requires greater sedation than a thoracentesis, is incorrect as the first step. Empiric therapy with either steroids or radiation to the mediastinal mass is inappropriate (in this case) because the mass is not causing symptomatic compression of the superior vena cava or airways, and either therapy may make diagnosis more difficult.

A 7-year-old presents with fatigue and abdominal pain. Physical exam reveals a pale child with a distended abdomen. CT scan shows a large abdominal mass encasing bowel and lesions in the kidneys, adrenals, and pancreas. Chemistries reveal elevated LDH, uric acid, and creatinine. Which of the following is the most likely explanation for the child's laboratory test results? a. Sepsis b. Tumor lysis syndrome c. Cytokine release from tumor cells d. Hypovolemic shock

B Tumor lysis syndrome occurs when renal function cannot sufficiently eliminate the byproducts of rapid tumor cell death. Laboratory abnormalities include hyperuricemia, hyperkalemia, hyperphosphatemia, and associated hypocalcemia. Clinically, hyperuricemia and hyperphosphatemia can result in formation of crystals in the renal tubules and result in renal failure. Hyperkalemia can result in fatal arrhythmias, and hypocalcemia can cause muscle cramps, tetany, laryngospasm, prolonged QTc, and torsade de pointes. Tumor lysis syndrome usually occurs 24 to 72 hours after initiation of therapy. However, bulky, rapidly growing tumors such as Burkitt lymphoma or LL can present with spontaneous tumor lysis. This is a medical emergency, and management includes frequent monitoring, aggressive hydration, careful electrolyte management, and uric acid reduction by xanthine oxidase inhibition or administration of recombinant urate oxidase.

A 4-year-old boy presents with a 6-week history of swelling below his jaw that has been slowly growing despite a 2-week course of antibiotics. Examination reveals a firm, fixed, nontender, 3-cm lymph node. Biopsy is performed. Histology shows nodular collections of small lymphocytes and histiocytes with scattered mononuclear cells, with convoluted irregular nuclei and occasional small nucleoli. By immunohistochemistry these cells are positive for CD19, CD20, CD79a, CD45, and BCL-6 but are negative for CD15, CD30, and EBV markers. Fluorescence in situ hybridization for MYC translocations is negative. Which of the following is the most likely diagnosis? a. Classic Hodgkin lymphoma b. Nodular lymphocyte-predominant Hodgkin lymphoma (nLPHL) c. Burkitt lymphoma d. Diffuse large B-cell lymphoma e. Lymphadenitis from atypical mycobacteria

B nLPHL is a B-cell lymphoma that is significantly different from Hodgkin lymphoma. nLPHL accounts for 10% to 20% of pediatric Hodgkin lymphoma; usually presents with early stage disease (IA, IIA); and has a male predominance, indolent course, and good prognosis but can have late and occasionally multiple relapses. The malignant cells of nLPHL are lymphocyte-predominant cells (formerly known as lymphocytic and histiocytic variants of Reed-Sternberg cells) and show a phenotype consistent with germinal center B cells. By immunohistochemistry the malignant cells will be positive for CD20, CD45, CD79a, PAX5, and BCL-6 but negative for CD10, CD15, and CD30. They also will express RNA transcription factors octamer-binding transcription factor 2 (Oct-2) and B-cell Oct-binding protein 1 immunoglobulin light B.1). In contrast, classic Hodgkin lymphoma usually shows Reed-Sternberg cells and has an immunophenotype that includes CD15+, CD30+, and stains for EBV antigens in 40% to 50% of cases. Patients with Burkitt lymphoma have translocations t(8;14)(q24;q32) in 70% to 80% of patients and t(2;8)(p12;q24) or t(8;22)(q24;q11) in 10% to 15% of patients. These translocations involve the cMYC oncogene and the immunoglobulin heavy chain, chain gene loci, respectively. Patients with DLBCL also can express BCL-6 and MYC. Both Burkitt and DLBCL are aggressive, mature B-cell lymphomas; express B-cell markers; and are readily detectable by flow cytometric analysis.

You have a 7-year-old male treated for a hematolymphoid malignancy relapse 10 months after completion of therapy. Which primary diagnosis carries the least favorable outcome at relapse? a. Average risk B-lymphoblastic leukemia b. Stage III B-lymphoblastic lymphoma c. Stage IIIA Hodgkin lymphoma d. Stage III anaplastic large-cell lymphoma

B. Based on published consortium trials, patients with stage III to IV B-lymphoblastic lymphoma have cure rates ranging from 80% to 95% when treated with standard chemotherapy. However, prognosis following relapse is abysmal. In contrast, the remaining diseases can often be cured with standard chemotherapy, which may or may not include autologous or allogeneic hematopoietic stem cell transplantation. In addition, several novel therapies (including small molecule inhibitors, naked antibodies, antibody drug conjugates, bispecific antibodies, or chimeric antigen receptor T-cells) have been shown to be highly active in the other diseases. Whether novel agents improve survival in relapsed B-lymphoblastic lymphoma remains to be seen.

An 11-year-old boy has been diagnosed with stage IIIB Hodgkin lymphoma with involvement of the mediastinum and para-aortic, iliac, and inguinal nodes. Your treatment plan includes cycles of multiagent chemotherapy and involved-node radiation. The parents are concerned about infertility because their son is too young for sperm donation before therapy. Which part of therapy would be most likely to cause infertility in this patient? a. Alkylating agents b. Radiation c. Bleomycin d. Corticosteroids e. Anthracycline

Because the testes are out of the direct field of pelvic radiation, permanent azoospermia is rarely associated with radiation therapy for Hodgkin lymphoma. However, alkylating agents, such as nitrogen mustard, cyclophosphamide, ifosfamide, and procarbazine, are very gonadotoxic to male patients and can result in azoospermia and infertility, depending on the dose. In contrast, female infertility is more strongly associated with radiation (although oophoropexy can be performed to spare some of the radiation effect). Compared with female fertility, male fertility is much more sensitive to alkylating agents, and therefore gender-based therapies have been developed for Hodgkin lymphoma. An example of this is the substitution of etoposide for procarbazine for male patients. Alkylating agents and topoisomerase II inhibitors have been associated with secondary leukemias and myelodysplasia. Radiation has been associated with thyroid, skin, and breast cancer (particularly in adolescent girls treated for Hodgkin lymphoma). Radiation used to treat Hodgkin lymphoma also has been associated with hypothyroidism, cardiovascular disease (including myocardial infarction and stroke), and spinal growth abnormalities. Bleomycin has been associated with pulmonary fibrosis. Corticosteroids have been associated with cataracts and osteopenia. Anthracyclines are associated with cardiomyopathy.

An 18-year-old man with history of multiply relapsed leukemia is interested in discussing his risk of infertility due to cancer treatment. He underwent two hematopoietic stem cell transplants as part of his treatment. After reviewing his previous treatment, which of the following agents contributes to his risk of azoospermia? a. Cytarabine b. Methotrexate c. Busulfan d. Fludarabine e. E Etoposide

C Risk of infertility is one the most common concerns voiced by young adult survivors of childhood cancer. Survivors treated with hematopoietic stem cell transplant have a high risk of gonadal dysfunction due to conditioning regimens and previous treatments. Surgery, radiation, and chemotherapy that affect the hypothalamic-pituitary-gonadal axis and reproductive organs increase the risk of infertility. Treatment factors that affect this axis and may lead to infertility include surgical removal of reproductive organs (oophorectomy/orchiectomy), alkylating agent chemotherapy, hypothalamic-pituitary radiation, and radiation to the reproductive system. Pelvic or spinal surgery may also lead to sexual dysfunction, including retrograde ejaculation in men after retroperitoneal lymph node dissection. Radiation fields that may affect reproductive organs in women include abdominal, pelvic, lumbosacral spine, and total body. In men, these fields include pelvic, testicular, sacral spine, and total body. Alkylating agents used for treatment of childhood cancer include busulfan, carmustine, chlorambucil, cyclophosphamide, ifosfamide, lomustine, mechlorethamine, melphalan, procarbazine, thiotepa, cisplatin, carboplatin, dacarbazine, and temozolomide.

A 25-year-old girl with a history of Hodgkin lymphoma presents to the oncology late effects clinic. She was treated with nitrogen mustard, vincristine, procarbazine, and prednisone followed by 25.5 Gy modified mantle radiation at age 15. What screening is needed for subsequent malignancies? a. Annual CBC b. Annual urinalysis c. Annual mammography and breast MRI d. Colonoscopy every 5 years e. Annual thyroid ultrasounds

C Subsequent malignancies are the leading cause of nonrelapse late mortality for childhood cancer survivors. The incidence of subsequent malignancies increases with age, and Hodgkin lymphoma survivors are at particularly high risk. Female patients who have received radiation to the breast are at increased risk for breast cancer. The Children's Oncology Group Long-Term Follow-Up Guidelines recommend annual mammography and MRI starting at age 25 or 8 years after breast radiation exposure, whichever occurs last. Although procarbazine and nitrogen mustard increase the risk of myelodysplasia/acute myeloid leukemia, the recommendations are to perform an annual targeted history and physical examination. In addition, the patient is at elevated risk of thyroid cancer due to exposure to radiation. Monitoring for thyroid cancer should be performed annually with a physical examination. The risk of thyroid cancer should be discussed with patients, and decisions about screening with ultrasound made through shared decision making. If screening thyroid ultrasounds are performed, they should be repeated every 3 to 5 years. Early screening for colon cancer is recommended for patients who received abdominal radiation. This patient received mantle radiation, which includes the neck, chest, and axilla, not the abdomen.

Which of the following types of lymphoma has the best outcome when the duration of treatment is at least 2 years and includes a maintenance phase of therapy? a. Stage IV anaplastic large-cell lymphoma b. Burkitt leukemia (>25% blasts in marrow) c. Stage III T-cell lymphoblastic lymphoma (LLy) d. Stage IVB Hodgkin lymphoma e. Stage III diffuse large B cell lymphoma

C The distinction between lymphoblastic lymphoma (LLy) and acute lymphoblastic leukemia (ALL) is that the ALL is defined as more than 25% blasts in the bone marrow. The exact biologic distinction between these two entities is the subject of ongoing research. For stage III and IV LLy, results have been best when patients are treated with regimens like those used for ALL. These include maintenance phases with a total duration of therapy of at least 2 years. The other common pediatric lymphomas, even with extensive marrow involvement, are treated with less than 1 year of cyclic chemotherapy.

A 2.5-month-old boy has a 4-week history of an enlarging mass of the right upper thigh. A doppler ultrasonography of the superficial lesion in this area done 4 weeks earlier was consistent with a 1 cm hemangioma. Today, there is a 5 cm by 5-cm painless, colorless mass in the proximal anterolateral aspect of the thigh without evidence of compartment syndrome or limb dysfunction. Magnetic resonance imaging of the right lower extremity shows tumor infiltrating into the quadricep femoris without vascular obstruction or inguinal adenopathy. Immunohistochemical staining of the tumor biopsy is negative for myogenin and myoblast determination protein 1. Cytogenetic analysis reveals a ETV6-NTRK3 fusion mutation. Pulmonary computed tomography reveals no evidence of metastatic disease. Of the following, the initial BEST treatment option is A.close observation B.gross total resection of the tumor C.neoadjuvant chemotherapy D.radiation therapy

CORRECT View Peer Results Average Correct: 20.23% Based on the age of presentation, imaging results, immunohistochemical staining and cytogenetics of the tumor, this patient has an infantile fibrosarcoma (IFS). Given the patient's age (<3 months old) and extent of tumor infiltration, the best choice is to initially observe with physical examination and imaging. Infantile fibrosarcoma is a rare, malignant, nonrhabdomyosarcoma soft tissue sarcoma (STS) that constitutes only 4.5% of all STSs. However, IFS is the most frequently occuring STS in children younger than 1 year of age, with a slight male predominance and most cases presenting before 2 years of age. Unlike other STSs, IFS rarely metastasizes and spontaneous regression has been reported in neonates. In general, risk factors that influence survival in pediatric non-rhabdomyosarcoma STSs are size of the primary tumor, presence of metastatic disease, extent of surgical resection, and histologic grade. Complete surgical resection of the tumor and metastases have been associated with long-term remissions. If the tumor cannot be resected due to considerable morbidity, neoadjuvant chemotherapy, radiation therapy, or both can potentially shrink STSs pre-operatively. Amputation is sometimes considered for STS of the extremities in which radiation therapy can produce significant long-term complications. Similar to other STS subtypes, IFS typically presents with rapid growth that may impinge on vital neurovascular bundles resulting in compartment syndrome. However, extensive surgery to achieve complete resection can cause significant morbidity in this age group. Since IFS is a chemosensitive tumor, neoadjuvant vincristine and actinomycin-D (VA) can be considered to reduce the size of the tumor, improve surgical resectability, and avoid alkylating chemotherapeutic agents in this age group. Typically, radiation therapy is not considered part of the treatment strategy in this age group due to significant morbidity. Unlike other STSs, spontaneous regression of IFS has been reported in neonates. The European Pediatric Soft Tissue Sarcoma Group conducted a study to conservatively treat IFS, minimizing the use of radical surgery and alkylating chemotherapy. They compared outcomes in 50 patients stratified into 3 groups, children with: 1) complete surgical resection followed by observation; 2) gross total resection with residual microscopic disease, and 3) gross residual disease. Patients with gross residual disease were further subdivided into observation for patients younger than 3 months of age, and neoadjuvant VA before surgical resection for older patients. Outcomes revealed a 3-year event-free survival rate of 84% and an overall survival rate of 94%. There was a good response rate to VA therapy, thus sparing the participants the long-term adverse effects of the alkylating agents typically used for STSs. In addition, the study reported sparing 71% of patients "mutilating" surgery. Of the 4 patients younger than 3 months of age who were initially observed, one experienced spontaneous regression, whereas the other 3 eventually received VA therapy with excellent response. On the basis of these results, a conservative approach could be considered for the child in this vignette. The vignette also highlights that IFS can be initially misdiagnosed radiologically as a benign vascular tumor, such as a hemangioma. Given the rarity of IFS, a clinician must have a high index of suspicion for IFS. Magnetic resonance imaging is considered the radiologic imaging modality of choice to help distinguish between benign and malignant lesions. Follow-up with a multidisciplinary team that specializes in vascular tumors and anomalies is recommended to ensure proper evaluation. Like IFS, many of the STSs have unique genetic mutations. Understanding how these genetic changes affect cellular mechanisms has led to targeted therapies such as tyrosine kinase inhibitors (TKIs). ETV6-NTRK mutations are highly prevalent in IFS. NTRK1, NTRK2, and NTRK3 encode neurotrophin tyrosine kinase receptors. Larotrectinib is a selective TKI that has demonstrated high response rates among children and adults with TRK fusion cancers. Although immunohistochemistry or fluorescence in situ hybridization pick up many of these TRK fusions and other fusion proteins for targeted therapy, these assays are not highly sensitive. Next-generation sequencing of STSs is recommended to identify TRK mutations when cytogenetics and immunohistochemical staining results are negative and to identify other mutations for available targeted therapies. PREP Pearls Infantile fibrosarcoma is the most common malignant soft tissue sarcoma in children younger than 1 year of age, presenting as an enlarging, painless mass that needs to be distinguished from hemangiomas and other vascular lesions and other vascular lesions with magnetic resonance imaging. Given the risk of morbidity of complete tumor resection and alkylating chemotherapy, infants younger than 3 months of age with a non-threatening infantile fibrosarcoma can be initially observed. If tumor progression is observed, vincristine and actinomycin-D are effective in reducing tumor burden pre-operatively. Although ETV6-NTRK fusions can be discovered by immunohistochemistry or fluorescent in situ hybridization in most cases of infantile fibrosarcoma, next-generation sequencing testing of all soft tissue sarcomas is recommended to look for mutations that have potentially associated targeted therapies. ABP Content Specifications(s)/Content Area Recognize the pathologic subtypes of soft tissue sarcomas other than rhabdomyosarcoma relative to prognosis and pattern of spread Recognize the clinical presentation of soft tissue sarcomas other than rhabdomyosarcoma by anatomic site Utilize appropriate imaging modalities and laboratory studies to determine the extent and metastatic spread of soft tissue sarcomas other than rhabdomyosarcoma Know the role of surgery in the treatment of soft tissue sarcomas other than rhabdomyosarcoma Know the role of irradiation in the treatment of soft tissue sarcomas other than rhabdomyosarcoma Know the role of chemotherapy in the treatment of soft tissue sarcomas other than rhabdomyosarcoma

A healthy 15-month-old girl is seen for a health maintenance visit. Her parents report no concerns today, nor any recent illness. She eats meat, vegetables, fruits, and grains and drinks no cow milk. On examination, she has pallor and a 2/6 systolic murmur on cardiac auscultation. At her 12-month visit, her blood lead and hemoglobin levels were normal. Today, her blood test reveals the following: Laboratory Test Result White blood cell count 8.0 × 103/µL (8.0 × 109/L ) Hemoglobin 6.2 g/dL (62 g/L) Mean corpuscular volume 75 µm3 (75 fL) Platelets 300 × 103/µL (300 × 109/L ) Absolute reticulocyte count 20 × 103/µL (20 × 109/L) Of the following, the MOST likely etiology of the girl's anemia is A.folate deficiency B.iron deficiency C.transient erythroblastopenia of childhood D.warm autoimmune hemolysis

CORRECT View Peer Results Average Correct: 76.53% The healthy toddler in this vignette has a normocytic anemia without reticulocytosis, indicating inadequate erythrocyte production. This presentation is most consistent with transient erythroblastopenia of childhood, a transient red blood cell aplasia that occurs in previously healthy young children. Dietary folate deficiency typically presents as a macrocytic anemia. Folate deficiency is relatively uncommon in the United States secondary to folic acid supplementation of processed grains. Other rare causes of folate deficiency include intestinal malabsorption, use of certain medications, and diets restricted to goat milk. Iron deficiency is common in infants and toddlers but is typically associated with microcytic anemia. Autoimmune hemolytic anemia in children typically occurs secondary to warm agglutinins or antibodies that react with red blood cell surface proteins at room temperature, presenting with acute-onset fatigue, shortness of breath, and jaundice. Anemia and brisk reticulocytosis are typical in warm autoimmune hemolytic anemia. Causes of anemia can be divided into two large categories: disorders of inadequate erythrocyte production and disorders of erythrocyte destruction or blood loss. In some instances, children may have more than one cause, but in general one is the main driver of anemia. Erythrocyte production is assessed by the reticulocyte count. Anemia without appropriate reticulocytosis, which is defined as absolute reticulocyte count of > 100 × 103/µL, indicates inadequate erythrocyte production, whereas anemia with reticulocytosis indicates hemolysis or blood loss. A peripheral blood smear can help delineate between hemolysis with evidence of fragmented red blood cells, spherocytes, sickle cells, amongst other poikilocytes versus evidence of the homogenous, normocytic red blood cells seen in blood loss. Causes of anemias can be further sub-categorized according to red blood cell size, as represented by age-appropriate mean corpuscular volume (MCV), into microcytic, normocytic, or macrocytic. Again, these causative categories can overlap, with some involving different sizes of red blood cells. Transient erythrocytopenia of childhood is classified as normocytic, but if it is evaluated during the recovery phase with brisk reticulocytosis, macrocytosis will be seen. Similar to age-appropriate hemoglobin values, MCV values change with age and ethnicity. Data from the National Health and Nutrition Examination Survey showed that black children had MCV values lower than those of white and Mexican American children. The Table presents common causes of anemia according to MCV and reticulocyte count. A complete blood count and reticulocyte count can help narrow the diagnostic possibilities of anemia. These results then help direct additional confirmatory testing. For example, in cases of microcytosis with reticulocytopenia, serum iron studies consisting of serum iron, ferritin, and total iron-binding capacity would be appropriate. In cases of macrocytic anemia with reticulocytosis, hemolytic markers such as bilirubin, lactate dehydrogenase, and haptoglobin help to further delineate etiology of this anemia. PREP Pearls Anemia without appropriate reticulocytosis which is defined as absolute reticulocyte count of > 100 × 103/µL, indicates inadequate erythrocyte production, whereas anemia with reticulocytosis indicates hemolysis or blood loss. Peripheral blood smear can help delineate between hemolysis with evidence of fragmented red blood cells, spherocytes, sickle cells, amongst other poikilocytes versus the homogenous, normocytic red blood cells seen in blood loss.

A 4-year-old boy, recently diagnosed with severe aplastic anemia and undergoing evaluation for hematopoietic stem cell transplantation, is seen in the clinic for epistaxis. Laboratory data are shown: Laboratory Test Result White blood cell count 2,600/µL (2.6 × 109/L) Hemoglobin 9.3 g/dL (93 g/L) Platelet count 8.0 × 103/µL (8.0 × 109/L) Of the following, the MOST appropriate blood product modifications for this patient's platelet transfusion are A.cytomegalovirus seronegative and leukoreduced B.HLA-matched and cytomegalovirus seronegative C.irradiated and leukoreduced D.irradiated and washed

CORRECT View Peer Results Average Correct: 85.50% The patient in the vignette, who is bring evaluated to receive a hematopoietic stem cell transplantation (HSCT), requires blood products that are irradiated and negative for cytomegalovirus (CMV), via either leukoreduction or testing for donor CMV seronegativity. Irradiation of blood products is indicated in immunosuppressed patients, including patients undergoing or even planning to undergo HSCT. Irradiation prevents transfusion-associated graft-vs-host disease, which is mediated by donor lymphocytes. With current techniques for leukoreduction, filters can remove 99.99% of leukocytes, and many apheresis machines have leukoreduction mechanisms. Leukoreduction significantly reduces the risk of transmitting viruses that reside within leukocytes, such as CMV and the Epstein-Barr virus. The efficacy of leukoreduction in reducing CMV transmission is considered equivalent to using CMV-seronegative donors. Other benefits of removal of white cells by leukoreduction include reduction of febrile nonhemolytic transfusion reactions and prevention of alloimmunization to donor human leukocyte antigens (HLAs) for HSCT recipients or patients who are transfusion dependent.. Human leukocyte antigen matching of platelets is indicated for patients who have demonstrated alloimmunization or platelet refractoriness, which is not present in this patient. Washing platelets removes plasma but also significantly decreases the quantity and quality of platelets being transfused and is therefore reserved for patients with IgA deficiency, neonatal alloimmune thrombocytopenia, a history of severe allergic reactions to transfusions, or a combination of these. Cytomegalovirus-seronegative HSCT recipients are at risk of developing transfusion-transmitted infection from CMV (TT-CMV), which can infect the lungs, liver, gastrointestinal tract, retina, and central nervous system. Antiviral treatments such as ganciclovir, foscarnet, and cidofovir as well as adoptive T-cell therapy have been used to treat TT-CMV. Whereas CMV serology can be used to determine recipient serologic status prior to HSCT, the quantitative polymerase chain reaction test is used to detect and monitor viral DNA burden in immunocompromised patients. As a result of the effectiveness of widespread leukoreduction, the risk of TT-CMV is exceedingly rare in recipients of leukoreduced blood products. In the United States, the estimated risk of infection after transfusion of blood products (red blood cells, platelets, plasma) is 1 in 1 million for hepatitis B virus, 1 in 1.2 million for hepatitis C virus, and 1 in 1.5 million for HIV. PREP Pearls Leukoreduction significantly reduces the risk of transmitting viruses that reside within leukocytes, such as cytomegalovirus and Epstein-Barr virus, reduces the incidence of febrile nonhemolytic transfusion reactions, and prevents alloimmunization to donor human leukocyte antigens. Leukoreduction is considered equivalent in efficacy of reducing cytomegalovirus transmission to the use of cytomegalovirus-seronegative donors. In the United States, the estimated risk of infection following transfusion of blood products (red blood cells, platelets, plasma) is less than 1 in 1 million for hepatitis B virus, 1 in 1.2 million for hepatitis C virus, and 1 in 1.5 million for HIV. ABP Content Specifications(s)/Content Area Know the risk of transmission of HIV‑1, hepatitis B, and hepatitis C, in transfusion of single-donor blood components Recognize the clinical and laboratory manifestations of transfusion-acquired CMV infection Know that frozen or filtered (leukocyte reduced) erythrocytes will reduce transmission of cytomegalovirus to recipients

A 2-day-old neonate prenatally diagnosed with Down syndrome is afebrile and normotensive with mild tachycardia and hepatosplenomegaly. He has a white blood cell count of 25,000/µL (25.0 × 109/L) with 10% blasts. The blasts are medium-sized with basophilic cytoplasm and cytoplasmic blebs. Of the following, the MOST helpful test in establishing a diagnosis is A.bone marrow aspirate and biopsy B.flow cytometry of peripheral blood C.karyotype analysis of peripheral blood D.liver biopsy

Children with Down syndrome are at an increased risk of developing leukemia compared with the general population, with a 150-fold increase in the incidence of acute myeloid leukemia (AML). Children with Down syndrome (Trisomy 21) may also develop transient abnormal myelopoiesis (TAM), also known as transient myeloproliferative disorder, during the newborn period and early infancy. Transient abnormal myelopoiesis is a preleukemic disorder that can be difficult to distinguish from AML at initial presentation. In TAM, megakaryocytic blasts are found in the peripheral blood of infants with Down syndrome or mosaic Down syndrome. The incidence of TAM (including silent TAM, discussed below) in infants with Down syndrome is 10% to 30%. In Down syndrome, expansion of megakaryocytic-erythroid progenitors occurs during hematopoiesis in the fetal liver. In TAM, a somatic mutation in GATA1 leads to the expression of a truncated GATA1 protein. GATA1 mutations cause impairment of megakaryocytic differentiation and uncontrolled proliferation of megakaryocytic blasts. Transient abnormal myelopoiesis is diagnosed within the first few weeks after birth and has a variable clinical presentation. Approximately 10% to 40% of affected infants are asymptomatic yet have peripheral blasts on a complete blood cell count. Approximately 25% of affected infants present with life-threatening illness, such as hydrops fetalis, hyperleukocytosis, liver failure, and pericardial or pleural effusions. Leukocytosis with a median white blood cell count of 28,000 × 106/µL to 40,000 × 106/µL (28.0-40.0 × 109/L) is common while about 25% of affected infants demonstrate hyperleukocytosis (white blood cell count > 100,000/µL [100 × 109/L]). Mild anemia and mild thrombocytopenia may also be present. Megakaryoblasts on the peripheral blood smear are characterized as medium to large, with basophilic cytoplasm, amorphous nuclei, and often cytoplasmic blebs. Flow cytometry of the peripheral blasts reveals megakaryoblastic antigens (CD61, CD41, CD42), myeloid antigens (CD13, CD33), stem cell antigens (CD34, CD117), or a combination of these. In TAM, peripheral blasts are more numerous than bone marrow blasts, reflecting the fact that fetal hematopoiesis occurs in other tissues such as the fetal liver. Bone marrow evaluation, therefore, is not routinely needed in these patients. If performed, bone marrow evaluation usually shows abnormal megakaryocytic maturation and may show dyserythropoiesis in some patients. Blasts may infiltrate solid organs, most commonly the liver, spleen, skin, and bone marrow. Hepatosplenomegaly occurs in more than one-half of patients with TAM. Liver failure and liver fibrosis occur in up to 15% of patients, and liver fibrosis is associated with a very poor prognosis. The conjugated bilirubin level is often elevated. Skin involvement by TAM cells manifests as papules, pustules, and vesicles. Coagulopathy occurs in 20% of patients. Disseminated intravascular coagulation, a poor prognostic factor, is seen in less than 10% of patients. Although there are no universally accepted diagnostic criteria for TAM, it is usually diagnosed via the identification of circulating megakaryocytic blasts and megakaryoblastic immunophenotype by means of flow cytometry, without thresholds for blast percentage or infant age. It is recommended that newborns with Down syndrome be screened with a complete blood cell count and peripheral smear at birth, even if asymptomatic. If blasts or abnormal blood counts are present, flow cytometry of peripheral blood is performed. Sequencing for GATA1 mutations is helpful. Identification of a somatic GATA1 mutation, usually in exon 2 or 3, is the most definitive method of diagnosis. In one study of newborns with Down syndrome, the GATA1 mutation was detected via sequencing in 20% of the study population without clinical features of TAM, a condition termed "silent TAM." The differential diagnosis of peripheral blasts in a newborn with Down syndrome includes infections, sepsis, severe congenital infections, and severe hemolysis. Although children with Down syndrome have an increased incidence of acute lymphoblastic leukemia (ALL) compared to the general population, infant ALL (< 1 year of age) is rare in patients with Down syndrome. Suspicion of TAM in patients without known Down syndrome should prompt evaluation for Down syndrome or Down syndrome mosaicism. Treatment of most patients with TAM primarily involves supportive care, including management for hyperleukocytosis, tumor lysis syndrome, and coagulopathy, when needed. Transient abnormal myelopoiesis typically resolves spontaneously within the first 3 months after birth. In one Children's Oncology Group study of TAM, the mean time to resolution of peripheral blasts was 36 days (range, 2-126 days), and the median time to resolution of clinical signs of TAM (such as organomegaly) was 49 days (range, 5-745 days). However, TAM is associated with mortality in up to 20% of cases. Risk factors for early mortality are hydrops fetalis, liver failure or fibrosis, and cardiorespiratory failure caused by pleural or pericardial effusions. Hyperleukocytosis and severe coagulopathy are unfavorable prognostic factors in some studies. The above factors are therefore used for risk stratification in the management of TAM. Patients at high risk can be treated with low-dose cytarabine or exchange transfusion/leukapheresis. In approximately 20% of patients with TAM, the condition will initially resolve as described above, but the patients will later develop AML, usually by 4 years of age. These patients sometimes have a preceding myelodysplastic phase. In patients who develop AML, the same clone-specific GATA1 mutation is found within both the TAM blasts and the AML blasts. However, the AML blasts often contain additional mutations in other pathways that may be responsible for the transformation to AML. One theory is that a subclinical population of TAM blasts persists after apparent clinical resolution of TAM that later transforms into AML as it acquires other mutations. In the setting of Down syndrome, GATA1 mutation appears to be present in all cases of AML. Patients with Down syndrome and AML have an excellent prognosis, with a 5-year event-free survival of 90%. Treatment of TAM with low-dose cytarabine does not reduce the risk of developing AML. All patients with TAM are clinically followed after the resolution of TAM because of the risk of later development of AML. There are no validated guidelines for how to monitor TAM patients, but a suggested schedule is to follow every child with TAM every 3 months until the age of 4 years. These visits include a history, physical examination, a complete blood count with differential, and a peripheral blood smear. The patient in this vignette has Down syndrome and peripheral blasts with megakaryocytic features, but he has no obvious signs of sepsis or severe congenital infection. Thus, TAM is highly suspected. Flow cytometry is the best test to establish a megakaryocytic immunophenotype, which would strongly support a diagnosis of TAM. Bone marrow evaluation is an invasive procedure that is unlikely to offer additional information useful in diagnosis and management because the peripheral blast count is usually higher than the bone marrow blast count in TAM. Conventional karyotype analysis on the peripheral blood will not offer additional information regarding TAM. Although identification of a GATA1 mutation would be helpful in diagnosis, gene sequencing rather than karyotype analysis is needed. Finally, while a liver biopsy may show infiltration of the liver by megakaryocytic blasts in TAM, this invasive procedure is not needed when circulating blasts are present. When the clinical picture is otherwise consistent with TAM, evaluating for other liver disease is not necessary because hepatomegaly is a common presenting feature of TAM. PREP Pearls Transient abnormal myelopoiesis occurs in 10% to 30% of newborns with Down syndrome (Trisomy 21) and has variable clinical presentations. Although most patients have spontaneous resolution of transient abnormal myelopoiesis within the first 3 months after birth and do not require therapy, a subset with high-risk features—such as hydrops fetalis, liver failure or fibrosis, hyperleukocytosis, severe coagulopathy, or cardiorespiratory failure caused by pleural or pericardial effusions—may benefit from low-dose cytarabine. Transient abnormal myelopoiesis is associated with a somatic GATA1 mutation, and 20% of patients will develop acute myeloid leukemia in early childhood with the same clone-specific mutation.

A previously healthy 12-year-old girl has a 3-month history of an intermittent facial "butterfly" rash and transiently painful swollen joints with a 2-day history of worsening right calf and thigh pain, swelling, and discoloration. She reports no trauma or pregnancy. She takes nonsteroidal anti-inflammatory agents and no other medication. Her family history is negative for thrombosis. Complete blood count and metabolic panel results are normal. Test results show that C-reactive protein and antinuclear antibody with reflex panel levels are elevated. Urinalysis reveals proteinuria. Doppler ultrasonography shows a complete occlusive right femoral and iliac vein thrombosis. Of the following, the screening test result that is MOST likely to lead to the underlying cause of the deep vein thrombosis is A.elevated D-dimer B.elevated fibrinogen C.prolonged activated partial thromboplastin time D.shortened prothrombin time

Correct Answer: C The patient in the vignette has deep vein thrombosis and is suspected to have systemic lupus erythematosus (SLE). There is an increased risk of thrombosis in patients with SLE who demonstrate a lupus anticoagulant (LA) antibody. Lupus anticoagulant encompasses a variety of immunoglobulins that attach to cell membrane proteins (eg, prothrombin, β2 glycoprotein) and phospholipids in the coagulation cascade, acting as a procoagulant in vivo and as an inhibitor of coagulation assays in vitro. A prolonged activated partial thromboplastin time (aPTT), obtained and processed properly, either reflects a coagulation factor deficiency or the possibility of a clotting factor inhibitor such as LA. Specifically, an aPTT with mixing study is a helpful initial screening test for an inhibitor or LA. An aPTT mixing study is suggestive of a specific clotting factor inhibitor or LA when the addition of normal platelet-free plasma to the patient's plasma fails to correct aPTT prolongation. If a specific clotting factor inhibitor is elucidated from additional antibody studies, then LA is present. In addition to physical findings and presence of LA on 2 separate studies 12 weeks apart, other studies supportive of antiphospholipid syndrome as the cause of thrombosis include an abnormal Russell viper venom time, abnormal phospholipid neutralization study, positive anticardiolipin antibodies test, and positive anti-β2 glycoprotein antibodies test. In otherwise healthy children and adolescents, the most commonly detected LAs are transient, not associated with SLE or thrombophilia, and often found incidentally during preoperative screening, manifesting as a prolonged aPTT. These children are often asymptomatic, although some may have a preceding viral or bacterial infection. Most LAs resolve spontaneously within 6 to 12 weeks as noted on repeat screening with aPTT. An isolated shortened prothrombin time is not associated with thrombosis. The D-dimer level is elevated as a result of disseminated intravascular coagulation or active thrombosis, which does not explain the cause of deep vein thrombosis in the girl in this vignette. Fibrinogen levels are elevated in various acute illnesses such as infection or inflammatory states such as SLE. In these conditions, an elevated fibrinogen level likely represents an acute-phase reactant but does not explain the cause of thrombosis. PREP Pearls Lupus anticoagulant antibodies detected incidentally in an otherwise healthy child are most likely to be transient and not associated with lupus or thrombophilia. In suspected acquired thrombophilic state, an activated partial thromboplastin time is useful as an initial screening test for lupus anticoagulant.

A 19-year-old man is referred to your clinic after 1 week of fevers and increased fatigue. His CBC reveals a WBC count of 75,000/μL, hemoglobin of 5.5 g/dL, and platelets of 15,000/μL. On peripheral blood smear, 86% of the WBCs are large, immature cells with scant cytoplasm and prominent nucleoli. Which cytogenetic finding is more likely to occur in this teenage patient compared with an infant or young child with a similar presentation? a. t(12;21) b. High hyperploidy (51-65 chromosomes) c. t(4;11) d. t(9;22) e. t(1;19)

D Adolescents and young adults (AYAs) with acute lymphoblastic leukemia have different cytogenetic abnormalities compared with infants and young children, which may be a factor in their decreased survival. Philadelphia chromosome-positive leukemia with t(9;22) is rare in young children (approximately 5% of diagnoses); however, the rate increases with age, with an incidence of 10% to 25% in AYAs. Patients with this translocation are considered to be at very high risk. Current treatment includes tyrosine kinase inhibitors. More favorable cytogenetic factors, such as t(12;21) and high hyperdiploidy, are more common in young children than AYAs. MLL rearrangements, particularly t(4;11), are the most common cytogenetic abnormality in infants and are associated with poor prognosis. The t(1;19) translocation was originally associated with an unfavorable prognosis; however, treatment with more intensive approaches has improved results, and it is no longer thought to be a prognostic factor. This translocation occurs in most age groups at approximately the same rate.

An 18-year old male patient with acute lymphoblastic leukemia recently started maintenance therapy and is complaining of increased hip pain. The pain increases during weight-bearing activity; however, it occasionally hurts at night as well. His CBCd is normal. Which of the following risk factors is most commonly associated with this process? a. Younger age at diagnosis b. Non-White race c. Low body-mass index d. Dexamethasone exposure e. Male sex

D Avascular necrosis (AVN) is a well-known complication of therapy for acute lymphoblastic leukemia and can lead to significantly impaired quality of life. AVN can develop during treatment or after therapy completion and is associated with exposure to glucocorticoids. Dexamethasone has more bone toxicity compared with equivalent doses of prednisone, and continuous exposure increases this risk. Weight-bearing joints are affected in 95% of patients with AVN, with the femoral head as the most commonly involved joint, though often it is multifocal. The mechanism of injury is thought to be multifactorial, including disruption of osteoblasts, intramedullary lipocyte proliferation impacting circulation, and fat embolization to subchondral arteries. Common risk factors include female sex, radiation exposure, White race, and obesity. Teenagers are more likely to develop AVN compared with younger patients; therefore, current treatment protocols limit the exposure to long courses of dexamethasone in adolescents.

Which of the following uses of radiation is considered standard practice in pediatric lymphoma? a. Involved node radiation for anaplastic large-cell lymphoma with residual mass after two cycles of chemotherapy b. Central nervous system (CNS) radiation for Burkitt lymphoma with CNS disease at diagnosis c. Prophylactic CNS radiation for all patients with stage III lymphoblastic lymphoma d. Involved node radiation to PET-avid residual tumor mass (with uptake markedly increased compared with the liver) after chemotherapy for a patient with stage IIIB nodular sclerosing Hodgkin lymphoma

D Explanation The use of low-dose involved field radiation is a routine component of treatment for pediatric patients with high-risk Hodgkin lymphoma who do not obtain a complete metabolic remission following chemotherapy. However, because radiation to the mediastinum of females is associated with a significant increased risk of breast cancer, clinical trials have been designed to test the hypothesis that radiation therapy can be reduced or eliminated for subsets of patients with Hodgkin lymphoma who have complete and rapid response to chemotherapy. Radiation therapy has not been shown to improve outcomes for most pediatric non-Hodgkin lymphomas (NHL). In pediatric NHL, radiation therapy is reserved for patients with life-threatening emergencies at diagnosis, such as airway compression due to a mediastinal mass. Some patients with lymphoblastic lymphoma with central nervous system (CNS) disease at diagnosis (stage IV) also receive radiation; however, ongoing research continues to reduce the number of patients who require this

A 14-year-old girl presents to survivor clinic for her first visit after transferring care from another institution. She was treated for acute lymphoblastic leukemia (ALL) at 2 years of age. She has been doing well since that time, but her family reports that she is struggling in school, especially with concentration and math. Which of the following treatments that she received places her at greatest risk for neurocognitive deficits? a. Dexamethasone b. Asparaginase c. Daunorubicin d. High-dose methotrexate e. Vincristine

D Neurocognitive deficits can occur in survivors treated with high-dose cytarabine, high-dose methotrexate, intrathecal methotrexate, and cranial radiation. These deficits are typically functional deficits in executive function, attention, memory, processing speed, visual-motor integration, and fine motor dexterity. Survivors also may have learning deficits, particularly in math and reading comprehension. New deficits may emerge over time. A meta-analysis of childhood acute lymphoblastic leukemia survivors treated with chemotherapy-only treatment regimens showed a significant impairment in IQ and other neurocognitive domains. Survivors with concerns may benefit from a formal neuropsychological evaluation.

A previously healthy 7-year-old male presents to his primary care doctor with fatigue, dyspnea on exertion, orthopnea, headaches, and cough, which have been progressive over the past 2 weeks. On examination, he is anxious, tachypneic, has facial edema and swelling, and distended neck veins. Chest x-ray reveals a large anterior mediastinal mass with no pleural or pericardial effusion. You are most concerned that his symptoms are due to what complication of newly presenting lymphoma? a. Tumor lysis syndrome b. Pulmonary leukostasis c. Superior vena cava syndrome d. Superior mediastinal syndrome

D Superior vena cava (SVC) syndrome is compression and obstruction of the SVC resulting in impaired venous return. This occurs with anterior mediastinal masses because the SVC has a thin wall, low intraluminal pressure, and is anatomically surrounded by the thymus in other lymph nodes. Superior mediastinal syndrome is SVC syndrome with airway compromise, which is the case with this patient. The trachea and mainstem bronchi are more compliant and compressible in children compared with adults. Some signs of respiratory compromise are evident in up to 75% of patients presenting with a new anterior mediastinal mass. Signs of airway compromise include cough, hoarseness, tachypnea, dyspnea, orthopnea, stridor, wheezing, and anxiety. Signs of venous obstruction include swelling; plethora; cyanosis of the face, neck, and upper extremities; engorgement of chest wall vessels; petechiae of the head, neck, arms, and trunk; edema; pleural effusions; and pulsus paradoxus. Patients may also have central nervous system signs including headache, confusion, lethargy, blurry vision, papilledema, syncope, and a sensation of fullness in their ears.

A 23-year-old woman with a history of rhabdomyosarcoma at age 4 years comes to the survivor clinic to discuss late effects. She is worried about her risk of infertility. Which of the following factors is not known to affect the risk of infertility for female survivors of childhood cancer? a. Age at treatment b. Dosage of alkylating agent chemotherapy c. Location of radiation d. Current age e. Race

E Many factors can affect the risk of infertility. Surgery, radiation, and chemotherapy that affect the hypothalamic- pituitary-gonadal axis and reproductive organs increase the risk of infertility. In general, female patients maintain ovarian function at higher cumulative alkylating agent dosages than testicular function in male patients. However, the risk of infertility increases with increased dosage of alkylating agents and radiation. Female patients are at high risk for premature ovarian insufficiency when they receive ovarian radiation dosages greater than 20 Gy. Compared with postpubertal girls, prepubertal girls are able to tolerate higher dosages of gonadotoxic chemotherapy and radiation before development of premature ovarian insufficiency. All girls are born with a finite number of primordial ovarian follicles that decrease over time until the number approaches 1,000 follicles, and menopause ensues. In female survivors of childhood cancer who have received gonadotoxic therapy, there may be an abrupt drop in the primordial follicle pool, leading to premature ovarian insufficiency or ovarian failure. However, women with a decreased ovarian reserve due to cancer therapy may have a window of time between the end of cancer treatment and the onset of ovarian insufficiency in which they are fertile and could conceive or undergo fertility preservation measures. There are no data supporting a difference in risk for infertility due to cancer treatment based on race.

A healthy 10-year-old boy is seen by his pediatrician for his annual routine visit with a normal history and physical examination. A complete blood count is also normal except for a seemingly left shift in the neutrophil count. The peripheral blood smear reveals many neutrophils that appear as "funny looking" band forms. The child's father has a similar genetic condition. Of the following, the blood smear that is MOST representative of the patient is: A. A. Response Choice A Choice A B. B. Response Choice B Choice B C. C. Response Choice C Choice C D. D. Response Choice D Choice D

For healthy individuals, neutrophils are the most common granulocytes in the peripheral blood and are recognized by their characteristic nuclear morphology and presence of cytoplasmic granules. They belong to the myeloid lineage and go through various stages of development in the bone marrow, which include myeloblasts (characterised by large oval nuclei, prominent nucleoli, few granules), promyelocytes (containing azurophilic granules) and myelocytes (containing peroxidase negative granules). Subsequently during metamyelocyte, band neutrophil (curved and not lobulated nucleus), and mature neutrophil stages, the cell goes through progressive nuclear condensation to develop the familiar poly-segmented nucleus. About 70% of neutrophils have 3 to 4 segments, 20% have 2 segments, 5% have more than 4 segments, and the rest have no segmentation at all. The characteristic morphology of neutrophils can change in various medical conditions which helps to differentiate amongst related disorders. This clinical vignette and Response Choice B are consistent with a benign genetic condition associated with hyposegmented neutrophils known as Pelger-Huët anomaly (PHA). Pelger-Huët anomaly is characterized by morphologically abnormal neutrophils with bi-lobed or uni-lobed nuclei that look like band cells or are 'left-shifted' on a peripheral blood smear without any obvious neutrophilia or underlying infectious, autoimmune, or malignant etiology. Nuclear chromatin is often coarsely clumped. The PHA is caused by mutations in the lamin B receptor (LBR) gene. Heterozygous carriers have bi-lobed neutrophils while homozygous individuals have uni-lobed neutrophils. These normal functioning neutrophils are not associated with immunodeficiency, although rarely mild skeletal anomalies have been reported in individuals with PHA. Acquired or pseudo-PHA forms are neutrophils with similar bi-lobed nuclei, but without the LBR mutation that can be seen in myeloproliferative disorders, autoimmune disease, or as a result of exposure to certain medications or infections. Typically pseudo-PHA is associated with no family history of PHA, fewer affected neutrophils (<10% compared to >50% in isolated PHA), and cytopenias. If pseudo-PHA is considered, additional studies may be warranted such as bone marrow studies to assess for myelofibrosis, myelodysplastic syndrome, and myeloid leukemias. The bone marrow smear in Response Choice A shows a macrophage with an ingested neutrophil precursor and red blood cell fragments consistent with hemophagocytic lymphohistiocytosis. The peripheral blood smear in Response Choice C reveals leukocytosis with predominantly neutrophils and myeloid precursors consistent with severe infection and septic shock. The peripheral blood smear in Response Choice D is an example of a hypersegmented neutrophil with more than 4 nuclear segments as often seen in vitamin B12 or folate deficiency. Please note, all images are courtesy of A Sharma. PREP Pearls Isolated Pelger-Huët anomaly is a benign condition caused by inherited mutations in the lamin B receptor gene and characterized by morphologically abnormal neutrophils with bi-lobed or uni-lobed nuclei that are often mistaken for band neutrophils (which have curved but not lobulated nuclei on a blood smear). Acquired or pseudo-Pelger-Huët anomaly is not inherited and is typically associated with other clinical signs or symptoms as in myeloproliferative disorders, autoimmune disease, infections, or medication exposures. ABP Content Specifications(s)/Content Area Recognize the morphologic alteration of neutrophils associated with the Pelger-Huet anomaly Recognize the conditions associated with Pelger-Huet anomaly

A male infant diagnosed with Hurler syndrome has been receiving enzyme replacement therapy with alpha-L-iduronidase (IDUA) and is undergoing evaluation for an allogeneic hematopoietic stem cell transplant (HSCT). He has one full biological sibling, who is also a confirmed carrier for Hurler syndrome with one defective copy of the IDUA gene. The planned conditioning regimen is a full myeloablative chemotherapy regimen. Of the following, the MOST preferred HSCT donor would be A.the biological mother, matched for 7 of 8 human leukocyte antigens B.the male sibling, matched for 8 of 8 human leukocyte antigens C.an unrelated female donor, matched for 8 of 8 human leukocyte antigens D.an unrelated male donor, matched for 8 of 8 human leukocyte antigens

Hematopoietic stem cell transplantation (HSCT) is the only known treatment option for Hurler syndrome that has been shown to prevent progressive neurologic deterioration. The best donor in the above vignette is the unrelated male donor matched for 8 of 8 human leukocyte antigens. Mucopolysaccharidosis type 1 (MPS 1) is a rare, inherited autosomal-recessive inborn error of metabolism. It is also a lysosomal storage disorder. This genetic defect causes a deficiency in alpha-L-iduronidase (IDUA). There are 3 clinical subtypes, with Hurler syndrome (HS) having the most severe phenotype because of severely deficient or absent enzyme activity. This lack of enzyme activity leads to progressive accumulation of glycosaminoglycans, dermatan sulfates, and heparan sulfates within cells and in the plasma. In the newborn period, patients are initially clinically asymptomatic. After 6 months of age, patients will begin to develop progressive multisystem organ dysfunction, which includes progressive psychomotor retardation, hearing loss, corneal clouding, organomegaly, cardiac failure, and pulmonary failure. Excluding family history, diagnosis is difficult, with the average age of diagnosis being 10 months. Enzyme replacement therapy with IDUA is available, which will improve the cardiac and pulmonary effects and visceral organomegaly. However, these patients continue to develop progressive neurologic deterioration owing to the inability of the enzyme replacement to cross through the blood-brain barrier. Allogeneic HSCT provides donor-derived mononuclear cells that are able to cross the blood-brain barrier and are able to metabolize the accumulated IDUA substrates. Success of an allogeneic HSCT in Hurler syndrome is based on early diagnosis, time to transplantation, and ultimately the degree to which the donor engraftment provides the maximal amount of donor-derived enzyme activity. Neurologic outcomes were best when patients received transplants early in life and from donors who had normal enzyme levels. Heterozygote carriers of the defective IDUA gene have enzyme levels that approach 50% of normal. Observational studies of patients with HS who received HSCT from noncarrier donors had the best neurologic improvement, with full chimerism and normal enzyme levels. On the basis of the findings of these early observational studies, conditioning regimens are typically full myeloablative regimens. Unless there is a related donor who is not shown to carry the trait and who has full normal enzymatic activity levels, an unrelated donor is preferred. Therefore, the mother and sibling are excluded as donors. Donor selection for an allogeneic HSCT is a complex process. Excluding the disease for which transplantation is considered, graft-vs-host disease (GVHD) is still the leading cause of morbidity and mortality in HSCT. Human leukocyte antigen mismatch, particularly with unrelated-donor HSCTs, is still the leading risk factor for developing GVHD. If multiple donors are identified, there are secondary factors that can be evaluated to minimize risk of GVHD or transplant-related morbidity. Secondary donor selection factors include age, sex, cytomegalovirus infection status, size of the donor, graft source (bone marrow vs peripheral blood stem cells), and ABO Rh blood typing. Studies have shown that male recipients have a higher incidence of GVHD and non-relapse mortality with transplants from female donors. This is thought to be due to the male minor histocompatibility antigens that are recognized by female donor T cells. Other studies have shown that parity of the female donor increases the risk of GVHD. On the basis of these findings, the unrelated male donor is preferred. PREP Pearls Hurler syndrome is an inborn error of metabolism that requires early diagnosis and hematopoietic stem cell transplantation to achieve the best long-term outcomes. SInce the most effective treatment for Hurler syndrome is hematopoietic stem cell transplant from donors with normal alpha-L-iduronidase enzyme levels, related donors need to be tested for the defective IDUA gene and heterozygous donors are to be avoided. If multiple hematopoietic stem cell transplant donors are available, male donors are preferred for male recipients due to increased risk of graft-vs-host disease associated with female donor T cells attacking recipient male minor histocompatibility antigens.

An 18-month-old girl who recently emigrated from Thailand is referred for anemia. The child has a normal diet and is growing at the 50th percentile for weight and height. Physical examination findings are unremarkable. Results from a complete blood count are shown: Laboratory Value Results White blood cell count 8,500/µL (8.5 × 109/L) Red blood cell count 6.0 × 106/µL (6.0 × 1012/L) Hemoglobin 8.0 g/dL (80 g/L) Mean corpuscular volume 60 µm3 (60 fL) Platelet count 320 × 103/µL (320× 109/L) Results from a hemoglobin typing by high-performance liquid chromatography are shown: Laboratory Value Results Hemoglobin E 60% Hemoglobin A2 8% Hemoglobin F 32% Of the following, the MOST likely diagnosis for this patient is A.hemoglobin E with a-thalassemia trait B.hemoglobin E with ß-thalassemia C.heterozygous hemoglobin E D.homozygous hemoglobin E

Hemoglobin (Hb) E is found in high prevalence in Asia. The patient in the vignette, who has moderate to severe microcytic anemia, has a presentation consistent with coinheritance of HbE with 0-thalassemia. Hemoglobin E with α-thalassemia trait would present with microcytosis but no anemia and chromatography would show the presence of HbE with normal levels of HbA2 and HbF. In patients with heterozygous HbE, the HbE concentration is about 30% without anemia, microcytosis, or elevation of A2. In patients with homozygous HbE, the concentration of HbE can be greater than 90%, but there is only mild anemia and microcytosis. Thalassemias are disorders caused by decreased production of 1 of the globin chains, leading to a relative excess of unpaired chains that precipitate and lead to impaired erythropoiesis and hemolysis. It is estimated that 1% to 5% of the world's population carries a Hb mutation, with the highest frequency of these mutations occurring in malaria-endemic regions. Heterozygotes for these Hb mutations (eg, α-thalassemia, β-thalassemia, HbE) have been shown to have greater resistance to Plasmodium falciparum. α-Thalassemia and β-thalassemia are the most common thalassemia syndromes. In general, disease severity correlates to the quantity of unpaired globin chains. α-Globin is encoded by 4 genes, 2 on each copy of chromosome 16. Most mutations seen in ɑ thalassemia syndromes are caused by large gene deletions. A single gene deletion (αα/α-) causes a silent carrier state with no clinical manifestations. When 2 genes have been deleted, this can occur in trans (1 deletion on each chromosome 16) or cis (both deletions on the same chromosome 16). The trans deletion is more common in African populations, whereas the cis deletion occurs more commonly in Asians. The cis deletion when combined with a silent carrier or trans mutation can lead to a 3-gene deletion and HbH disease; its combination with another cis deletion can lead to hydrops fetalis. Thus, although the α-thalassemia trait can be found in high prevalence in both Africa and Asia, the occurrence of severe forms of α-thalassemia such as HbH disease or hydrops fetalis is more common in Asia. The clinical manifestations of HbH disease can vary widely in severity; some patients have only a mild anemia, whereas others are symptomatic at birth and become dependent on transfusions. Most patients with HbH disease are not transfusion dependent but may need occasional transfusions during times of increased hemolysis (eg, with infection) or aplastic crisis. The majority of patients with HbH disease will develop some sequelae of ineffective erythropoiesis and extramedullary hematopoiesis by adulthood, such as iron overload and hepatosplenomegaly, or, less commonly, gallstones, bone deformities, growth impairment, or leg ulcers. β-Globin is encoded by 2 genes, one on each copy of chromosome 11. More than 250 mutations have been identified as causing β-thalassemia, most of which are point mutations, including substitutions, deletions, and insertions of single nucleotides or oligonucleotides, which leads to variability in the level of β-globin expression. Thus, β-globin mutations are categorized into β0- or β+-thalassemia with considerable heterogeneity in disease phenotype according to the amount of β globin produced. Traditionally, β-thalassemia was classified into minor (ie, heterozygous mutation, trait, carrier status), intermedia (ie, moderate anemia, intermittent transfusion requirement), and major (ie, homozygous mutation, severe anemia, transfusion dependence); however, it is now also classified into non-transfusion-dependent thalassemia and transfusion-dependent thalassemia. The Table summarizes the molecular abnormalities, high-prevalence ethnic populations, disease phenotype, and clinical features of several common and noteworthy hemoglobin disorders. PREP Pearls Thalassemias are disorders caused by decreased production of one of the hemoglobin chains, leading to a relative excess of unpaired chains that precipitate and lead to hemolysis and impaired erythropoiesis. The clinical manifestations of HbH disease (α-thalassemia) can vary in severity from mild anemia or needing occasional transfusions during times of increased hemolysis (eg, with infection) or aplastic crisis to transfusion dependency. Non-transfusion-dependent β-thalassemia syndromes include β-thalassemia minor (ie, heterozygous mutation, trait, carrier status) and β-thalassemia intermedia (ie, moderate anemia, intermittent transfusion requirement) whereas transfusion-dependent thalassemia, also known as β-thalassemia major, is associated with homozygous mutation, severe anemia, and transfusion dependence. In patients with heterozygous HbE, the HbE concentration is about 30% without anemia or microcytosis, whereas in homozygous HbE, the concentration of HbE can be greater than 90% with mild anemia and microcytosis. ABP Content Specifications(s)/Content Area Identify the molecular abnormalities associated with the various types of thalassemia syndromes, including alpha, beta, and delta-beta thalassemia, hgb E, and hgb Lepore Know the differences in the inheritance of abnormal alpha genes between blacks and Asians with alpha-thalassemia

A 9-year-old girl has had scleral icterus and mild fatigue for 3 months. She experienced similar symptoms 2 years ago. She has a family history of hereditary spherocytosis in her mother, aunt, maternal grandfather, and cousin. She has no palpable hepatosplenomegaly. Laboratory data from an outside hospital are shown: Laboratory Test Result White blood cell count 9,600/µL (9.6 × 109/L) Red blood cell count 3.2 × 106/µL (3.2 × 1012/L) Hemoglobin 9.1 g/dL (91 g/L) Hematocrit 28% Mean corpuscular volume 80 fL Mean corpuscular hemoglobin concentration 38 g/dL (380 g/L) Bilirubin 3.5 mg/dL (60.0 µmol/L) Direct antiglobulin test Negative An osmotic fragility test performed on a non-incubated specimen of blood demonstrated no increase in lysis. Of the following, what BEST explains this patient's laboratory results? A.her erythrocytes have increased lipid peroxidation B.her specimen was not incubated prior to osmotic fragility testing C.she does not have hereditary spherocytosis D.she has less penetrance than her family members

Hereditary spherocytosis (HS) is a group of heterogeneous disorders caused by a congenital defect of vertical integration between the cell membrane and the underlying cytoskeleton of the erythrocyte. Membrane cytoskeleton abnormalities associated with HS include deficiency of spectrin only (most common), deficiencies in ankyrin and spectrin, and defects in band 3 protein and protein 4.2. It is commonly inherited in an autosomal dominant fashion with variable penetrance. Autosomal recessive forms of HS constitute approximately 20% of HS cases, often having a more severe hemolytic anemia associated with early diagnosis and neonatal transfusion requirements. The patient in this vignette has a strong family history of HS, a normocytic normochromic anemia, elevated mean corpuscular hemoglobin concentration, indirect (unconjugated) hyperbilirubinemia, and a negative direct antiglobulin test result. In addition to her symptoms and family history, if spherocytes and microspherocytes are present on her peripheral blood smear, her diagnosis is most consistent with HS. To support this suspected diagnosis, an osmotic fragility (OF) test was obtained. However, her osmotic fragility test was performed on a non-incubated specimen of blood, which may not be informative in determining the presence or absence of HS. To confirm a diagnosis of HS, an increase in red blood cell lysis is observed in an incubated OF test. An incubated OF test is not required in all clinical cases. An incubated OF test requires a blood sample to be incubated at 37°C for 24 hours prior to OF testing. During incubation, HS cells lose membrane surface area and demonstrate greater osmotic lysis when exposed to different concentrations of saline as compared to unaffected red blood cells. A non-incubated specimen may result in a true-positive OF test result only when there are greater than 1% to 2% circulating spherocytes amongst the erythrocytes at the time of the test. Approximately 25% of patients with HS will have a false-negative result when a non-incubated specimen is used for the OF test, as in this vignette. Lipid peroxidation is a marker of erythrocyte oxidation that is seen in enzyme disorders and other forms of oxidative stress. Increased peroxidation may result in a false-positive OF test result due to increased red blood cell fragility but never a false-negative OF test result, as in this vignette. Reticulocytosis may also lead to a false-negative OF result. To determine whether the patient in this vignette demonstrates less penetrance of the HS mutation than her affected family members, a properly conducted, incubated OF test or an eosin-5'-maleimide (EMA) binding assay needs to be performed. The EMA binding assay uses flow cytometry to measure the EMA dye binding covalently to band 3 protein, an anion transport protein found in abundance in normal red blood cells. Decreased EMA binding is suggestive of HS and is increasingly being used to diagnose HS, especially in newborns and infants up to 6 to 12 months of age for whom there are no established normal values for an incubated OF assay. The Figure is a diagnostic algorithm for a newborn suspected to have HS. PREP Pearls An incubated osmotic fragility test is commonly used to help diagnose hereditary spherocytosis. About 25% of patients with hereditary spherocytosis can have falsely normal results on osmotic fragility tests when testing non-incubated erythrocytes. During incubation at 37°C, red blood cells affected by hereditary spherocytosis lose membrane surface area and demonstrate greater osmotic lysis when exposed to different concentrations of saline as compared to unaffected red blood cells. The eosin-5'-maleimide binding assay is becoming more accessible and can be helpful in the diagnosis of hereditary spherocytosis, especially in newborns. ABP Content Specifications(s)/Content Area

A 16-year-old adolescent girl with a history of poorly controlled asthma is admitted to the intensive care unit with worsening back pain and malaise. She has bilateral lower lobe crackles, cardiac systolic ejection murmur, and no adenopathy or hepatosplenomegaly. A chest radiograph shows bilateral lower lobe infiltrates. Echocardiogram reveals decreased left ventricular ejection fraction with a pericardial effusion. Laboratory data are shown: Laboratory Test Result Hemoglobin 13.3 g/dL (133 g/L) Platelet count 121 × 103/µL (121 × 109/L) White blood cell count 33,000/µL (33.0 × 109/L) Neutrophils 38% Lymphocytes 10% Monocytes 2% Eosinophils 50% Creatinine 1.25 mg/dL (111 µmol/L) C-reactive protein 16.4 mg/dL (1,562 nmol/L) Alanine aminotransferase 96 U/L Aspartate aminotransferase 139 U/L Lactate dehydrogenase 2,113 U/L IgE 1,104 µg/L (1.1 mg/L) Vitamin B12 965 pg/mL (712 pmol/L) Peripheral blood flow cytometry Negative for leukemic blasts Of the following, the MOST appropriate next test to obtain to facilitate diagnosis is A.computed tomography of chest and abdomen B.fluorescence in situ hybridization of bone marrow C.pulmonary function test D.serum tryptase level

Hypereosinophilia is defined as a peripheral blood eosinophil count greater than 1,500/µL (1.5 × 109/L). Elevated eosinophil counts may be either secondary to a nonhematologic process or a primary myeloproliferative neoplasm caused by a clonal abnormality. Patients who have persistent hypereosinophilia, evidence of end-organ damage, and systemic symptoms require a more extensive evaluation for an underlying cause. The physical examination findings and peripheral blood hypereosinophilia without lymphoblasts or myeloblasts seen for the patient in the vignette indicate that the most likely diagnosis is a clonal hematologic abnormality. Fluorescence in situ hybridization (FISH) for a myeloproliferative mutation in peripheral blood or bone marrow is the most efficient diagnostic test to confirm this impression. An additional supportive test is a serum tryptase level, which is elevated both in systemic mastocytosis and in hypereosinophilic myeloproliferative disorders and therefore not diagnostic. Although computed tomography to evaluate for lymphoma or adenocarcinoma would help to identify a cause of secondary hypereosinophilia, in this vignette there is no evidence of adenopathy or hepatosplenomegaly. Pulmonary function tests will help to determine end-organ involvement caused by inflammation related to hypereosinophilia but are not diagnostic. The incidence and prevalence of hypereosinophilia are not well understood. The frequency of eosinophilia caused by the FIP1L1-PDGFRA fusion in patients with hypereosinophilia ranges from 3% to 56% (median, 23%). The male-to-female ratio has been reported as 1.47, and rates of disease increase with age. The World Health Organization (WHO) revised the classification of eosinophilic disease in 2008, creating a new major category: myeloid and lymphoid neoplasms with eosinophilia and abnormalities of PDGFRA, PDGFRB, and FGFR1. Also included in the WHO semi-molecular classification scheme is chronic eosinophilic leukemia, not otherwise specified (see Gotlib J). Previous criteria had required that eosinophilia persist for 6 months, but this requirement is less consistently applied today because of improved diagnostic techniques and the need for prompt treatment in some patients to reduce organ damage. The most common presentation of hypereosinophilia includes weakness and fatigue, cough, dyspnea, rash, fever, and myalgias or angioedema. Leukocytosis with peripheral eosinophilia ranging from 30% to 70% of the overall white blood cell count is also common. Anemia and thrombocytopenia may also be present. End-organ damage can result from elevated eosinophil counts, so it is important to identify and address the underlying cause. Pulmonary manifestations occur in up to 40% of patients, with cardiac disease not related to other causes identified in 20%. Current guidelines suggest that evaluation for hypereosinophilia should begin with a detailed history and physical examination with specific assessment for allergic disorders, rashes, and cardiorespiratory, gastrointestinal, and systemic symptoms to rule out secondary (reactive) eosinophilia (Figure). Common secondary causes include parasitic infection, neoplasms, and allergic reactions. Careful attention should be given to travel and drug exposures. Patients who have an elevated eosinophil count without an obvious cause should be investigated for a hematological neoplasm via FISH for FIP1L1-PDGFRA, ETV6-ABL1, ETV6-FLT3, or JAK2 mutations. The FIP1L1-PDGFRA fusion leads to eosinophil proliferation and a myelodysplastic syndrome. A more detailed discussion of the molecular analysis of hypereosinophilia is available in the WHO guideline on eosinophilic disorders (doi:10.1002/ajh.22062). End-organ damage is the most immediate concern with hypereosinophilia. Unprovoked thrombotic events may also occur. Emergency treatment for severe or life-threatening inflammation from hypereosinophilia is corticosteroids. Treatment for clonal eosinophilia depends on the molecular abnormality. Imatinib is a current treatment option for patients with FIP1L1-PDGFRA or ETV6-ABL1, sunitinib or sorafenib are treatment options for patients with ETV6-FLT3 fusion, and ruxolitinib is a treatment option for patients with JAK2 mutation. Historically, overall clinical outcomes have been poor but may be improved with more targeted therapy. PREP Pearls Hypereosinophilia is defined as a peripheral blood eosinophil count greater than 1,500/µL (1.5 × 109/L) which can be either secondary to a nonhematologic process such as infection, allergy, or malignancy, or due to a primary myeloproliferative neoplasm with a clonal abnormality. Patients who have persistent hypereosinophilia, evidence of end-organ damage, and systemic symptoms need to undergo an extensive evaluation for an underlying cause. Emergency treatment for severe or life-threatening inflammation resulting from hypereosinophilia is corticosteroids. Treatment for clonal eosinophilia associated with FIP1L1-PDGFRA, ETV6-ABL1, ETV6-FLT3, or JAK2 mutations depends on the molecular abnormality.

A 17-year-old adolescent boy presents with a second relapse of Hodgkin lymphoma, 3 months after receiving autologous stem cell transplantation followed by post-transplant treatment with brentuximab. He is a potential candidate for a clinical trial involving pembrolizumab. The patient has good functional status, but has incurred some acute and chronic adverse effects from prior chemotherapy and radiotherapy. Of the following, the LEAST likely side effect for this patient if he receives the proposed therapy is A.cytopenias B.hepatitis C.pneumonitis D.rash

Immune checkpoint inhibitors make up a new class of cancer therapy that targets key regulatory pathways in the immune system. Cancer is characterized by uncontrolled growth and invasion, facilitated in part by an ability to evade immune surveillance. Immune checkpoint inhibitors work by re-enlisting the immune system in the fight against tumor cells. These therapies, including nivolumab, pembrolizumab, and ipilimumab, have been variably approved for use in the treatment of melanoma, non-small cell lung cancer (NSCLC), refractory or multiply relapsed Hodgkin lymphoma, renal cell carcinoma, urothelial carcinoma, and squamous cell carcinoma of the head and neck. Immune checkpoint inhibitors have resulted in durable remissions in diseases that are difficult to treat, and they are generally well-tolerated. The primary toxicities of immune checkpoint inhibitors are related to immune-mediated adverse events, such as rashes, pneumonitis, colitis, pancreatitis, hypophysitis, and hepatitis. Significant cytopenias are not a common side effect of these agents. The immune system includes inhibitory cells such as myeloid-derived suppressor cells and regulatory T cells, which dampen an activated immune response to prevent autoimmunity. Tumors can recruit suppressive or regulatory immune cells to the tumor microenvironment and thereby downregulate the immune response to the tumor. Neoplastic cells can also express antigens that suppress the immune system, one of which is programmed cell death ligand (PD-L1). Programmed cell death protein 1 (PD-1) receptor is expressed on activated T-cells. The binding of PD-1 with the PD-L1 ligand on cancer cells results in inhibition of T lymphocyte proliferation, survival, cytotoxicity, and cytokine release. This binding also induces apoptosis of tumor-specific T-cells and the differentiation of T-cells into regulatory T-cells that further suppress the immune response. Additionally, effector T-cells can reach a level of exhaustion, mediated by checkpoint molecules including PD-1, cytotoxic T-lymphocyte antigen-4 (CTLA-4), and T-cell lymphocyte activation gene-3 (LAG-3), among others. Inhibition of the PD-1/PD-L1 interaction prevents the down-regulation of T-cell effector functions, thereby maintaining T-cell mediated responses that can result in tumor cell death. Nivolumab is a human IgG4 anti-PD-1 monoclonal antibody approved for the treatment of patients with metastatic melanoma (30%-50% response rate) and metastatic NSCLC (20%-30% response rate). It has been approved for use in multiply relapsed Hodgkin lymphoma and, recently, small cell lung cancer as well. Safety and efficacy are being evaluated in hematologic malignancies and pediatric solid tumors. Pembrolizumab is also a human IgG4 anti-PD-1 monoclonal antibody that is being studied in hematological malignancies. Anti-PD-L1 antibodies have also been developed and are being studied. Cytotoxic T-lymphocyte antigen-4 belongs to the immunoglobulin superfamily and is expressed in activated lymphocytes. It is a negative regulator of T-cell activation and effector function, thereby downregulating the immune response. Ipilimumab is a human IgG1 kappa monoclonal antibody with specificity against CTLA-4. It is approved for the treatment of melanoma and is being tested in hematological malignancies. Combination with PD-1 inhibition appears to be more effective than monotherapy with either drug in solid tumors. These drugs have not been extensively studied in pediatric patients. Heterogeneous immune checkpoint expression, including PD-1 and PD-L1, has been reported in a variety of pediatric cancers, and pediatric studies are currently ongoing. In the event of immune-mediated adverse events, management usually consists of discontinuation of the agent and initiation of corticosteroid therapy. In a large meta-analysis including 11,000 patients, the most common adverse effect was rash in 13.9%, which seemed to be more common with CTLA-4 inhibitors. High-grade adverse effects were generally low, but the potential for late-onset autoimmunity has been reported. Most immune-mediated organ damage has been found to be reversible, but some endocrine organ damage has been irreversible and has necessitated hormone replacement therapy. PREP Pearls Immune checkpoint inhibitors are designed to overcome a tumor's ability to evade immune detection. Immune checkpoint inhibitors have been approved for use in the treatment of melanoma, refractory or multiply relapsed Hodgkin lymphoma, and several adult solid tumors. The primary toxicities of immune checkpoint inhibitors are related to immune-mediated adverse events, such as rashes, pneumonitis, colitis, pancreatitis, hypophysitis, and hepatitis.

A previously healthy 2-year-old boy has a 4-day history of fever, rhinorrhea and cough. He is currently afebrile without lymphadenopathy or hepatosplenomegaly. Laboratory data are as follows: Laboratory Test Result White blood cell count 2,000/µL (2.0 × 109/L) Hemoglobin 11.5 g/dL (115 g/L) Platelet count 450 × 103/µL (450 × 109/L) Neutrophils 20% Lymphocytes 65% Monocytes 10% Eosinophils 5% Of the following, the MOST likely anticipated clinical course for this patient is A.life-long neutropenia B.progression to leukemia C.recurrent neutropenia every 21 days D.resolution of neutropenia in 1 month

In this vignette, the isolated neutropenia with an absolute neutrophil count (ANC) of 0.4 × 109/L is most likely due to a preceding or current viral infection, which, in most cases, resolves spontaneously within 1 month. Life-long mild neutropenia with an ANC typically above 1.0 × 109/L can be seen in benign ethnic (familial) neutropenia, which is associated with African American, Yemenite Jewish, West Indian, and Arab Jordanian ancestries and with mutations in the Duffy antigen/receptor chemokine gene (DARC). Neutropenia associated with a potential progression to acute myeloid leukemia is typically associated with bone marrow failure syndromes, such as severe congenital neutropenia and Shwachman-Diamond Syndrome. Typically these disorders are characterized by multiple cytopenias or pancytopenia with a history of recurrent severe infections during infancy. Recurrent neutropenia on a 21-day cycle as seen in cyclic neutropenia, is a rare condition associated with mutations in the neutrophil elastase gene, ELANE, which leads to a shortened neutrophil lifespan and recurrent infections. The most common causes of neutropenia in children in order of frequency are acquired due to infection, drugs, and less commonly associated with underlying immune disorders. Most infections can be associated with neutropenia; the most frequent ones are summarized in the Table. Childhood infections such as respiratory syncytial virus, influenza, and parvovirus typically cause neutropenia during the first few days of illness with count recovery occurring within weeks to months. When evaluating a patient with neutropenia and signs and symptoms of infection, it is important to consider whether the neutropenia is the cause or result of the infection. Risk for bacterial and fungal infection is related to the degree, duration, and mechanism of neutropenia. An ANC less than 0.5 × 109/L with evidence of decreased bone marrow production and lack of neutrophil reserve correlates with higher infectious risk. Thus, transient, postinfectious, isolated neutropenia in previously healthy, immunocompetent children with no other cytopenias, complaints, or physical abnormalities may portend a lower risk for serious bacterial infections. Because most cases of acquired isolated neutropenia in children are transient, monitoring blood counts every few weeks to months until resolution is a reasonable approach if there are no other cytopenia disorders, history of severe infection, or clinical symptoms to suggest a systemic disease. PREP Pearls The most common causes of neutropenia in children are acquired due to infection or drugs. Childhood infections such as respiratory syncytial virus, influenza, and parvovirus typically cause neutropenia during the first few days of illness with neutrophil count recovery within weeks to months. Since most cases of acquired isolated neutropenia in children are transient, monitoring blood counts every few weeks to months until resolution is a reasonable approach if there are no other cytopenia disorders, history of severe infection, or any clinical symptoms to suggest a systemic disease.

A 5-year-old boy with a history of relapsed acute myeloid leukemia is now day +10 (or 10 days) after a 10/10 human loci antigen (HLA)-matched (with allele compatibility at HLA-A/B/C/DRB1/DQB1 loci), unrelated donor, hematopoietic stem cell transplant (HSCT). His pre-HSCT conditioning regimen consisted of busulfan and cyclophosphamide. He has mucositis and develops a fever to 38.3°C. His absolute neutrophil count is 200/µL (0.20 × 109/L). Blood cultures are obtained from his central venous catheter. Of the following, the MOST likely organism causing his fever is A.Candida albicans B.cytomegalovirus C.herpes simplex virus D.Staphylococcus epidermidis

Infection risk after hematopoietic stem cell transplant (HSCT) is multifactorial. Patients who have received myeloablative conditioning regimens commonly develop mucositis, which increases the risk of bacteremia. Central venous catheters allow direct access of bacteria and fungi into the bloodstream. In addition, immune suppression is compounded by pre- and post-HSCT medications given to prevent graft-vs-host disease (GVHD). Typical anti-GVHD agents include calcineurin inhibitors (cyclosporine, tacrolimus), mTOR inhibitors (sirolimus), methotrexate, and/or mycophenolate mofetil. During the pre-engraftment or neutropenia phase of HSCT, which may last for 2 to 4 weeks after stem cell infusion, the risk of infection is greatest for bacterial and fungal infection (Figure). During the pre-engraftment period, the risk of bacterial infection is highest between 15% to 50%, while the risk of invasive fungal infection is 10% to 20%. The patient in this vignette developed fever during the pre-engraftment phase, when risk is highest for bacterial infections with the added risks from an intact central venous catheter and concurrent mucositis. Although the risk for bacterial infection decreases after neutrophil engraftment, bacterial infection risk persists until a patient's central venous catheter is removed and immunosuppressive anti-GVHD agents are withdrawn. The patient in this vignette has had a fever for only one day, which makes a bacterial infection more likely than a fungal infection. Concern for fungal infection increases after a prolonged period of fever despite broad-spectrum antibiotics. Traditionally, fever lasting more than 4 days raises concern for fungal infection and indicates the need for fungal surveillance imaging, such as computed tomography. After stem cell engraftment, risks can be categorized into the early engraftment (up to day +100) period and the late engraftment (after day +100) period. During early engraftment, after the absolute neutrophil count (ANC) has recovered (defined as an ANC of 0.5 x 103/μL for 3 consecutive days), immune competence is not completely restored, so the risk of bacterial and fungal infection is less than in the pre-engraftment phase but still increased because of indwelling catheters and resolving mucositis. During the early engraftment phase, patients continue to receive anti-GVHD prophylaxis directed at T cells, leading to an increased risk of viral reactivation from the herpetic family of viruses (cytomegalovirus, Epstein-Barr virus, human herpesvirus 6) and adenovirus. Adenovirus may be seen in up to 20% of patients receiving allogeneic HSCT. Respiratory viral infections are observed in approximately 10% of the pediatric population after HSCT. Infection with any of these viruses may lead to increased morbidity and mortality. During late engraftment, patients are gradually weaned from immunosuppressive medications over the next 1 to 2 years allowing for gradual recovery of lymphocyte function. Reactivation of varicella accounts for 40% of all infections during this phase. Recovery of immune competence will vary from patient to patient depending on the underlying disease pre-HSCT, the associated pre-HSCT conditioning regimen, and anti-GVHD regimen. For example, patients with malignancy, such as leukemia, are weaned from anti-GVHD medications around day +100 after HSCT, while patients with nonmalignant diseases, such as sickle cell disease, are weaned closer to day +180. The shortened time to wean these medications in leukemias is related to enhancing the beneficial graft-vs-leukemia effect after HSCT. PREP Pearls Infection risk after hematopoietic stem cell transplant relates to the time of different white blood cell lineage engraftment, time until recovery from conditioning regimen-related mucositis, nutritional status, and the presence of central venous catheters. During the pre-engraftment or neutropenia phase of hematopoietic stem cell transplant, approximately 2 to 4 weeks after stem cell infusion, patients are at highest risk for bacterial and fungal infections. During the engraftment phase of hematopoietic stem cell transplant, after neutrophil recovery, the heightened risk for herpetic viral reactivation (cytomegalovirus, Epstein-Barr virus, herpes simplex virus, human herpesvirus 6) and nosocomial viral infection (adenovirus, influenza, respiratory syncytial virus) is due to graft-vs-host disease prophylactic medications delaying lymphocyte recovery. Hematopoietic stem cell patients remain immune compromised until they are completely weaned off of graft-vs-host disease prophylactic medication

A previously healthy 26-month-old boy has a several-day history of fever, persistent vomiting, lethargy, and hematuria. A computed tomographic scan of the abdomen shows a renal mass 3 cm × 3.5 cm × 2 cm with areas of hemorrhage and necrosis. Magnetic resonance imaging of the brain shows a 2-cm × 2.5-cm × 1.5-cm heterogeneous mass in the midline posterior fossa. Biopsy of the renal mass shows an alveolar pattern of large monomorphic cells that stain positive for keratin, vimentin, epithelial membrane antigen, desmin, and neurofilament and stain negative in WT1 and INI1 nuclear staining. Of the following, the factor that has been MOST linked with increased 4-year overall survival in this primary renal tumor is A.age greater than 24 months B.hypocalcemia at diagnosis C.male sex D.treatment with doxorubicin

Malignant rhabdoid tumor of the kidney (MRTK) is rare and represents less than 2% of pediatric renal tumors. Although MRTK commonly presents with fever and hematuria, unlike Wilms tumor, 75% of cases present with advanced-stage disease (stage 3 or higher). The median age at diagnosis of MRTK is 11 months. Approximately 20% of MRTK cases exhibit concurrent central nervous system (CNS) tumor involvement, which usually represents a second primary rhabdoid tumor known as atypical teratoid rhabdoid teratoma, likely owing to a genetic cancer predisposition. Rhabdoid tumors may present in a variety of sites, including the kidney, soft tissues, and the CNS. Malignant rhabdoid tumor of the kidney has a poor prognosis, with a 4-year overall survival rate of less than 25%. Rhabdoid tumors are all associated with INI1 mutations on chromosome 22, which differentiates them from other soft-tissue neoplasms (including rhabdomyosarcoma, which often has a similar pathological appearance). Loss of INI1 expression leads to loss of the protein and negative staining results as seen in this vignette. Although relatively specific for rhabdoid tumors, INI1 mutations have also been associated with epithelioid sarcomas and renal medullary carcinomas. Renal medullary carcinoma usually does not present before age 10 years and is more common in those with sickle cell trait. Epithelioid sarcomas are slow-growing soft-tissue sarcomas, which usually present in teens and young adults as hard, nonpainful masses in the extremities involving the subcutaneous tissue, fascia, and tendons but may invade blood vessels, nerves, and dermis, leading to pain and ulceration of the masses. Historically, children diagnosed with MRTK have been treated according to National Wilms' Tumor Study protocols, with a reported decreased 4-year overall survival associated with CNS tumors and a younger age at diagnosis, particularly in those aged 0 to 5 months. The highest survival rates had been noted in patients older than 24 months. Although a slightly higher proportion of males have been diagnosed with MRTK, sex has not had a statistically significant impact on survival rates. Treatment regimens involving doxorubicin have not shown any statistically significant difference in 4-year overall survival compared with regimens without doxorubicin, yet in one study preoperative administration of doxorubicin did demonstrate a larger decrease in tumor volume. Hypocalcemia has not been associated with prognosis in pediatric renal tumors. However, both congenital mesoblastic nephroma, a benign renal tumor of infancy, and MRTK have been associated with hypercalcemia owing to tumor secretion of a parathyroid hormone-related protein. This hypercalcemia typically resolves when the tumor is removed. PREP Pearls Malignant rhabdoid tumor of the kidney represents less than 2% of pediatric renal tumors, with a median age at diagnosis of 11 months and a typical presentation of fever, hematuria, and advanced-stage disease. Presence of a central nervous system tumor and younger age at diagnosis have been associated with decreased 4-year overall survival in patients with malignant rhabdoid tumor of the kidney. Malignant rhabdoid tumor of the kidney and atypical teratoid rhabdoid tumor are associated with germline INI1 mutations. Hypercalcemia results from the release of parathyroid hormone-related protein in malignant rhabdoid tumor of the kidney and resolves with tumor resection.

A healthy 1-week-old male neonate who is born to a primigravida mother with an uneventful pregnancy, is experiencing persistent blood oozing from the umbilical stump, soaking a clean gauze pad every few hours. The newborn was delivered atraumatically at home with the assistance of a trained midwife. He has received no medications and is breastfed. The mother takes no medications except prenatal vitamins. Although father's family members have prolonged bleeding after minimal trauma, there are no bleeding disorders in either parent or in the mother's family. The infant has stable vital signs, is alert, is uncircumcised, with an oozing umbilical stump and a 5-cm scalp hematoma. Results of a complete blood count are normal. Of the following, the BEST next approach would be to A.administer vitamin K intramuscularly B.check factor VIII and IX activities C.reassure and continue to observe D.transfuse platelets

Oozing of blood from the umbilical stump at 1 week of age that is persistent, excessive, or both is a manifestation of a congenital or acquired bleeding disorder. In an infant, other worrisome signs or symptoms that warrant investigation and intervention include excessive hemorrhage after circumcision, easy bruising, cephalohematomas, subgaleal and/or intracranial hemorrhage. Therefore reassurance and observation alone is not the next best approach to the infant in this vignette. Neonates are vitamin K-deficient at birth owing to minimal placental transfer, lack of gastrointestinal bacteria to make sufficient vitamin K, and low amounts of vitamin K in breastmilk. Home deliveries in which newborns have not received oral or intramuscular vitamin K are at high risk of vitamin K deficiency-associated bleeding (VKDB), previously known as hemorrhagic disease of the newborn. Based on the history and clinical presentation, the child in this vignette needs an immediate dose of intramuscular vitamin K to avoid additional bleeding symptoms. Although the exact incidence of VKDB is not known, it is estimated at 5 per 100,000 births in developed countries owing to the universal administration of vitamin K at birth. In developing countries, the incidence was reported to be 50 to 80 per 100,000 births before the institution of vitamin K prophylaxis. Depending on the time of presentation, VKDB has been classified into three distinct entities—early, classical, and late—associated with separate risk factors and presentations (Table). Vitamin K is a cofactor that is responsible for γ-carboxylation of several procoagulants (factors II, VII, IX, and X) and anticoagulants (protein C, protein S, and protein Z). Without γ-carboxylation, these proteins display very limited function, resulting in abnormal coagulation. Individuals with mild or transient vitamin K deficiency can have an isolated prolongation of prothrombin time (PT) owing to depletion of factor VII, which has the shortest half-life of the vitamin K-dependent proteins. However, prolonged or severe deficiency of vitamin K results in prolongation of both the PT and activated partial thromboplastin time (aPTT) owing to depletion of factors II, IX, and X, with signs and symptoms of excessive bleeding. The American Academy of Pediatrics recommends a single intramuscular dose of 0.5 to 1 mg phylloquinone (phytonadione) as vitamin K prophylaxis administered shortly after birth. Daily oral vitamin K doses have not been shown to be as effective. In infants who have developed VKDB, intramuscular or subcutaneous injection of 1 to 2 mg/day vitamin K results in reversal of the hemostatic defect within a few hours. Intravenous administration of vitamin K can cause anaphylactic shock and is used only in life-threatening clinical situations. A systematic approach to determining the cause of excessive bleeding in an infant must include a thorough evaluation of the family history as well as prenatal and perinatal complications. Causes of neonatal thrombocytopenia include maternal viral infections (e.g. cytomegalovirus), and maternal illness such as systemic lupus erythematosus or preeclampsia. Neonatal sepsis associated with prolonged rupture of membranes, chorioamnionitis, or intrauterine infection can also present with bleeding or clotting problems. In addition, several inherited bleeding disorders (e.g. deficiencies of factors VIII, IX, XI, and XIII) or antibody-mediated platelet disorders (e.g. neonatal alloimmune thrombocytopenia) may present during the neonatal period with bleeding symptoms. Significant head trauma associated with labor and vaginal delivery or child abuse can also result in excessive bleeding without any other inherited or acquired predisposition. Typically, treatment of acute, excessive bleeding in infants with platelet aggregation defects or severe thrombocytopenia include platelet transfusions. In this vignette, the child has a normal complete blood count and no evidence of petechiae which is a common manifestation of platelet aggregation defects, therefore a platelet transfusion is not indicated. Factor VIII deficiency and Factor IX deficiency (Hemophilias A and B, respectively) are X-linked disorders that manifest with bleeding symptoms in male infants who inherit the defective X chromosome from a carrier mother. In this vignette, members of the father's side of the family report bleeding after minimal trauma, NOT the infant's father, mother, or mother's family. PREP Pearls Vitamin K is responsible for γ-carboxylation or functional activation of procoagulant factors II, VII, IX, and X and anticoagulant protein C, protein S, and protein Z. Refusal of vitamin K administration at birth and exclusive breastfeeding are risk factors for development of classical vitamin K deficiency, which presents with excess bleeding of the skin, umbilical stump, circumcision site, or gastrointestinal tract. The American Academy of Pediatrics recommends a single intramuscular dose of 0.5 to 1 mg phylloquinone (phytonadione) as vitamin K prophylaxis to be administered shortly after birth.

13-year-old female with no history of immune deficiency presented with shortness of breath, abdominal distension, abdominal pain, and urinary frequency and urgency. Imaging revealed abdominal mass, ascites, and pleural effusions. Pleural fluid aspirate was positive for diffuse large B-cell lymphoma (DLBCL). Her central nervous system (CNS) and marrow were not involved. After three cycles of chemotherapy, follow-up imaging studies showed her to be in complete remission with no evidence of disease. Following the completion of therapy, what is the appropriate off-therapy follow-up for this patient? a. PET imaging of the chest, abdomen, and pelvis at regular intervals for 1 year b. PET imaging of the chest, abdomen, and pelvis at regular intervals for 2 years c. PET imaging of the chest, abdomen, and pelvis at regular intervals for 5 years d. None of the above

PET and PET-CT are used to diagnose, stage, and monitor how well treatment is working. Available evidence from clinical studies suggests that using PET or PET-CT to monitor for recurrence does not improve outcomes and therefore generally is not recommended for this purpose. False-positive PET-CT tests can lead to unnecessary and invasive procedures, overtreatment, unnecessary radiation exposure, and incorrect diagnoses. Off-therapy follow-up for patients with pediatric Hodgkin and non-Hodgkin lymphoma should include medical history and physical examination at regular intervals to evaluate for possible recurrence. Although not a specific content specification, surveillance CT imaging for patients who have completed therapy is controversial. While many oncologists use surveillance CT imaging at regular intervals following completion of therapy, there is no proven survival benefit from routine surveillance imaging over clinical surveillance. In the absence of concerning signs, symptoms, or laboratory abnormalities, additional imaging for patients who have obtained a complete response while on therapy may not be indicated. Monitoring for late effects of chemotherapy should be performed according to evidence-based guidelines.

An 8-year-old boy has new-onset pallor 2 weeks after he arrived home from camp. One year ago, he had received a related, allogeneic hematopoietic stem cell transplantation for β thalassemia major. He had been transfusion independent for the past 9 months with normal complete blood cell counts. He takes tacrolimus. He had been exposed at camp to a counselor who had developed fever with subsequent joint pain one day after camp ended. Today, he reports fatigue but denies fever, diarrhea, and weight loss. He is pale and has no rashes, adenopathy, or hepatosplenomegaly. His serum complete metabolic panel and complete blood count are normal except for a hemoglobin level of 3 g/dL (30 g/L) and reticulocyte count of 0.05%. Of the following, the MOST likely cause of his severe anemia with reticulocytopenia is A.graft failure B.graft-vs-host disease C.iron deficiency D.parvovirus B19 infection

Parvovirus B19 is mainly transmitted by respiratory secretions, maternal-fetal transmission (resulting in hydrops fetalis), and rarely by blood products. Exposure is prevalent and increases over a life-span, with 40-80% of adults possessing B19-specific, IgG antibodies. In healthy children with no underlying conditions, parvovirus B19 infection is associated with Fifth disease, a historical reference to the fifth pediatric viral exanthem described, also referred to as erythema infectiosum. Days to weeks after resolution of flu-like symptoms, such as fever and body aches, an initial "slapped cheek"-appearing, erythematous, macular facial rash then extends to the trunk and extremities as a pink, slightly raised, lacy pattern that waxes and wanes for up to 3 weeks. In adults and older children, the primary symptoms of fever and headache later give rise to polyarticular arthritis instead of the rash, as described in the camp counselor in this vignette. Signs and symptoms result from the antigen-antibody complexes that form in response to parvovirus infection, with eventual halting of virulence by initial IgM production within 1-2 weeks, lasting 3 to 6 months, and sustained with concurrent IgG specific production within 1-2 weeks, lasting decades. Parvovirus B19 is the most common cause of viral-associated "aplastic crisis" in children with sickle cell disease and other congenital hemolytic anemias. The viral DNA has an affinity for binding to the P antigen on red blood cells, which results in viral invasion into erythroblasts and arrest of erythroblast production. In individuals with underlying congenital hemolytic anemia who are dependent on the brisk rate of erythropoiesis to maintain their normal baseline hemoglobin level, the arrest of erythroblast production causes a rapid drop in hemoglobin levels. Parvovirus B19 virulence is stopped by IgG formation targeting the virus. Supportive care often includes packed red blood cell transfusions. In the patient who is immunosuppressed, the ability of the immune system to arrest parvovirus B19 infection is impaired or absent entirely. The virus can affect other cell lines, resulting in myelosuppression and cytopenia due to unknown mechanisms. Diagnosis of parvovirus often requires serologic testing for IgG and IgM antibodies in addition to quantitative polymerase chain reaction for parvovirus B19 DNA. In immunosuppressed children, treatment involves decreasing immunosuppressive medication and enhancing adaptive immunity with intravenous immunoglobulin. In this vignette, given the previous direct infectious exposure to the camp counselor and timeline of anemia presentation in an immunosuppressed child, infection with parvovirus is highly likely. Severe iron deficiency anemia seems unlikely given normal complete blood cell counts prior to camp, transfusion independence for 9 months, and no bleeding symptoms. Graft failure rarely manifests as isolated anemia and more typically with pancytopenia. Acute graft-vs-host disease is unlikely given the absence of dermatologic, hepatic, or gastrointestinal tract involvement. PREP Pearls Parvovirus B19 infection, which is spread by droplets and rarely blood-borne, can cause transient hypoplastic or aplastic anemia in immunocompromised individuals or in individuals with congenital hemolytic anemia. In children, parvovirus B19 infection is classically associated with manifestations of Fifth disease such as fever, body aches, and reddened cheeks followed by a pink, lacy body rash, while in older teens and adults, fever followed by bone and joint pain is more common.

A 6-month-old female infant is in hypovolemic shock following an acute illness with fever, vomiting, and diarrhea. All growth parameters are below the 10th percentile. After resuscitation, evaluation reveals severe metabolic acidosis, failure to thrive, and severe pancytopenia. Stool pancreatic elastase levels are less than 10 μg/g, implying exocrine pancreatic insufficiency. Bacterial cultures and viral studies have negative results. There is no family history of anemia, and the rest of her physical examination findings are normal. Bone marrow examination demonstrates abnormal marrow precursors (Figure). Southern blot analysis of blood and bone marrow confirms a large mitochondrial DNA deletion of approximately 4 kb, spanning ND4 to CYTB. Of the following, the MOST likely diagnosis is A.Diamond-Blackfan anemia B.hereditary sideroblastic anemia C.Pearson syndrome D.Shwachman-Diamond syndrome

Pearson syndrome, also known as Pearson marrow-pancreas syndrome, is caused by gene deletion mutations in mitochondrial DNA. The result is defective oxidative phosphorylation leading to multisystem dysfunction. The classic hematologic findings in Pearson syndrome are refractory sideroblastic anemia and cytoplasmic vacuolization in hematopoietic precursors. Disorders caused by mitochondrial DNA mutations are inherited maternally. However, most cases of Pearson syndrome are due to sporadic mutations as most children die in infancy or early childhood. The family history helps to distinguish Pearson syndrome from other bone marrow failure syndromes such as sideroblastic anemia (X linked) and Diamond-Blackfan anemia (DBA) (autosomal dominant, rarely X linked). Vacuolization of hematopoietic precursors seen in Pearson syndrome (Figure) does not occur in hereditary sideroblastic anemia, DBA, or Shwachman-Diamond syndrome (SDS). Diamond-Blackfan anemia is a pure red cell aplasia most often caused by mutations in ribosomal protein genes. Erythrocyte adenosine deaminase is elevated in DBA. Nearly 50% of children with DBA have associated physical abnormalities such as hand (triphalangeal thumbs) and upper limb defects, craniofacial anomalies, cardiac anomalies, and urogenital malformations. Similar to Pearson syndrome, SDS is characterized by exocrine pancreatic insufficiency, neutropenia, and anemia. Children with SDS have less severe anemia compared to those with Pearson syndrome. In SDS, bony abnormalities such as epiphyseal and metaphyseal dysostosis often occur. Vacuolization of hematopoietic precursors unrelated to Pearson syndrome occurs in children whose diets do not contain sufficient amounts of riboflavin, phenylalanine, or copper. Chloramphenicol may also cause vacuolization in hematopoietic precursor cells. Pearson syndrome should be suspected in an infant with failure to thrive, fatty stools, pancytopenia, refractory anemia, chronic diarrhea, severe infections, metabolic acidosis, and lactic acidemia. Multisystem involvement is variable; however, most children will have bone marrow failure and exocrine pancreatic insufficiency. Hepatic, renal, cardiac, and endocrine failure are common. Unlike other mitochondrial disorders, Pearson syndrome does not cause neuromuscular weakness. Children with Pearson syndrome usually die in infancy or early childhood. The median survival is 4 years. The most common cause of death is infection, metabolic disturbance, or hepatic failure. With early recognition of the syndrome and aggressive supportive care, affected children can survive longer. Prompt evaluation of fever and initiation of antibiotics along with medical management of metabolic acidosis and electrolyte disturbance is key to prolonging survival. Red blood cell transfusions are needed for the refractory anemia. Respiratory enzyme activity may be improved with the administration of coenzyme Q10 and levocarnitine. Exocrine pancreatic insufficiency is treated with the replacement of pancreatic enzymes and fat-soluble vitamins. The drugs that are toxic to mitochondria and should not be given to children with Pearson syndrome include chloramphenicol, aminoglycosides, linezolid, valproic acid, and nucleoside reverse transcriptase inhibitors. Some children may recover marrow and pancreatic function with age, which underscores the need for prompt recognition and aggressive supportive care. Pearson syndrome is probably underdiagnosed and must be considered in infants and children with cytopenias associated with sepsis, acidosis, and renal or liver disease. PREP Pearls Pearson syndrome is a refractory sideroblastic anemia associated with cytoplasmic vacuolization in hematopoietic precursors and deletions of mitochondrial DNA. Pearson syndrome manifests in infants and children with cytopenias associated with failure to thrive, refractory anemia, sepsis, acidosis, pancreatic insufficiency, and renal or liver disease. Pearson syndrome is often fatal in infancy or early childhood with a median survival of 4 years.

A 15-year-old adolescent boy is referred by the endocrinology service for a serum ferritin level of 2,430 µg/L and transferrin saturation of 86%, which were identified during an evaluation of delayed puberty. Repeat testing yields similar results. He is of Italian heritage, and his parents are distant cousins. Of the following, the gene that is MOST likely mutated in this patient is A.HFE B.HJV C.SLC40A1 D.TFR2

Persistent, extreme elevation of serum ferritin and transferrin saturation are highly suggestive of disorders of iron metabolism, such as hereditary hemochromatosis (HH), severe forms of which can lead to significant multiorgan dysfunction caused by chronic iron deposition. Although very rarely symptomatic or recognized in the pediatric population, when severe HH is identified in young patients, hypogonadism and cardiac involvement are the most commonly encountered clinical features. Abnormal skin pigmentation, diabetes, arthropathy, and liver fibrosis are additional possible sequelae. Despite the significant iron loading, HH can also be clinically silent, especially at younger ages. The hepcidin-ferroportin axis is key to understanding iron metabolism and the 4 primary types of HH (types 1-4) and their genetic causes. Hepcidin (encoded by HAMP), a small peptide primarily synthesized in the liver, is the critical regulatory hormone governing iron homeostasis. Ferroportin (encoded by SLA40A) is a transmembrane molecule responsible for exporting iron across the membranes of enterocytes (basolateral side) and macrophages. Ferroportin mediates dietary iron absorption along with iron release from the reticuloendothelial system, which are the 2 primary sources of iron to meet total body demand. The binding of hepcidin to ferroportin leads to endocytosis and degradation of ferroportin, thus down-regulating cellular iron export and reducing circulating iron concentration. Conditions that impact hepcidin levels include inflammation and iron deficiency, which generally increase and decrease hepcidin expression, respectively, to titrate plasma iron concentration. Relative hepcidin deficiency underlies the first 3 types of HH, whereas ferroportin defects account for type 4 HH. Type 1 HH, also known as classical or HFE-related hemochromatosis, is caused by specific mutations in HFE, with autosomal inheritance. Although the precise mechanism is unclear, HFE, expressed along the lumen of the gastrointestinal tract, regulates hepcidin and plays an important role in absorption of dietary iron. Homozygosity for the C282Y mutation in HFE is the most common cause of HH worldwide, since the allele frequency for this variant is around 10% among Europeans, leading to an estimated prevalence of 0.4% to 0.5% for the homozygous state in the United States and Western Europe. The penetrance of this genotype, however, is not high, and other factors are thought to modulate disease expression, resulting in phenotypic variability among homozygous individuals. Many homozygous individuals remain asymptomatic throughout their lifetimes, while others develop some of the HH-related complications listed above. Another HFE variant (H63D) contributes to some cases through compound heterozygosity with C282Y. Type 1 HH is generally recognized as the mildest of the 4 types, as affected individuals do not develop significant iron overload until middle age (40-60 years old). Many cases are only identified from family screening studies performed after classical HH has been diagnosed in an affected relative or after routine screening chemistries suggest possible iron overload (eg, transferrin saturation > 45%, ferritin > 150-200 µg/L). To avoid misdiagnosis of hemochromatosis, transferrin saturation levels should be drawn as fasting, early morning blood specimens, and testing must be repeated to confirm abnormal results. Types 2 and 3 of HH each have much lower estimated population prevalence of approximately 1 in 5 to 6 million and are inherited in an autosomal recessive manner. Type 2A HH is mediated by genetic lesions in HJV encoding hemojuvelin, while Type 2B HH results from mutations in HAMP. Both mutated genes are thought to modulate hepcidin expression. Types 2A and 2B HH or "juvenile hemochromatosis" can cause significant iron loading during the first decade of age, which may be clinically silent. As patients age, hypogonadism and cardiac disease (eg, cardiomyopathy, arrhythmias) become the most common early complications. Many cases of HJV-related hemochromatosis have been reported in Italy. Type 3 HH is caused by mutations in the gene encoding transferrin receptor 2 (TFR2). Type 3 HH has disease severity that is intermediate between that of type 1 and type 2 HH. Autosomal dominantly transmitted mutations in SLC40A1 (encoding ferroportin) account for 2 distinct subtypes of type 4 HH, classified according to the functional impact on ferroportin. Type 4A defects impair the iron export function of ferroportin. Type 4A HH individuals feature low or normal transferrin saturations as iron accumulates in reticuloendothelial cells. Mild anemia may also be present, possibly due to reduced iron availability to the erythron. Type 4A is the most common non-HFE form of HH, with an estimated population prevalence of 1 in 1,400 and more common among individuals of African ancestry. Type 4A is typically mild in clinical severity. Type 4B defects impair hepcidin binding to ferroportin and thus clinically resemble the other hepcidin-deficient HH disorders, although hepcidin levels tend to be elevated because of the functional hepcidin deficiency. The patient described in the vignette is too young to be clinically affected by classical or HFE-related hemochromatosis (type 1 HH). He is also too young and with significant clinical severity, unlike the presentation in individuals with TFR2-related disease (type 3 HH). Hemochromatosis caused by SLC40A1 mutations (type 4 HH) is transmitted in an autosomal dominant manner, may feature low or normal transferrin saturation, and is clinically mild, unlike in this vignette. The patient's consanguineous parents, Italian heritage, and severe phenotype are most consistent with a diagnosis of juvenile hemochromatosis mediated by mutations in HJV (type 2 HH). PREP Pearls Although extremely rare, the 2 forms of juvenile hemochromatosis caused by mutations in HJV (type 2A) or HAMP (type 2 B) can result in significant iron loading during childhood or adolescence with subsequent hypogonadism and cardiac disease being the most common clinical manifestations. Hallmarks of most forms of hereditary hemochromatosis include persistently increased serum ferritin level and transferrin saturation, except for type 4A with low or normal transferrin saturation. To avoid misdiagnosis of hemochromatosis, transferrin saturation levels should be drawn as fasting, early morning blood specimens, and testing must be repeated to confirm abnormal results. Hereditary hemochromatosis types 1, 2A, 2B, and 3 are inherited in an autosomal recessive manner, whereas types 4A and 4B feature autosomal dominant inheritance.

A 15-year-old girl with a history of osteosarcoma presents to survivor clinic for her first evaluation. Her mother complains that she does not listen well and is wondering if she may have trouble hearing. Which of the follow is true regarding platinum-associated hearing loss? a. Platinum chemotherapy is most often associated with conductive hearing loss. b. Low-frequency volumes are affected first. c. Older age at exposure increases risk. d. Platinum-associated hearing loss is due to destruction of the cochlear hair cells. e. Carboplatin is more ototoxic than cisplatin.

Platinum-associated sensorineural hearing loss is due to the destruction of cochlear hair cells. The hair cells are arranged tonotopically; therefore, the high-frequency hair cells (>2000 Hz) are affected first. As cumulative dose increases, injury progresses toward the cochlear apex, where lower frequencies in the audible range are affected. Cisplatin is more ototoxic than carboplatin. Younger age at exposure (younger than 5 years), higher doses, receipt of multiple ototoxic agents, and combination treatment with cisplatin and cranial radiation places patients at increased risk for hearing loss. Radiation is associated with both conductive and sensorineural hearing loss; however, platinum agents are typically associated with sensorineural hearing loss only.

A surveillance metaiodobenzylguanidine (MIBG) scan suggests that a 5-year-old boy with a history of high-risk neuroblastoma has relapsed at the site of his primary disease (paraspinal location in upper abdomen) 12 months off intensive multimodal therapy that included myeloablative therapy with stem cell rescue and anti-GD2 antibody (dinutuximab). Biopsy of the tumor tissue is consistent with neuroblastoma and negative for ALK mutation or amplification. Of the following, the treatment that BEST merits inclusion in subsequent therapy for this patient is A.crizotinib B.metaiodobenzylguanidine radiolabeled with 131I (131I-MIBG) C.nifurtimox D.topotecan and cyclophosphamide as given during original induction

Roughly half of patients with high-risk neuroblastoma who achieve remission eventually relapse. Long-term outcomes for this population are dismal, with 5-year overall survival rates below 20%. Treatment with iodine-131 metaiodobenzylguanidine (131I-MIBG) offers the highest objective response rate (approximately 30%) of any single agent in patients with relapsed or refractory neuroblastoma. Tumor response rates are similar between relapsed and refractory disease, but patients treated in relapse with 131I-MIBG have higher rates of progressive disease and lower overall survival after therapy than patients with refractory disease. Therapy with 131I-MIBG may also reduce pain from neuroblastoma and thus fulfill a palliative role independent of any objective tumor response. Despite some tumors responding to this targeted radiotherapy, subsequent disease progression remains high. There is a critical need for better treatment options that may include 131I-MIBG in synergistic combination with other agents or modalities, including novel targeted therapies. Crizotinib is an anaplastic lymphoma kinase (ALK) inhibitor approved for use in adults with non-small cell lung cancer, but it has demonstrated activity in some patients with neuroblastoma driven by an ALK mutation or amplification, which occurs in about 10% to 15% of cases. However, the patient in this vignette does not demonstrate aberrant ALK expression. Moreover, given the high rates of crizotinib resistance observed in ALK-mutated neuroblastomas, current trials focus on next-generation ALK inhibitors such as ceritinib. Nifurtimox is an antiparasitic used to treat Chagas disease, but it has demonstrated activity against neuroblastoma in preclinical models and a phase 1 study. Its utility in neuroblastoma is the focus of an ongoing phase 2 study (NCT00601003). Although topotecan and cyclophosphamide are active against neuroblastoma and are common induction agents to treat relapsed disease, reuse of the same chemotherapy agents used to treat the original tumor is not common practice. The underlying biology of relapsed neuroblastoma is likely to have evolved from that of the original lesion at diagnosis, leading to chemoresistance to previously used agents. Metaiodobenzylguanidine is a structural analog of norepinephrine for which about 90% of neuroblastomas have avidity, based on their surface expression of the active human norepinephrine transporter. Due to superior imaging characteristics, 123I is typically used as a radiolabel for diagnostic and staging purposes to scintigraphically identify areas of increased MIBG uptake by tissues of neuroendocrine origin such as neuroblastomas and pheochromocytomas. The use of 131I offers more favorable radiation dosing for therapy purposes, leading to its incorporation into a radiopharmaceutical (131I-MIBG) that has been used for over 30 years to localize cytotoxic radiation to sites of MIBG uptake. Maximal uptake occurs 24 to 96 hours after administration, and elimination is primarily renal. Assuming normal renal function, 50% of the administered dose is cleared by 24 hours and 90% by 96 hours after administration. Treatment with 131I-MIBG requires meticulous radiation isolation of the patient and thus highly specialized training of staff and families as well as unique infrastructure (eg, lead-shielded room) and procedures. It is thus available at only a limited number of sites worldwide (eg, about a dozen in North America as of 2018). Nausea, vomiting, sialadenitis, and hypertension are common adverse effects of 131I-MIBG therapy. Hypertension typically occurs during the first 48 hours after infusion, necessitating careful monitoring and occasional use of antihypertensive therapy. Myelosuppression is the primary dose-limiting toxicity. At typical 131I-MIBG dosing levels, autologous stem cell support is required in about one-third of cases and may be even more likely in a patient with prior myeloablative therapy, due to diminished hematopoietic reserve. Although 131I-MIBG therapy necessitates potassium iodide administration before and for several days after treatment to block thyroid uptake of the radionuclide and prevent radiation injury, hypothyroidism may still ensue, sometimes requiring thyroid hormone replacement therapy. Thyroid nodules and papillary thyroid carcinoma have also been reported, along with secondary myelodysplastic syndrome, transformation to acute myelogenous leukemia, and other secondary malignant neoplasms (SMN). One study reported SMN frequencies of 8% at 5 years and 14% at 10 years after 131I-MIBG therapy, highlighting the importance of long-term surveillance and the uncertainty regarding precise causality of SMNs among heavily treated patients who have received a variety of intensive therapies. Nevertheless, due to high rates of objective response to 131I-MIBG in relapsed and refractory disease, multiple cooperative group trials include it in current study protocols to assess its role in upfront therapy for newly diagnosed high-risk neuroblastoma. Since 131I is a β-emitting radionuclide with a relatively long path length (0.8 mm), one concern is that emitted energy is ineffective at killing cells that take up 131I-MIBG and those in their immediate vicinity, potentially leading to decreased efficacy against micrometastatic disease. Indeed, at least one study suggested better outcomes in patients with isolated soft tissue relapse compared with patients who also had bone marrow involvement (ie, where microscopic disease may exist). Alternative radionuclides with shorter path lengths are thus being evaluated. PREP Pearls Although 131I-MIBG is a primary therapy option for relapsed and refractory neuroblastoma, institutions that offer this treatment are limited, the suboptimal, objective response rate is approximately 30%, and the long-term survival rate is < 20%. The mechanism of action of 131I-MIBG requires human norepinephrine transporter on the surface of targeted cells which is present in 90% of neuroblastoma cases. Toxicities of 131I-MIBG therapy include myelosuppression and hypothyroidism, which may require treatment with autologous hematopoietic stem cell rescue and thyroid replacement therapy, respectively.

A 13-year-old adolescent girl with sickle cell anemia is seen for routine care. She has had rare vaso-occlusive pain crises that have required hospitalization, and 1 episode of acute chest syndrome at 10 years of age. Echocardiography performed at 11years of age revealed normal findings except for a slightly elevated tricuspid regurgitant jet velocity (2.4 m/s). She has no complaints and lung auscultation is clear. She has a resting peripheral capillary oxygen saturation of 90% measured on pulse oximeter which is her baseline. Her laboratory data are shown: Laboratory Test Result White blood cell count 14,500/µL (14.5 × 109/L) Hemoglobin 6.3 g/dL (63 g/L) Reticulocytes 19% Absolute reticulocyte count 654 ×103/µL (654 ×109/L) Lactate dehydrogenase Elevated She was wondering what her risks of developing a leg ulcer are compared to other teenagers. Of the following, the MOST accurate statement is that her increased risk of developing a leg ulcer in adolescence is associated with her A.hemolytic subphenotype B.history of acute chest syndrome C.oxygen saturation D.pulmonary hypertension

Sickle cell disease is thought to be divided into 2 subphenotypes: A hemolytic subphenotype characterized by hyperhemolysis with vascular injury caused by high levels of cell-free plasma hemoglobin A vaso-occlusive subphenotype characterized by a high level of vascular occlusion While the clinical differences between the 2 phenotypes are not commonly apparent during early childhood, they become more distinct in adolescence and adulthood as observed by the frequency of certain long-term consequences of sickle cell disease that differ between the 2 phenotypes. The patient in the vignette is not acutely ill but does have evidence of hyperhemolysis demonstrated by her hemoglobin level, elevated reticulocyte count, and elevated lactate dehydrogenase level. This hemolytic subphenotype places her at an increased risk of developing leg ulcers, stroke, proteinuria, cholelithiasis, and pulmonary hypertension in adolescence and adulthood compared to individuals with the vaso-occlusive subphenotype of sickle cell disease as well as to individuals without sickle cell disease. Priapism has also been associated with the hemolytic subphenotype. It is believed that the excess hemolysis-endothelial dysfunction from increased cell-free plasma hemoglobin and potentially decreased nitric oxide leads to a proliferative vasculopathy. In adults with sickle cell disease, an echocardiogram with an elevated tricuspid regurgitant velocity of >/= 2.5 m/s suggests mild or evolving pulmonary hypertension or right-sided heart strain (pulmonary artery systolic pressure of minimum 30 mm Hg). The predictive value of a slightly elevated tricuspid regurgitant velocity that is < 2.5 m/s at age 11 years is not known. She currently has no evidence of pulmonary hypertension. The development of a leg ulcer has not been found to be dependent on the development of pulmonary hypertension. Her resting low peripheral capillary oxygen saturation suggests that she may be developing or is at increased risk of developing pulmonary hypertension. Other possible causes of this apparent mild hypoxemia may also reflect her baseline anemia, or potential underlying pulmonary damage from previous acute chest syndrome. This slightly low oxygen saturation and history of acute chest syndrome has not been associated with an increased risk of developing leg ulcers. The vaso-occlusive subphenotype of sickle cell disease is characterized by a relatively high hemoglobin level, with increased episodes of vaso-occlusive pain crisis, acute chest syndrome, and osteonecrosis compared to the hemolytic subphenotype. The primary mechanism of excess viscosity is believed to be due to greater erythrocyte sickling compared to the hemolytic subphenotype in addition to other factors. PREP Pearls Vaso-occlusive (high viscosity) and hemolytic subphenotypes of sickle cell disease have been associated with differing incidences of complications in adolescents and adults. Leg ulcers, stroke, and pulmonary hypertension in adolescence and adulthood are more common with the hemolytic subphenotype of sickle cell disease compared to the vaso-occlusive subphenotype. The vaso-occlusive subphenotype of sickle cell disease is characterized by a relatively high hemoglobin level, with increased episodes of vaso-occlusive pain crisis, acute chest syndrome, and osteonecrosis compared to the hemolytic subphenotype.

A 15-year-old girl presents with stage IIIB nodular sclerosing Hodgkin lymphoma involving thoracic and abdominal lymph nodes. PET imaging shows no other sites of disease. After two cycles of chemotherapy, her lymph nodes have all decreased in size, with the largest nodal aggregate deceasing from 13 cm in its longest axis to 6 cm. Her mediastinal mass has reduced in diameter by half. Her tumor remains PET-avid with maximal standard uptake values in the nodal aggregate of 2.1 compared with 2.8 in the mediastinum. Which of the following most accurately describes her response to therapy? a. Complete metabolic response b. Partial response c. Stable disease d. Refractory disease

The Deauville score is a 5-point scale used to assess fluorodeoxyglucose (FDG) avidity in both Hodgkin and non-Hodgkin lymphoma. It is internationally accepted as the standard of care for evaluation of response to therapy in Hodgkin lymphoma. Deauville Score 1 = FDG-PET Result: No uptake above background Deauville Score 2 = FDG-PET Result: Uptake ≤ mediastinum Deauville Score 3 = FDG-PET Result: Uptake > mediastinum but ≤ liver Deauville Score 4 = FDG-PET Result: Uptake moderately increased compared with the liver at any site Deauville Score 5 = FDG-PET Result: Uptake markedly increased compared with the liver at any site Deauville Score X = FDG-PET Result: New areas of uptake that are considered unlikely to be related to lymphoma A Deauville score of 1 to 3 is generally accepted as metabolic complete remission. However, to prevent undertreatment, some clinical trials testing reduction of therapy consider a Deauville score of 3 as an inadequate response. A Deauville score of 1 or 2 is always considered a metabolic complete remission, and when it occurs during an interim analysis, it is usually associated with good prognosis with standard care. In patients with the nodular sclerosing subtype of Hodgkin lymphoma, it is common to have a complete metabolic response despite residual mass.

A 6-year-old boy has beta thalassemia major and receives chronic red blood cell transfusions. His parents are interested in other treatment options. Despite being intermittently compliant with iron chelation, he has evidence of mild iron overload in the liver without fibrosis or hepatomegaly. He has an HLA-identical, unaffected, 3-year-old sibling with a cryopreserved umbilical cord blood unit (total nucleated cell dose of 4.0 × 107/kg of recipient's body weight). There are also two 10/10 HLA-matched unrelated hematopoietic stem cell donors. Of the following, the BEST recommendation for treatment is A.enhance compliance with iron chelation and continue chronic transfusions B.proceed with a HLA-matched sibling umbilical cord blood transplantation C.proceed with a HLA-matched unrelated donor allogeneic hematopoietic stem cell transplant D.proceed with referral for gene therapy clinical trial

The best answer in this scenario is B., proceeding with an umbilical cord transplant with the given total nucleated cell dose per kilogram of recipient's body weight. Advances in supportive care measures for transfusion-dependent beta thalassemia major have significantly improved affected patients' health-related quality of life and survival rates. Life expectancy of patients with beta thalassemia major in developed countries now approaches the fifth decade. When counseling affected families, it is essential to weigh the risks and benefits of curative therapies for beta thalassemia disease. Allogeneic hematopoietic stem cell transplant (HSCT) is currently the only curative option for beta thalassemia major. Pre-HSCT risk stratification has been helpful in deciding whether curative HSCT is an affected individual's best option. Children have better outcomes after HSCT compared to adults, with children younger than 7 years having the best outcomes. The Pesaro group further analyzed pediatric patients (< 17 years) undergoing HSCT and determined 3 independent poor prognostic factors: hepatomegaly greater than 2 cm, portal fibrosis on liver biopsy, and history of inadequate iron-chelation therapy. All 3 risk factors are related to the degree of iron overload. Individuals identified in Pesaro class 1 have no risk factors, while individuals in class 3 have all 3 risk factors. Disease-free survival after HSCT has been reported between 85% to 90% for the Pesaro class 1 patients versus 65% to 70% for the class 3 patients. With improved HLA-typing techniques and supportive care measures, the rate of transplant-related mortality (TRM) has decreased to less than 5% in designated "lower-risk" Pesaro patients. However, HSCT-related severe sinusoidal obstructive syndrome of the liver, severe acute graft-vs-host disease (GVHD), chronic GVHD, risk of secondary malignancy, and neuroendocrine late effects remain causes of significant morbidity for patients in any class designation. Despite these risks, HSCT has been performed in more than 3,000 patients with thalassemia major, with most HSCT having been performed in Italy, with detailed long term follow-up evaluation of the rates of survival, TRM, and morbidity. Overall survival and disease-free survival rates after matched, unrelated donor HSCT are inferior to rates after HLA-matched sibling HSCT, with an increased risk of both acute and chronic GVHD in unrelated donor HSCT. Parallel to Pesaro risk stratified outcomes in HLA-matched sibling HSCT, Pesaro 'higher-risk' group patients after unrelated donor HSCT have experienced lower survival rates and increased rates of GVHD compared to Pesaro 'lower- risk' patients after unrelated donor HSCT. Most HSCTs for individuals with transfusion-dependent beta thalassemia have been performed with an HLA-matched related donor utilizing bone marrow as the stem cell source. Umbilical cord blood transplantation from an HLA-matched sibling donor has survival rates comparable to HLA-matched sibling bone marrow transplants. However, umbilical cord blood units with less than 3.5 × 107 total nucleated cell (TNC) count /kg of recipient's body weight have inferior survival rates compared to HSCT using cord blood units with higher TNC count. Peripheral blood stem cell (PBSC) collection is typically avoided because of its associated increased risk of GVHD compared to marrow or cord blood. Alternative donor HSCT (unrelated umbilical cord blood transplant, matched unrelated donor HSCT, haploidentical HSCT) can be considered if the patient lacks a suitable donor. However, with increased risk of morbidity and mortality due to chronic GVHD, the health care team and family must weigh the risks and benefits of chronic GVHD and improved life expectancy with current supportive care strategies with chronic transfusions and iron chelation. In the event one needs to consider a HLA matched unrelated donor transplant, unrelated umbilical cord transplants have had higher rates of graft failure. Therefore unrelated umbilical cord blood units are not considered a suitable graft source without more clinical investigation. Although encouraging compliance with chelation therapy for the patient in the vignette is highly recommended, this recommendation does not equate to cure. Currently, this patient's only poor prognostic risk factor, as assessed by the Pesaro risk classification, is his noncompliance with oral chelation. In this vignette, the patient's age (<7 years old) , lower-risk Pesaro pre-HSCT factor, and adequate dose of TNCs in the umbilical cord blood sample (4.5 × 107 TNC/ kg of recipient's body weight) make an HLA-matched sibling cord blood HSCT the best treatment option. Given the risks of allogeneic HSCT and low rate of identifying an HLA-matched sibling for patients with beta thalassemia, gene therapy has been recently described . Clinical trials in which the patient's own PBSCs were modified with a lentivirus construct containing a β-globin gene and infused after myeloablative chemotherapy resulted in an eliminated or reduced need for long-term transfusions. Studies of other gene therapy strategies are also underway. Studies evaluating duration of response and factors associated with improved outcomes are ongoing. WIthout long term results, this would not be the recommended best treatment option. PREP Pearls Allogeneic hematopoietic stem cell transplant with an HLA-matched related donor is currently the best curative option for beta thalassemia major. Risk factors associated with poor outcomes after hematopoietic stem cell transplant for beta thalassemia major include unrelated donor stem cells, recipient age greater than 7 years, hepatomegaly greater than 2 cm, portal fibrosis on liver biopsy, and history of inadequate iron-chelation therapy. Gene therapy trials have reduced the need for transfusions in select adult patients with beta thalassemia major, but long-term efficacy has not been established.

A 5-year-old boy is referred for evaluation of microcytic anemia. His parents report that his diet consists of meat, pasta, and some vegetables. He rarely drinks cow milk (< 2 oz/day)." He has gas and diarrhea frequently and an occasional rash. He has fallen off of his growth curve in the last 6 months. He is hemodynamically stable with a grade 2/6 systolic ejection murmur. His abdomen is distended, but nontender, with no organomegaly. Abnormal laboratory data are shown: Laboratory Test Result Hemoglobin. 8 g/dL (80 g/L) Mean corpuscular volume. 60 µm3 (60 fL) Red blood cell distribution width. 20.5% Red blood cell count. 3.3 × 106/µL (3.3 × 1012/L) Ferritin 3 ng/mL (3 µg/L) Of the following, the MOST appropriate next step in this child's treatment is A.elimination of milk from his diet B.oral iron supplementation C.packed red blood cell transfusion D.parenteral iron supplementation

The child in the vignette has iron deficiency anemia (IDA). The most common cause of IDA worldwide is nutritional deficiency; in children in industrialized countries, the intake of too much cow's milk can lead to IDA. However, this child's diet is not suggestive of this etiology. Rather, this child has evidence of celiac disease caused by gluten sensitivity, which should be considered in the diagnostic evaluation of children with IDA. As such, he may be unlikely to absorb oral iron supplementation until his intestines have recovered with elimination of gluten from his diet. Packed red blood cell transfusion would not be appropriate for this level of anemia in a hemodynamically stable child. Therefore, parenteral iron is the treatment of choice. Iron deficiency anemia affects up to 3% of children in the United States and up to 3 billion individuals worldwide. Signs and symptoms include fatigue, pallor, dizziness, exercise intolerance, and pica (the craving for nonfood items). However, patients are often asymptomatic, and IDA is identified incidentally or through screening programs. Iron deficiency has been associated with lower scores on developmental testing up to 2 decades after diagnosis. Diagnosis of IDA often does not entail extensive testing. With the appropriate history of excessive milk intake in a toddler, a complete blood count showing microcytic anemia is sufficient to justify a trial course of oral iron and elimination or drastic reduction of milk intake. If further laboratory evaluations are necessary, obtaining a ferritin level is the most reliable method of establishing iron deficiency. A C-reactive protein level can help identify an unrecognized inflammatory state that can result in a falsely normal ferritin level because ferritin is an acute phase reactant. Other iron indices can help support the diagnosis of IDA but are often not necessary to make the clinical diagnosis. The differential diagnosis of IDA includes thalassemia syndromes, other hemoglobinopathies, and anemia of inflammation. If thalassemia or another hemoglobinopathy is suspected, hemoglobin electrophoresis should be performed, or newborn screening records should be sought. It is important to replenish iron stores before obtaining hemoglobin electrophoresis because iron deficiency interferes with the production of hemoglobin A2 (Hb A2) making the results of this test difficult to interpret. Anemia of chronic inflammation is a diagnosis of exclusion in the appropriate clinical setting. Additionally, in a patient in whom a dietary history of low iron intake cannot be established, effort should be made to identify a causative etiology (Table). As illustrated for the patient in the vignette, celiac disease limits iron absorption. Other causes of poor iron absorption include autoimmune enteritis and surgical short gut. A history of blood loss leading to iron deficiency should also be sought. Microscopic gastrointestinal blood loss from parasitic infection or other causes should be considered. Treatment of IDA primarily relies on oral replacement therapy. However, for a variety of reasons, some patients cannot or will not take oral iron. For these patients, or in patients for whom oral iron is not likely to be absorbed, intravenous iron is a reasonable option. Newer formulations have resulted in less risk of anaphylaxis with infusion. Packed red blood cell transfusion should be reserved for patients with profound anemia and risk of hemodynamic instability. PREP Pearls Although nutritional deficiency is the most common cause of iron deficiency anemia worldwide, other causes include poor oral iron gastrointestinal absorption and blood loss. Most cases of iron deficiency anemia can be treated successfully with oral iron replacement, but some patients may require intravenous iron

A 5-month-old male infant is seen for recurrent pustular lesions to the scalp. He has a history of eczema, 2 episodes of otitis media, and 1 episode of pneumonia. Laboratory data are shown: Laboratory Test Result White blood cell count 10,200/µL (10.2 × 109/L) Hemoglobin 11.5 g/dL (115 g/L) Mean corpuscular volume 76 fL Platelet count 225 × 103/µL (225 × 109/L) Neutrophils 70% Lymphocytes 17% Monocytes 5% Eosinophils 8% IgG (reference, 241-613 mg/dL [2.4-6.1 g/L]) 505 mg/dL (5.05 g/L) IgM (reference, 26-60 mg/dL [0.26-0.60 g/L]) 55 mg/dL (0.55 g/L) IgA (reference, 10-46 mg/dL [0.10-0.46 g/L) 35 mg/dL (0.35 g/L) IgE (reference, 2-34 IU/mL) 200 IU/mL (200 kU/L) Of the following, the MOST likely underlying etiology of recurrent infections is A.DOCK8 deficiency B.eosinophilic granulomatosis with polyangiitis C.hyperimmunoglobulin E syndrome D.Wiskott-Aldrich syndrome

The child in the vignette with multiple episodes of bacterial infection (eg, otitis media, pneumonia) and eczema is now exhibiting pustular folliculitis, eosinophilia, and elevated levels of immunoglobulin E (IgE) consistent with hyperimmunoglobulin E syndrome (HIES). Children with DOCK8 deficiency also have recurrent respiratory tract infections (eg, otitis media, pneumonia), eosinophilia, and elevated IgE levels but have recurrent cutaneous viral infections (eg, herpes simplex viruses, molluscum), hepatic disorders (eg, sclerosing cholangitis, hepatitis), and lymphocytopenia, none of which are seen in this patient. Eosinophilic granulomatosis with polyangiitis (ie, Churg-Strauss) presents in the second and third decades of life with atopic disease, rhinitis, and asthma prior to the development of vasculitis. Patients with Wiskott-Aldrich syndrome can have eczema, elevated IgE levels, and eosinophilia, but typically have bleeding secondary to thrombocytopenia. Hyperimmunoglobulin E syndrome, formerly called "Job syndrome," is an autosomal-dominant syndrome associated with elevated levels of IgE (ie, at least 10 times normal), dermatitis, and recurrent sinopulmonary and skin infections. Affected individuals may also have coarse facial features (eg, facial asymmetry, prominent forehead, deep-set eyes, typically broad nasal base and bridge) and skeletal abnormalities (eg, delayed loss of primary teeth, scoliosis) which are notable by 16 years of age. Diagnosis in children without a family history of HIES can be challenging. Frequent pulmonary and cutaneous infections or recurrent abscesses in the setting of chronic eczema should raise suspicion of HIES. In particular, the presence of eosinophilic pustular folliculitis of the scalp plus recurrent otitis media or pneumonia in the first month after birth requires HIES evaluation. Eczema is seen in almost all patients; however, compared with atopic dermatitis, the dermatitis in HIES has a severe prolonged course and starts at an earlier age. Cold abscesses and chronic candidiasis are a common occurrence. Staphylococcus aureus is the most common bacterium isolated from skin abscesses and pneumonia. Pneumatocele formation is a common complication. Diagnosis is based on the combination of clinical and laboratory findings with confirmatory genetic defect (STAT3 mutation). This elevation of IgE can be seen even in infancy. The IgG, IgM, and IgA levels are normal. Almost all patients with HIES have elevated IgE levels, although the level is not consistent and not related to the severity of the disease. For diagnosis, IgE should be greater than or equal to 2,000 IU/mL (2,000 kU/L) or 100-200 IU/mL (100-200 kU/L) in infants. Eosinophilia (greater than 2 SDs above normal) is also commonly noted in patients and may rise before acute infections. Similar to those with other primary immunodeficiency diseases, patients with HIES are at risk of developing autoimmune diseases such as systemic lupus erythematosus and immune thrombocytopenia. PREP Pearls Hyperimmunoglobulin E syndrome, formerly called "Job syndrome," is an autosomal-dominant syndrome associated with elevated levels of immunoglobulin E (ie, at least 10 times normal), dermatitis, and recurrent sinopulmonary and skin infections. Presence of eosinophilic pustular folliculitis of the scalp plus recurrent otitis media or pneumonia in the first month after birth requires evaluation for hyperimmunoglobulin E syndrome. Diagnosis of hyperimmunoglobulin E syndrome is based on the combination of clinical and laboratory findings with confirmatory genetic defect (STAT3 mutation).

A 5-year-old boy has a history since infancy of mucocutaneous bleeding occurring spontaneously or after minor trauma. His physical examination is normal except for splenomegaly. His parents and sister are asymptomatic. Since the age of 10 months, his hemoglobin value has ranged from 9.2 to 11.2 g/dL (92 to 112 g/L) and platelet counts have ranged from 30 to 100 × 103/µL (30 to 100 × 109/L). The mean platelet volume and the reticulocyte count are elevated. The peripheral blood smear reveals normal white blood cells, microspherocytes, and dysmorphic platelets with a subpopulation of large platelets. A globin chain synthesis analysis reveals a mild unbalanced α-globin to non-α-globin chain ratio. The α-globin cluster is analyzed and reveals a normal α-globin genotype (αα/αα), indicating that the unbalanced chain synthesis ratio was not caused by a α-globin gene defect or rearrangement. Bone marrow examination reveals dyserythropoiesis. The immature megakaryocytes are normal; however, the mature megakaryocytes are small and dysmorphic with decreased azurophilic granules in the cytoplasm. Of the following, the MOST likely diagnosis is A.congenital erythropoietic porphyria B.Diamond-Blackfan anemia C.Wiskott-Aldrich syndrome D.X-linked thrombocytopenia with thalassemia

The child in this vignette has X-linked thrombocytopenia with thalassemia (XLTT), one of several disorders caused by GATA1 deficiency. The GATA family includes 6 structurally related genes, GATA1 to GATA6, which function as transcriptional regulators. GATA1, GATA2, and GATA3 each have a role in normal hematopoiesis. Mutations in these genes lead to clinically significant hematopoietic disturbances. The GATA1 erythroid transcription factor is highly expressed in red blood cells, megakaryocytic cells, eosinophils, mast cells, and basophils. In GATA1 deficiency, megakaryocytes expand dramatically but fail to undergo terminal differentiation. The opposite effect occurs in erythroid precursors that undergo apoptosis in the absence of GATA1. Deficiency of GATA1 results from mutations in GATA1, located on X p11.23, and may result from germline mutations or be acquired somatic mutations. Germline mutations in GATA1 lead to several X-linked recessive disorders with the clinical manifestations of hereditary thrombocytopenia and dyserythropoietic anemia. GATA1 R216Q and R216W mutations impair the ability of GATA1 to bind DNA resulting in X-linked thrombocytopenia with thalassemia (XLTT). Male patients with XLTT present with mild dyserythropoiesis, red cell hemolysis, maturation defects in megakaryocytes, macrothrombocytopenia with α granule deficiency and abnormalities of the cytoplasmic membrane system. Unbalanced α:β hemoglobin chain production resembling mild β-thalassemia occurs. Instead of α2:β2 hemoglobin molecules, α3:β1 and α4 molecules are formed; these molecules precipitate and deform red blood cells, which are then sequestered and destroyed in the spleen. This hemolytic anemia causes splenomegaly. The β-thalassemia minor phenotype is seen in XLTT but not in XLT. Female patients carrying the mutation may be asymptomatic. Some female carriers manifest mild to moderate symptoms and abnormal laboratory values, related to the proportion of cells containing the GATA1 mutation. GATA1 mutations have also been described in cases of congenital erythropoietic porphyria (CEP) and Diamond-Blackfan anemia (DBA). Most cases of CEP are autosomal recessive and are caused by mutations in uroporphyrinogen III synthase, an enzyme involved in heme synthesis. An X-linked recessive form of CEP is caused by a GATA1 mutation, R216W. The clinical expression of GATA1 deficiency resulting from both mutations includes photosensitivity with bullous dermatosis, anemia, splenomegaly, hirsutism, and fluorescence of body fluids. Over half of children with DBA harbor mutations in a ribosomal gene, most commonly RPS19. However, an X-linked DBA is caused by a GATA1 mutation, L74V, resulting in a splicing defect that leads to the expression of a truncated GATA1 isoform, GATA1s. Affected children have the typical hematologic abnormalities of DBA, such as normocytic anemia with reticulocytopenia, but lack the extra-hematopoietic manifestations of DBA, such as cardiac malformations, thumb anomalies, renal anomalies, midline craniofacial defects, and growth retardation. X-linked thrombocytopenia is also caused by a mutation in WAS and is called a WAS-related disorder. It can be distinguished from XLT caused by GATA1 deficiency by the presence of small platelets and the lack of dyserythropoietic anemia. The transcription factor GATA1, together with its essential transcriptional cofactor, FOG1 (friend of GATA1), regulates erythrocyte and megakaryocyte differentiation. Mutations in GATA1 or FOG1 binding sites of its N-terminal zinc finger result in different illnesses. In the GATA1 mutations V205M, G208R, D218Y, D218G, and G208S, there is a missense mutation that results in a single amino acid substitution that reduces the affinity of GATA1 for FOG1. These mutations cause a rare form of X-linked thrombocytopenia (XLT) in which there is a reduced number of platelets, large platelets that contain few α granules, and dyserythropoietic anemia. Thrombocytopenia presents in infancy, and bleeding and bruising occur spontaneously or after trauma. The degree of anemia varies depending on the mutation and may be mild with only minimal dyserythropoiesis on bone marrow examination. G218G and D208s mutations have less severe hematologic manifestations, while V205M, G208R, and D218Y have more severe clinical expression. Some affected children may be red blood cell transfusion dependent, and severe fetal hydrops may necessitate intrauterine transfusion. Children with Down syndrome often acquire mutations in GATA1. GATA1 deficiency in Down syndrome results in the development of transient abnormal myelopoiesis and acute megakaryocytic leukemia. PREP Pearls Deficiency of GATA1 results from germline or acquired somatic mutations in GATA1. GATA1 mutations R216Q and R216W are associated with X-linked thrombocytopenia with thalassemia. GATA1 mutations V205M, G208R, D218Y, D218G, and G208S are associated with a rare form of X-linked thrombocytopenia and dyserythropoietic anemia. GATA1 mutation R216W has been associated with congenital erythropoietic porphyria and GATA1 mutation L74V has been associated with rare cases of Diamond-Blackfan anemia. Acquired GATA1 mutations in Down syndrome result in the development of transient abnormal myelopoiesis and acute megakaryocytic leukemia.

A 5-year-old boy has a history since infancy of mucocutaneous bleeding occurring spontaneously or after minor trauma. His physical examination is normal except for splenomegaly. His parents and sister are asymptomatic. Since the age of 10 months, his hemoglobin value has ranged from 9.2 to 11.2 g/dL (92 to 112 g/L) and platelet counts have ranged from 30 to 100 × 103/µL (30 to 100 × 109/L). The mean platelet volume and the reticulocyte count are elevated. The peripheral blood smear reveals normal white blood cells, microspherocytes, and dysmorphic platelets with a subpopulation of large platelets. A globin chain synthesis analysis reveals a mild unbalanced α-globin to non-α-globin chain ratio. The α-globin cluster is analyzed and reveals a normal α-globin genotype (αα/αα), indicating that the unbalanced chain synthesis ratio was not caused by a α-globin gene defect or rearrangement. Bone marrow examination reveals dyserythropoiesis. The immature megakaryocytes are normal; however, the mature megakaryocytes are small and dysmorphic with decreased azurophilic granules in the cytoplasm. Of the following, the MOST likely diagnosis is A.congenital erythropoietic porphyria B.Diamond-Blackfan anemia C.Wiskott-Aldrich syndrome D.X-linked thrombocytopenia with thalassemia

The child in this vignette has X-linked thrombocytopenia with thalassemia (XLTT), one of several disorders caused by GATA1 deficiency. The GATA family includes 6 structurally related genes, GATA1 to GATA6, which function as transcriptional regulators. GATA1, GATA2, and GATA3 each have a role in normal hematopoiesis. Mutations in these genes lead to clinically significant hematopoietic disturbances. The GATA1 erythroid transcription factor is highly expressed in red blood cells, megakaryocytic cells, eosinophils, mast cells, and basophils. In GATA1 deficiency, megakaryocytes expand dramatically but fail to undergo terminal differentiation. The opposite effect occurs in erythroid precursors that undergo apoptosis in the absence of GATA1. Deficiency of GATA1 results from mutations in GATA1, located on X p11.23, and may result from germline mutations or be acquired somatic mutations. Germline mutations in GATA1 lead to several X-linked recessive disorders with the clinical manifestations of hereditary thrombocytopenia and dyserythropoietic anemia. GATA1 R216Q and R216W mutations impair the ability of GATA1 to bind DNA resulting in X-linked thrombocytopenia with thalassemia (XLTT). Male patients with XLTT present with mild dyserythropoiesis, red cell hemolysis, maturation defects in megakaryocytes, macrothrombocytopenia with α granule deficiency and abnormalities of the cytoplasmic membrane system. Unbalanced α:β hemoglobin chain production resembling mild β-thalassemia occurs. Instead of α2:β2 hemoglobin molecules, α3:β1 and α4 molecules are formed; these molecules precipitate and deform red blood cells, which are then sequestered and destroyed in the spleen. This hemolytic anemia causes splenomegaly. The β-thalassemia minor phenotype is seen in XLTT but not in XLT. Female patients carrying the mutation may be asymptomatic. Some female carriers manifest mild to moderate symptoms and abnormal laboratory values, related to the proportion of cells containing the GATA1 mutation. GATA1 mutations have also been described in cases of congenital erythropoietic porphyria (CEP) and Diamond-Blackfan anemia (DBA). Most cases of CEP are autosomal recessive and are caused by mutations in uroporphyrinogen III synthase, an enzyme involved in heme synthesis. An X-linked recessive form of CEP is caused by a GATA1 mutation, R216W. The clinical expression of GATA1 deficiency resulting from both mutations includes photosensitivity with bullous dermatosis, anemia, splenomegaly, hirsutism, and fluorescence of body fluids. Over half of children with DBA harbor mutations in a ribosomal gene, most commonly RPS19. However, an X-linked DBA is caused by a GATA1 mutation, L74V, resulting in a splicing defect that leads to the expression of a truncated GATA1 isoform, GATA1s. Affected children have the typical hematologic abnormalities of DBA, such as normocytic anemia with reticulocytopenia, but lack the extra-hematopoietic manifestations of DBA, such as cardiac malformations, thumb anomalies, renal anomalies, midline craniofacial defects, and growth retardation. X-linked thrombocytopenia is also caused by a mutation in WAS and is called a WAS-related disorder. It can be distinguished from XLT caused by GATA1 deficiency by the presence of small platelets and the lack of dyserythropoietic anemia. The transcription factor GATA1, together with its essential transcriptional cofactor, FOG1 (friend of GATA1), regulates erythrocyte and megakaryocyte differentiation. Mutations in GATA1 or FOG1 binding sites of its N-terminal zinc finger result in different illnesses. In the GATA1 mutations V205M, G208R, D218Y, D218G, and G208S, there is a missense mutation that results in a single amino acid substitution that reduces the affinity of GATA1 for FOG1. These mutations cause a rare form of X-linked thrombocytopenia (XLT) in which there is a reduced number of platelets, large platelets that contain few α granules, and dyserythropoietic anemia. Thrombocytopenia presents in infancy, and bleeding and bruising occur spontaneously or after trauma. The degree of anemia varies depending on the mutation and may be mild with only minimal dyserythropoiesis on bone marrow examination. G218G and D208s mutations have less severe hematologic manifestations, while V205M, G208R, and D218Y have more severe clinical expression. Some affected children may be red blood cell transfusion dependent, and severe fetal hydrops may necessitate intrauterine transfusion. Children with Down syndrome often acquire mutations in GATA1. GATA1 deficiency in Down syndrome results in the development of transient abnormal myelopoiesis and acute megakaryocytic leukemia. PREP Pearls Deficiency of GATA1 results from germline or acquired somatic mutations in GATA1. GATA1 mutations R216Q and R216W are associated with X-linked thrombocytopenia with thalassemia. GATA1 mutations V205M, G208R, D218Y, D218G, and G208S are associated with a rare form of X-linked thrombocytopenia and dyserythropoietic anemia. GATA1 mutation R216W has been associated with congenital erythropoietic porphyria and GATA1 mutation L74V has been associated with rare cases of Diamond-Blackfan anemia. Acquired GATA1 mutations in Down syndrome result in the development of transient abnormal myelopoiesis and acute megakaryocytic leukemia. ABP Content Specifications(s)/Content Area GATA 1 deficiency: recognition and risks

A 15-month-old boy who recently emigrated from Palermo, Italy, is found to have anemia. He is generally well, is slightly pale, and has no scleral icterus or hepatosplenomegaly. His laboratory data are as follows: Laboratory Test Result White blood cell count 16,300/µL (16.3 × 109/L) Red blood cell count 3.9 × 106/µL (3.9 × 1012/L) Hemoglobin 9.3 g/dL (93 g/L) Mean corpuscular volume 62.8 fL Red blood cell distribution width 29.9% Platelet count 407 × 103/µL (407 × 109/L) Peripheral smear Target cells present Nucleated red blood cells 7% Hemoglobin A 76.7% Hemoglobin A2 3.3% Hemoglobin F 20.0% Red cell distribution, Hemoglobin F heterocellular Of the following, the MOST likely diagnosis is A.heterozygous hereditary persistence of fetal hemoglobin B.heterozygous delta beta-thalassemia C.homozygous beta zero-thalassemia D.homozygous hereditary persistence of fetal hemoglobin

The child in this vignette has low levels of adult hemoglobin (Hb A), elevated fetal hemoglobin (Hb F), normal levels of Hb A2, and a heterocellular red cell distribution pattern of Hb F which support a diagnosis of either heterozygous (delta beta-) ẟβ-thalassemia or heterozygous hereditary persistence of fetal hemoglobin (HPFH). Individuals with heterozygous ẟβ-thalassemia have mild to moderate microcytic anemia, many target cells, elevated Hb F (5-20%), as in this vignette, in addition to evidence of hemolysis such as reticulocytosis, low haptoglobin, and unconjugated hyperbilirubinemia. Individuals who have HPFH alone, whether heterozygous or homozygous, are normocytic without anemia and rarely are target cells present. While individuals with heterozygous HPFH produce moderately elevated levels of Hb F (15-35%) with a heterocellular red cell distribution pattern of Hb F, individuals with homozygous HPFH produce a homocellular or pancellular red cell distribution pattern of Hb F, elevated levels of Hb F sometimes greater than 90%, and no levels of Hb A or Hb A2. The laboratory results in this vignette are not consistent with (beta zero-) β0-thalassemia which typically manifests with transfusion-dependence by 6 months of age, after protective Hb F levels naturally decrease and a more severe microcytic anemia, with lower levels of Hb A and elevated levels of Hb A2. The production of Hb F and β-globin is regulated through promoter genes as well as a protein complex with GATA-1, FOG1, and BCL11A. The BCL11A protein binds upstream from the ẟ-globin gene (Hb A2), and it inhibits Hb F production and promotes Hb A production. ẟβ-Thalassemia occurs when mutations delete the ẟ-globin and β-globin genes through a variety of genetic injuries commonly affecting the regulatory sites, although the globin genes involved may also be damaged. The most common subtype of ẟβ-thalassemia is the Sicilian form, in which a large deletion of the ẟ- and β-globin genes results in microcytic anemia with increased Hb F with a heterocellular red cell distribution pattern. The Sicilian type of ẟβ-thalassemia is related to a large deletion (13,377 kb) affecting the β-globin gene and areas just outside this gene. It affects the binding site for the BCL11A complex, which decreases its ability to regulate Hb F production. When levels of Hb F are greater than 15-35%, to help differentiate amongst various types of large or non-deletional HPFH and ẟβ-thalassemia or other β-thalassemia, whole blood can be tested by flow cytometry of RBC content with fluorescein anti-Hb F. Heterozygous HPFH, which is associated with normal RBC indices and moderate levels of Hb F (< 25%), reveals a heterocellular pattern with peaks at both Hb A and Hb F. Homozygous HPFH usually exhibit higher levels of Hb F (> 70%) associated with a homocellular (or pancellular) pattern with a single peak in the area between where the normal Hb A and Hb F peaks reside. Rare homozygous ẟβ-thalassemia can have levels of Hb F of 100% without any Hb A or Hb A2 detected so flow cytometry demonstrating a heterocellular pattern, and molecular testing are helpful to differentiate this from homozygous HPFH. PREP Pearls Hereditary persistence of fetal hemoglobin, whether the heterozygous or homozygous form, is frequently associated with absence of δ globin production with no anemia, normal red blood cell indices, and rarely target cells. Heterozygous ẟβ-thalassemia is typically associated with a mild to moderate microcytic anemia, reticulocytosis, low haptoglobin, unconjugated hyperbilirubinemia, target cells, an elevated Hb F (5-20%), no elevation in Hb A2 , and a heterocellular red cell distribution pattern of Hb F. Homozygous ẟβ-thalassemia presents with the same clinical picture as the heterozygous form except with levels of Hb F of up to 100% without any Hb A or Hb A2.

A 13-year-old boy with severe aplastic anemia has not responded to 6 months of immunosuppressive therapy. He has no siblings or unrelated human leukocyte antigen- (HLA-) matched donors for allogeneic hematopoietic stem cell transplant (HSCT). His mother is the HLA-matched haploidentical stem cell donor. His conditioning regimen for HSCT consists of fludarabine, cyclophosphamide, and antithymocyte globulin. Graft-versus-host disease prophylaxis consists of post-transplantation cyclophosphamide, tacrolimus, and mycophenolate mofetil. On day +20 after stem cell infusion, he develops dysuria and frank hematuria. Of the following, the medication MOST likely associated with his current condition is A.cyclophosphamide B.fludarabine C.mycophenolate mofetil D.tacrolimus

The child in this vignette has signs and symptoms most consistent with post-transplant hemorrhagic cystitis due to cyclophosphamide. Neither fludarabine, mycophenolate mofetil, nor tacrolimus has been associated with hemorrhagic cystitis or hematuria. Cyclophosphamide is an oxazaphosphorine medication, which is a derivative of nitrogen mustard and classified as an alkylating agent. It is a prodrug that is metabolized by cytochrome p450 enzymes within the liver to the active compound, 4-hydroxycyclophosphamide, and the isomer, aldophosphamide. Aldophosphamide is broken down in cells into phosphoramide mustard and acrolein. Phosphoramide mustard causes irreversible DNA crosslinking between and within DNA strands resulting in apoptosis. Cyclophosphamide metabolites are excreted into the urine. Acrolein irritates the uroepithelium of the bladder leading to hemorrhagic cystitis. Enhancing bladder voiding with aggressive intravenous hydration and concurrently administering mesna, which binds to acrolein, significantly decrease the risk of hemorrhagic cystitis. Acrolein is also known to have a toxic effect on myocardial and endothelial cells typically associated with cyclophosphamide doses ranging between 170 to180 mg/kg of body weight or at lower doses in patients with other underlying cardiac problems. Since hematopoietic stem cell transplantation (HSCT) usually involves a maximum single cyclophosphamide dose of 50 mg/kg of body weight, cardiac toxicities are less likely to occur unless the HSCT recipient has baseline cardiac issues. Dose modifications of cyclophosphamide are recommended in the setting of renal and/or hepatic dysfunction. Concurrently administered medications that affect the cytochrome p450 enzymatic pathway may decrease the activity or increase toxicity of cyclophosphamide. Aldehyde dehydrogenase exists naturally in high concentrations in bone marrow stem cells, liver cells, and intestinal epithelium. Aldehyde dehydrogenase inactivates aldophosphamide (a metabolite of cyclophosphamide) by converting aldophosphamide into carboxycyclophosphamide before aldophosphamide can be broken down into active metabolites. As a result of aldehyde dehydrogenase's enzymatic activity, cyclophosphamide has less toxicity than other alkylating agents rendering cyclophosphamide more myelosuppressive than myeloablative. Cyclophosphamide given after HSCT known as posttransplantion cyclophosphamide (PTCy) typically does not cause damage to the new stem cell graft in haploidentical HSCT recipients. HIstorically, mismatched human leukocyte antigen (HLA) donor HSCT had been associated with increased risk of refractory graft-versus-host disease (GVHD) and rejection. Recent clinical trials using PTCy have reduced rates of GVHD and rejection in mismatched-HLA HSCT recipients comparable to recipients of HLA-matched unrelated donor HSCT. As a result of PTCy benefits, haploidentical donors have become a viable option for HSCT thereby increasing the HSCT donor pool. In addition to anti-tumor activity, cyclophosphamide has significant immunosuppressive activity. Cyclophosphamide has been used extensively in treating autoimmune diseases such as systemic lupus erythematosus and rheumatoid arthritis, perhaps due to elimination of reactive T-cells and induction of suppressor T-cells. In HSCT, immunosuppression secondary to cyclophosphamide may be due to prevention of the proliferation of CD4+ effector T-cells, inducing impairment of CD4+ and CD8+ T-cells, and faster recovery of CD4+ regulatory T-cells. PREP Pearls Posttransplant cyclophosphamide use has decreased the incidence of severe graft-versus-host disease in haploidentical hematopoietic stem cell transplantation, thereby increasing the donor pool for patients who need transplantation. Cyclophosphamide is a prodrug that is converted to aldophosphamide, that in turn is metabolized to phosphoramide mustard, the active metabolite causing irreversible DNA crosslinking and tumor cell apoptosis. Aldehyde dehydrogenase, which is present in hematopoietic stem cells, liver cells, and intestinal epithelium, prevents the metabolism of cyclophosphamide into aldophosphamide and acrolein, thereby rendering cyclophosphamide more myelosuppressive and less myeloablative than other alkylating agents. Acrolein, a byproduct of the conversion of aldophosphamide into phosphoramide mustard (the active metabolite of cyclophosphamide), is the underlying cause of cyclophosphamide- associated hemorrhagic cystitis and cardiac toxicity.

A 1-year-old child has a persistent erythematous, scaling, papular, purpuric diaper rash with erosive, crusty lesions along his inguinal skin folds (Figure 1). He is otherwise well, with no systemic symptoms. A complete blood cell count, liver function test results, prothrombin time, and activated partial thromboplastin time are normal. A skin biopsy is planned. Of the following, the cellular marker of the skin sample that will BEST identify the likely diagnosis is A.CD1a B.CD16 C.CD22 D.CD61

The child in this vignette presents with a rash consistent with Langerhans cell histiocytosis (LCH). Langerhans cells (LCs), which are the pathologic cells associated with LCH, are a subtype of dendritic cells (DCs) and the primary antigen-presenting cells of skin. Surface marker CD1a is the most characteristic of LCs; CD16 is a marker for monocytes, CD22 is a B-cell marker, and CD61 is a platelet marker. Dendritic cells are antigen-presenting cells that can activate naive T lymphocytes and can render T cells tolerant to specific antigens (Figure 2). Different types of DCs include conventional DCs, which are present in lymphoid tissues at all times, and inflammatory DCs, which arise in response to specific signals. Other functions of DCs include secretion of cytokines and immune mediators such as tumor necrosis factor. Additionally, some DCs reside in the lymphoid tissue, while others are migratory, going between lymphoid tissue and organs that interface with the external environment, such as the skin and mucous membranes. Dendritic cells are related to monocytes and macrophages as part of the mononuclear phagocyte system and are often difficult to distinguish histopathologically. Various nomenclature systems have been proposed to clarify discussion. Most rely on both the ontogeny and function of the cells to simplify classification. One complicating factor is that DCs may take on different functions over their lifespan. Specific markers can help distinguish LCs from other types of DCs and can help discriminate between the different types of histiocytic disorders. Birbeck granules, which are tennis racket-shaped organelles present in LCs, are associated with the internalization of the surface marker CD207, also known as langerin. Some normal DCs also express CD11c. Pathologic LCs will express both of these markers, but will also express S-100. Some data suggest that pathologic LCs in LCH may not derive from normal LCs, but may be more closely related to myeloid dendritic precursors. PREP Pearls Dendritic cells are antigen-presenting cells that can activate naive T lymphocytes and can render T cells tolerant to specific antigens. Langerhans cells, which are the pathologic cells associated with Langerhans cell histiocytosis, are a subtype of dendritic cells and the primary antigen-presenting cells of skin. Specific markers such as CD1a, Birbeck granules (CD207), and S-100 staining can help distinguish Langerhans cells from other types of dendritic cells and can help discriminate between the different histiocytic conditions.

A 12-year-old boy with sickle cell anemia has chest pain and hypoxemia. He is diagnosed with acute chest syndrome after a chest x-ray, complete blood count, reticulocyte count, and blood culture are obtained. The decision is made to start intravenous antibiotics and transfuse red blood cells. He has a history of stroke and receives monthly exchange blood transfusions. It has been 3 weeks since his last exchange transfusion. Shortly after beginning the transfusion, he develops a fever of 39°C. His vital signs are stable. The transfusion is stopped. A blood specimen for a transfusion reaction evaluation and a blood culture are obtained. His urine specimen appears darker than usual and he develops back pain. A urine analysis is obtained. Of the following, the next BEST step in management is to: A.change intravenous antibiotics B.initiate dialysis C.provide an intravenous normal saline bolus D.provide intravenous opioids

The classic signs and symptoms of acute hemolytic transfusion reaction (AHTR) are fever with or without chills, abdominal, back, and/or flank pain (due to renal capsular distention), and red/brown urine (due to hemoglobinuria) occurring during or within hours of completion of red blood cell transfusion. Some patients may initially present with fever alone. Acute hemolytic transfusion reaction is commonly caused by recipient antibodies to donor minor blood group antigens in patients who have previously received frequent transfusions as in this vignette. Other antibody mediated causes of AHTR which are more rare include blood group antigen incompatibility due to a systems error in blood typing, or incompatible plasma products such as intravenous immune globulin, apheresis platelets, or fresh frozen plasma, which react with the recipient's red blood cells. The patient in this vignette is showing signs and symptoms of AHTR (ie, fever, dark urine, back pain) which if left untreated can lead to life-threatening disseminated intravascular coagulation, renal failure, and shock. To reduce morbidity and mortality risks, immediate supportive care is initiated prior to confirmatory results from the tested blood samples submitted to the blood bank. Initial supportive measures after stopping the transfusion, include a rapid intravenous normal saline bolus to support blood pressure and encourage urine output, even prior to signs of hypotension. Additional interventions that may be necessary include maintenance intravenous fluids, diuretics, vasopressors, and other blood products if disseminated intravascular coagulation develops. Patients with sickle cell disease are prone to infections with encapsulated bacteria such as Streptococcus sp, Neisseria meningitidis, and Escherichia coli, due to autoinfarction of the spleen. Therefore, when fever develops, blood cultures are obtained and broad spectrum intravenous antibiotics are given. The child in this vignette is already receiving broad spectrum antibiotics for presumptive treatment of an infiltrate on chest x-ray consistent with acute chest syndrome. Until an infectious organism is identified, antibiotics do not need to be changed. In addition, the dark urine and back pain seem more consistent with AHTR than with bacteremia from the transfused blood product. Although dialysis may be needed if this patient develops renal failure or electrolyte imbalances, dialysis would not be the initial step in management of an individual experiencing AHTR. Patients with sickle cell disease frequently present with back pain due to vaso-occlusive crisis. However, the back pain in this vignette is accompanied by fever and dark urine, in the setting of the transfusion, these seem more likely due to AHTR than isolated vaso-occlusive crises. Typical pain management starts with topical warm packs, oral acetaminophen and nonsteroidal anti-inflammatory drugs and can include oral or intravenous opioids if symptoms worsen. Though pain management will need to be addressed, initial management in AHTR involves cardiovascular stabilization with an intravenous normal saline bolus. PREP Pearls Antibody-mediated acute hemolytic transfusion reaction can result from the development of recipient antibodies due to blood group incompatibility, minor red blood cell antigen incompatibility in a patient previously transfused, or due to plasma transfusions containing antibodies against the recipient's red blood cell antigens. The classic signs and symptoms of acute hemolytic transfusion reaction are fever with or without chills, abdominal, back and/or flank pain (due to renal capsular distention), and red/brown urine (due to hemoglobinuria) occurring during or within hours of completion of red blood cell transfusion. Since acute hemolytic transfusion reaction can result in life threatening hypotension, disseminated intravascular coagulation, and acute kidney injury, immediate management prior to laboratory confirmation of hemolysis includes stopping the transfusion and initiating a rapid infusion bolus of intravenous normal saline.

A previously healthy 7-year-old boy has a two-day history of fever, emesis, abdominal distension, and pain. He has tachycardia, tachypnea, absent bowel sounds, and a diffusely firm abdomen with guarding and rebound tenderness. Laboratory test results reveal elevated lactate dehydrogenase and uric acid levels. A computed tomographic scan of the neck, chest, abdomen, and pelvis is obtained immediately. It reveals a 15 × 15-cm solid hypoechoic abdominal mass arising from the ileocecal region with asymmetric bowel wall thickening, dilatation of the bowel lumen, and multiple 4-cm mesenteric lymph nodes. Biopsy findings show a diffuse proliferation of moderate-sized lymphocytes expressing CD20 and CD10 with irregular nuclear borders, clumped chromatin, and numerous mitoses. Of the following, the MOST useful role of positron emission tomography/computed tomography in this patient is to assess for A.bone involvement B.brain involvement C.initial tumor staging D.tumor response after therapy

The clinical presentation of the patient in the vignette is suggestive of an aggressive non-Hodgkin lymphoma (NHL), such as Burkitt lymphoma. The role of positron emission tomography/computed tomography (PET/CT) in the initial staging of pediatric aggressive NHL is under-studied, possibly because of the futility of initial PET/CT scanning in patients who are acutely ill at presentation, as in this vignette. The uptake of the fluorine-18-fluorodeoxyglucose (FDG) in PET/CT correlates to tumor proliferation, as well as to inflammation resulting from injury or infection. Therefore, PET/CT would not be helpful in assessing this patient's initial extent of tumor activity owing to the overlapping inflammation caused by intestinal obstruction and recent biopsy. Because the patient is acutely ill with bulky, rapidly proliferating tumor, it is urgent to start supportive care and cancer therapy promptly without delaying to obtain PET/CT. Burkitt lymphoma tumor staging can be completed with bilateral bone marrow aspiration and biopsies and diagnostic lumbar puncture and it is not dependent on PET/CT. Depending on the clinical presentation of Burkitt lymphoma, additional imaging of other potential tumor sites (head, maxillofacial, orbit, spine, or bone) may be necessary but is not consistent with this vignette. Response on PET is measured via the Deauville score. This is a five-point visual scoring system for FDG uptake, based on comparison of uptake in liver and mediastinum with that in other tissues. A Deauville score of 1 to 3 is considered negative and a score of 4 to 5 is positive. In this vignette, PET/CT would be useful if the patient fails to achieve complete response to chemotherapy, as demonstrated on conventional CT or magnetic resonance imaging, to determine whether any residual mass represents active tumor or areas of fibrosis. There is evidence in pediatric NHL that FDG-PET/CT positive findings are sensitive and correlate with active tumor in a biopsied residual mass. However, the decision to change treatment is not based solely on positive PET/CT results. Histological confirmation is strongly suggested with removal or large biopsy of residual tumor as clinically indicated. A negative FDG-PET/CT finding is supportive of complete response to therapy and usually eliminates the need for biopsy of suspected residual lymphoma. In pediatric patients with Hodgkin lymphoma, the combined functional and anatomical imaging modality of PET/CT is standard of care for initial staging and for monitoring of response to therapy. Early response to therapy is used to tailor further therapy, particularly radiotherapy. Early response to therapy is defined as rapid early resolution of previously FDG-avid tumor sites after two cycles of chemotherapy, even in the presence of residual disease as seen in conventional imaging/CT. Early response is also correlated to improved event-free survival. Positron emission tomography/computed tomography is not typically used in assessing brain involvement in lymphoma. Bone involvement in pediatric lymphoma is rare and typically manifests as bone pain and or a soft-tissue mass. If bone involvement is suspected, FDG PET scan, if available, is the preferred imaging modality (rather than technetium-99 bone scan) owing to its higher sensitivity and specificity. PREP Pearls Positron emission tomography/computed tomography using fluorine-18-fluorodeoxyglucose (FDG) is helpful in assessing nodal and extranodal extent in Hodgkin and non-Hodgkin lymphoma for initial staging and to assess treatment response. Limitations of FDG positron emission tomography/computed tomography in lymphoma activity include false-positive results after surgical inflammation or infection or with normal physiologic metabolic activity, as seen in bone epiphyseal plate and brown fat.

A screening complete blood count from a healthy 12-month-old girl reveals a platelet count of 28 × 103/µL (28 × 109/L) with normal mean platelet volume, white blood cell count, and hemoglobin level. A repeat complete blood count the following day shows a platelet count of 26 × 103/µL (26 × 109/L) with the peripheral smear revealing an isolated clump of average- sized platelets. The patient has no bleeding symptoms and no family history of excessive bleeding. Her physical examination is normal. Of the following, the MOST likely cause of thrombocytopenia is A.decreased production of WAS protein B.loss of function of c-MPL C.mutation in MYH9 D.platelet agglutination due to EDTA

The healthy child in this vignette, who has no personal or family bleeding history, a normal physical exam, and has evidence of platelet clumping on the peripheral blood smear, most likely has spurious thrombocytopenia or pseudothrombocytopenia (PTCP). Pseudothrombocytopenia is a rare occurrence due to an in vitro problem most often resulting from platelet agglutination arising in blood specimen collection tubes containing ethylenediaminetetraacetic acid (EDTA). This in vitro agglutination results in the automated counter (hematology analyzer) reporting a falsely low measurement of the platelet count, while the mean platelet volume may be normal or unable to be determined. A proposed mechanism of EDTA-associated PTCP is that the platelet surface glycoproteins are altered when incubated with EDTA, stimulating platelet autoantibodies and agglutination. Another potential mechanism of EDTA- associated PTCP called platelet satellitism, results in platelet-neutrophil agglutination as evidenced by numerous platelets surrounding the granulocytes on the peripheral blood smear. Although EDTA is widely used in blood specimen collection tubes to stabilize blood cells for enumeration in automated counters, the prevalence of EDTA-associated PTCP is estimated to be 0.07% to 0.20%. Platelet agglutination occurs less frequently in blood sample collection tubes containing sodium citrate, sodium heparin, or both. Other causes of PTCP include phlebotomy issues with difficult venipuncture or overfilling of the blood collection tube, in vitro platelet cold agglutination of uncertain etiology, and inherited or transient large platelets which may be incorrectly enumerated as leukocytes in automated counters. Given that some automated counters that use electrical impedance rely on cell size to distinguish cell type (eg, erythrocyte fragments may erroneously be counted as platelets), it is important to review the peripheral smear when assessing platelet counts. Mutations in the WAS gene (Xp11.23) can lead to decreased production of the WAS protein, as seen in X-linked thrombocytopenia and Wiskott-Aldrich syndrome, which are characterized by immunodeficiency, eczema, and microthrombocytopenia. Congenital amegakaryocytic thrombocytopenia is caused by mutations in the MPL gene (1p34), resulting in complete loss of function in the thrombopoietin receptor c-MPL, inadequate platelet production, and bleeding symptoms. Mutations in MYH9 (22q12.3) can lead to congenital macrothrombocytopenia disorders with development of a variety of nonhematologic clinical features, including renal insufficiency, high-frequency hearing loss, and cataracts. PREP Pearls Thrombocytopenia as reported by automated counters (hematology analyzers) for individuals with no personal or family bleeding history, a normal physical exam, and with evidence of platelet clumping on the peripheral blood smear, is consistent with spurious thrombocytopenia or pseudothrombocytopenia. Pseudothrombocytopenia is a rare occurrence due to an in vitro problem most often resulting from platelet agglutination arising in blood specimen collection tubes containing ethylenediaminetetraacetic acid (EDTA). Other causes of pseudothrombocytopenia include phlebotomy issues with difficult venipuncture or overfilling of the blood collection tube, in vitro platelet cold agglutination of uncertain etiology, and inherited or transient large platelets which may be incorrectly enumerated as leukocytes in automated counters.

A 7-week-old girl was born with a slightly raised mass with a purpuric, bruised appearance and telangiectasias on the left shoulder that have gradually spread to involve the upper arm and chest. During the past few days, the mass has rapidly become larger, firm, and painful. Blood work reveals severe thrombocytopenia, anemia, and hypofibrinogenemia. The infant is not receiving any medications. Of the following, the MOST likely diagnosis is A.ataxia telangiectasia B.infantile fibrosarcoma C.Kasabach-Merritt syndrome D.Peutz-Jeghers syndrome

The infant described in the vignette has a vascular tumor and has developed Kasabach-Merritt syndrome. Kasabach-Merritt syndrome, also called Kasabach-Merritt phenomenon (KMP), was first described in 1940. The syndrome is defined as a vascular tumor with consumptive thrombocytopenia. Kasabach-Merritt syndrome is a complication of the aggressive vascular tumors kaposiform hemangioendothelioma (KHE) and tufted angioma (TA). These proliferating vascular lesions arise from both lymphatic and capillary endothelia. They rapidly enlarge and are often firm, painful, bruised, and purpuric. It is thought that platelets are activated after becoming trapped in the abnormal tortuous vasculature. Platelet activation leads to thrombocytopenia, consumptive coagulopathy, and microangiopathic hemolytic anemia. The mean age at KMP diagnosis is 2 months. Approximately 70% of infants with KHE and up to 10% of infants with TA will develop KMP. The risk of developing KMP is greater when vascular lesions are located in the thoracic cavity or retroperitoneum. Vascular lesions in deep visceral locations are not easily identified on physical examination. Magnetic resonance imaging is indicated if a visceral lesion is suspected. Biopsy of a vascular lesion may be helpful in establishing the correct histologic diagnosis; however, if KMP has developed, biopsy is not recommended because of the risk of bleeding. The KMP is rare but may be life-threatening. Mortality is high at 30% and death usually occurs due to local invasion of the aggressive vascular tumor, hemorrhage, and cardiac failure. Laboratory evaluation of complex vascular anomalies such as KHE and TA should include a complete blood count, fibrinogen level, prothrombin time, activated partial thromboplastin time, and D-dimer. Treatment options for KHE and TA differ depending on whether or not KMP has developed. There are no randomized trials or large observational studies that validate expert opinion-based treatment recommendations. In cases of KHE or TA without KMP, observation is an option, because spontaneous remission has been reported. Surgery is the definitive treatment; however, complete resection is difficult, because these tumors may infiltrate other tissues and have ill-defined borders. Pharmacologic therapy with aspirin 5 mg/kg per day is recommended for TA and with oral prednisone 2 mg/kg per day for KHE. Aspirin, 2 to 5 mg/kg per day, may also be added along with steroids for KHE. Second-line therapy with other agents have also been used and include propranolol, sirolimus, vincristine, interferon-α, and ticlopidine. When KMP develops, supportive therapy to prevent life-threatening complications is necessary, along with treatment of the underlying vascular tumor. Thrombocytopenia is often severe, but life-threatening hemorrhage is rare. Platelet transfusion should be reserved for active bleeding, because transfused platelets may become trapped in the abnormal vasculature and thus cause enlargement of the tumor and worsening consumptive coagulopathy. Packed red blood cells should be transfused for symptomatic anemia, and cryoprecipitate can be used to correct hypofibrinogenemia. Fresh frozen plasma may be indicated to replace other clotting factors. Recombinant human factor VIIa can be given for severe bleeding. Surgery is not advised when KMP has developed owing to the risk of bleeding complications. Vascular embolization is difficult to accomplish. Radiation has been used in the past but carries a significant risk of secondary late effects in infants and young children. Pharmacologic treatment may be indicated to improve the consumptive coagulopathy and decrease symptoms caused by tumor mass. Agents used include high-dose systemic corticosteroids, vincristine, sirolimus, and interferon-α. Propranolol has been suggested as a possible treatment owing to its efficacy in treating infantile hemangiomas; however, in a small case series, it showed only limited effectiveness in treating KHE and TA. Most experts recommend vincristine and steroids or sirolimus with or without steroids as first-line treatment for KMP. Investigators are conducting clinical trials to compare the effectiveness of vincristine with sirolimus for the treatment of KHE/KMP. The other diagnoses are not compatible with the findings described in this infant. Children with ataxia-telangiectasia have oculocutaneous telangiectasias, immune deficiency, and progressive cerebellar degeneration with ataxia, abnormal eye movements, and other neurologic problems. Infantile fibrosarcoma is a poorly circumscribed spindle cell tumor that may be congenital or rapidly develop during the first year after birth. The tumor develops in soft tissues, and its most common location is the distal extremities. Skin over a large tumor may become erythematous and ulcerative but does not usually appear purpuric or bruised. This tumor does not cause thrombocytopenia or coagulopathy. Children with Peutz-Jeghers syndrome are at increased risk for developing cancer. Dark-colored mucocutaneous macules and multiple hamartomatous polyps in the gastrointestinal tract are consistent with this syndrome. PREP Pearls Kasabach-Merritt syndrome is a complication of the aggressive vascular tumors, kaposiform hemangioendothelioma and tufted angioma, that leads to consumptive thrombocytopenia. Supportive therapy to prevent life-threatening complications may be necessary, along with treatment of the underlying vascular tumor.

A 6-month-old female infant has bruises and a 3-day history of pallor. She is small for her age with microcephaly, widely spaced eyes, low-set, prominent ears, small fingers, and a mass in the right side of her abdomen. She has normal vital signs. Laboratory data are shown: Laboratory Test. Result White blood cell coun. 2,000/µL (2.0 × 109/L) Neutrophils. 70% Bands. 3% Lymphocytes. 27% Hemoglobin. 7.9 g/dL (79 g/L) Platelet count. 68 × 103/µL (68 × 109/L) Her gravida 1 para 1 mother has a history of epilepsy and during pregnancy received cetirizine, levothyroxine, phenytoin, prenatal vitamin, and acetaminophen. Of the following, the tumor that is MOST likely associated with this infant's drug exposure history is A.adrenal adenoma B.hepatoblastoma C.neuroblastoma D.Wilms tumor

The infant in the vignette has physical stigmata consistent with fetal hydantoin syndrome, a genotoxicity syndrome associated with prenatal exposure to phenytoin. The facial features, ears, microcephaly, and short stature are classical features of this embryopathy, which manifests with additional variable features. Phenytoin usage during pregnancy has been associated with an increased risk of leukemia, lymphoma, and neuroblastoma in the offspring with fetal hydantoin syndrome but not in offspring without fetal hydantoin syndrome. Phenytoin intake by older children and adults with epilepsy is associated with a less clear risk of oncogenesis. Although Wilms tumor has been linked in case reports to prenatal phenytoin exposure, the presence of bruising, pallor, and pancytopenia are not consistent with Wilms. Children with Beckwith-Wiedemann syndrome are at increased risk of Wilms tumor and hepatoblastoma. Beckwith-Wiedemann syndrome is associated with mutation of WT1, manifesting with hemihypertrophy not dysmorphic facial features and microcephaly. Genotoxicity syndromes occur when protective enzymes are not present because of a mutation in the mother or fetus that leaves an active gene exposed to damage by a chemical encountered by the fetus that would have otherwise been cleared by the mother. Fetal alcohol syndrome and certain subgroups of infant acute myeloid leukemia are other examples of genotoxicity syndromes. To date, adrenal adenomas and hepatoblastomas are not associated with a genotoxicity syndrome. Medications that are associated with the development of malignancies in adults and children are listed in the 14th edition of the Report of Carcinogens from the US Department of Health and Human Services (https://ntp.niehs.nih.gov/pubhealth/roc/index-1.html). PREP Pearls Fetal exposure to phenytoin during pregnancy has been associated with the fetal hydantoin syndrome and an increased risk of leukemia, lymphoma, and neuroblastoma in the exposed infant. There is controversy over the oncogenic potential of phenytoin outside of fetal exposure.

The state newborn screen results show hemoglobin pattern of F,A,Barts. The infant's mother is from China and her father is from Nigeria. Of the following, this patient is MOST likely to carry the trait for A.ɑ-thalassemia B.ß-thalassemia C.hemoglobin C D.hemoglobin E

The newborn screen results of the patient in the vignette is consistent with ɑ-thalassemia trait or disease, which presents as hemoglobin (Hb) F,A,Barts. Hemoglobin types are presented on newborn screening tests in the descending order of Hb predominance (ie, the most abundant Hb is listed first and least abundant is listed last). Fetal Hb is designated HbF and adult Hb as HbA; Hb Barts represents a decreased production of ɑ globin chains. Hemoglobin is a tetramer consisting of four subunit proteins that bind to oxygen. The composition of these four subunits determines the type of Hb, which changes during the embryonic-to-fetal-to-adult stage. The predominant Hb produced in the neonatal period is HbF, which is composed of two ɑ chains and 2 ɣ chains. By approximately 6 months of age, the predominant hemoglobins produced are HbA and HbA2 while HbF production is reduced. HbA is composed of two ɑ chains and two β chains, and HbA2 is composed of two ɑ chains and two δ chains. Typically, the Hb F,A pattern indicates a normal newborn Hb pattern, while common β globin gene mutations, resulting in production of HbC, HbE, or HbS, are identified and reported along with HbF on the newborn screen. However, Hb F,A can also be seen in infants with transfusion-dependent β-globin disorders, which may not manifest with significant hemolytic anemia until around 6 months of age because HbA cannot be produced effectively due to the inherited β-globin gene mutations. Non-transfusion-dependent forms of β-thalassemia, including β-thalassemia minor, may also present with the Hb F,A pattern on newborn screen because HbF production is naturally higher than HbA at birth. When an affected infant is about 6 months of age and HbA cannot be produced, the clinical signs of co-inherited β globin chain disorders involving β-thalassemia, HbS, HbC, and/or HbE begin to manifest. The thalassemias are disorders caused by quantitative abnormalities in Hb chain synthesis, leading to abnormal globin chain ratios. In ɑ-thalassemia, the decreased production of ɑ chains leads to a relative excess of unpaired ɣ globin chains (when HbF is predominant) or unpaired β globin chains (when HbA is predominant). These unpaired globin chains can precipitate, leading to the formation of ɣ tetramers, known as Hb Barts in the newborn period, or β tetramers, known as HbH, around and after 6 months of age. The prevalence of ɑ-thalassemias and β-thalassemias are highest in malaria-endemic areas. ɑ-thalassemia genes are frequently seen in Africa as well as in southeast Asian regions, including southern China, Malaysia, and Thailand. β-thalassemia is common in Africa and near the Mediterranean, although mutations for specific populations in Asia have also been described. ɑ-thalassemia is caused by mutations, typically gene deletions, affecting the synthesis of ɑ globin genes. There are four ɑ globin genes; the severity of the ɑ-thalassemia condition is determined by the number of functioning genes that are lost. A single gene deletion (ɑɑ/ɑ-) leads to a silent carrier state, with normal Hb and normal mean corpuscular volume. Individuals with this deletion often have normal newborn screening results (or may have a trace amount of Hb Barts) and normal Hb electrophoresis results. Two gene deletions (ɑ-/ɑ- or ɑɑ/--) lead to ɑ-thalassemia minor (or trait), which presents as Hb Barts (approximately 3%-8%) in the newborn period and as mild microcytic anemia throughout life. The two-gene deletion in cis (ɑɑ/--) is more common in the Asian population and can lead to hydrops fetalis in the homozygous state. The two-gene deletion in trans (ɑ-/ɑ-) is typically seen in the African population and does not usually lead to hydrops fetalis. Three gene deletions (ɑ-/--) lead to HbH disease, with up to 30% HbH and 4% HbA2 present on hemoglobin electrophoresis. Patients with HbH disease usually have a moderate microcytic anemia. Four gene deletions (--/--) present as hydrops fetalis with severe microcytic anemia and death in utero. The Hb analysis will show Hb Barts and the embryonic Hb Portland with an absence of HbA, HbA2, and HbF. The clinical manifestations of HbH disease can vary widely in severity; some patients may have only a mild anemia, whereas others are symptomatic at birth. Most patients with HbH disease are not transfusion dependent but may need occasional transfusions during times of increased hemolysis (eg, with infection) or aplastic crisis. The majority of patients with HbH disease will develop some sequelae of ineffective erythropoiesis and extramedullary hematopoiesis by adulthood, such as iron overload and hepatosplenomegaly, or, less commonly, gallstones, bone deformities, growth impairment, or leg ulcers. PREP Pearls A single ɑ globin gene deletion (ɑɑ/ɑ-) leads to a silent carrier state, with normal complete blood count and newborn screen. Two ɑ globin gene deletions (ɑ-/ɑ- or ɑɑ/--) lead to ɑ-thalassemia minor (or trait), which presents as hemoglobin Barts on the newborn screen and mild microcytic anemia throughout life. The trans form (ɑ-/ɑ-) is typically seen in the African population and does not usually lead to hydrops fetalis. The cis form (ɑɑ/--) is more common in the Asian popluation and can lead to hydrops fetalis in the homozygous state. Three ɑ globin gene deletions (ɑɑ/ɑ-) lead to hemoglobin H disease, with up to 30% hemoglobin H, and clinical manifestations ranging from mild anemia to rare episodes of infection-triggered hemolytic or aplastic crisis that can require transfusions. Four ɑ globin gene deletions (--/--) present as hydrops fetalis with severe microcytic anemia and death in utero.

A previously healthy 2-year-old girl presents with pallor. The physical examination is otherwise unremarkable. Abnormal blood test results are listed below: Red blood cell count 3.5 x 106/uL (3.5 × 1012/L) Hemoglobin 8.4 g/dL (84 g/L) Mean corpuscular volume 60 fL Platelets 472 x 103/uL (472 x 109/L) Reticulocytes 1% Absolute reticulocyte count 0.035 x 106/uL Of the following, the MOST likely condition in this patient is: A.hemoglobin E trait B.hemoglobin H disease C.iron deficiency anemia D.transient erythroblastopenia of childhood

The patient in the vignette has a microcytic anemia with a low red blood cell (RBC) count, which is most consistent with iron deficiency anemia. Since iron is necessary for RBC production, iron deficiency leads to a decrease in the RBC count. Hemoglobin H disease, a form of alpha thalassemia, is associated with an elevated RBC count due to increased production of the microcytic RBCs. The Mentzer index (mean corpuscular volume/RBC count) is sometimes used to help predict whether microcytic anemia is more likely due to iron deficiency or a form of thalassemia. When the Mentzer index is greater than 13, the diagnosis is more likely to be iron deficiency, whereas a Mentzer index less than 13 is suggestive of a form of thalassemia. Hemoglobin E trait, a qualitative abnormality in the beta globin chains, manifests with microcytosis without anemia. Transient erythroblastopenia of childhood is typically associated with macrocytosis, due to a reticulocytosis, or a normocytic anemia. The differential diagnosis of microcytic anemia includes iron deficiency anemia, thalassemias, lead poisoning, sideroblastic anemia, anemia of acute or chronic inflammation, and some unstable hemoglobins. To differentiate among these etiologies, a thorough history, physical examination, peripheral blood smear, iron studies, and hemoglobin electrophoresis are helpful. In addition to the Mentzer index findings, iron deficiency anemia is often associated with a combination of poor iron intake, blood loss, elevated red cell distribution width (RDW), low serum iron levels, elevated total iron binding capacity, and low ferritin. Various forms of thalassemia are sometimes associated with a family history of anemia, elevated RDW, target cells or anisopoikilocytosis, normal iron studies, and abnormal hemoglobin electrophoresis. Symptoms of lead poisoning in children include irritability, behavioral changes, abdominal pain, constipation, decreased growth, and microcytic hypochromic anemia with basophilic stippling. Several of the enzymes necessary for heme production are inhibited by lead. One of these enzymes, ferrochelatase, when impaired, leads to the accumulation of the hemoglobin precursor, protoporphyrin. Measurement of elevated erythrocyte protoporphyrin levels in the peripheral blood may be useful in monitoring lead toxicity, especially in patients with lead levels over 20 μg/dL. The sideroblastic anemias are a rare, heterogenous group of disorders characterized by ineffective iron utilization during erythropoiesis, in the presence of normal to high iron stores. In the bone marrow, non-heme iron accumulates in the mitochondria of erythroid precursors and creates a ring around the nucleus forming the cells known as ringed sideroblasts. Sideroblastic anemia can be inherited, acquired, or idiopathic and is associated with microcytosis or macrocytosis depending on the cause. Hereditary sideroblastic anemia is an X-linked recessive condition, which leads to defects in the enzymes involved in heme production. Mutations in ALAS2, SLC25A38, GLRX5, HSPA9, ABCB7, TRNT1 genes cause microcytic anemia whereas mutations in the SLC19A2, PUS1, YARS2 genes lead to macrocytic anemia. Acquired causes of sideroblastic anemia include drugs (eg, antibiotics, isoniazid, hormones, chemotherapy, copper chelators), toxins, copper deficiency, alcohol use, lead toxicity, or chronic neoplastic disease. Anemia due to inflammation or chronic disease is often a mild normocytic or microcytic anemia, with elevated ferritin levels despite low serum iron, and low total iron binding capacity. Rare unstable hemoglobins can be associated with signs and symptoms of hemolysis (jaundice, splenomegaly, reticulocytosis, low haptoglobin) with a microcytic anemia and normal hemoglobin electrophoresis. Hemoglobin stability studies help establish this diagnosis. PREP Pearls Since iron is necessary for red blood cell production, iron deficiency leads to a decrease in the red blood cell count. Alpha thalassemia, including hemoglobin H disease, a form of alpha thalassemia, is associated with an elevated red blood cell count due to increased production of the microcytic red blood cells. When the Mentzer index (mean corpuscular volume/red blood cell count) is greater than 13, the diagnosis is more likely to be iron deficiency, whereas a Mentzer index less than 13 is suggestive of a form of thalassemia. The differential diagnosis of microcytic anemia includes iron deficiency anemia, thalassemias, lead poisoning, sideroblastic anemia, anemia of chronic inflammation, and unstable hemoglobins. ABP Content Specifications(s)/Content Area Know the differential diagnosis of microcytic anemia

A 5-year-old boy is experiencing his second nosebleed today. He has experienced 4 to 5 episodes of epistaxis every month since infancy and each episode lasts up to 40 minutes. His sister was recently diagnosed with von Willebrand disease during evaluation for menorrhagia. His laboratory data are shown: Laboratory Test Result Hemoglobin 9.8 g/dL (98 g/L) Mean corpuscular volume 70 fL Platelet count 95 ×103/µL (95 × 109/L) Prothrombin time 11.2 s Partial thromboplastin time 30 s Von Willebrand activity 15% Von Willebrand antigen 29% Factor VIII activity 54% Of the following, the MOST appropriate next step in management is to administer A.fresh frozen plasma transfusion B.intranasal desmopressin C.intravenous desmopressin D.oral aminocaproic acid

The patient in the vignette has clinical and laboratory findings consistent with von Willebrand disease (vWD), likely of the type 2B variant, which is associated with thrombocytopenia. Antifibrinolytics, such as oral aminocaproic acid and tranexamic acid, can be a first-line treatment directed at improving hemostasis to control mucosal bleeding symptoms such as epistaxis. Desmopressin can be used to increase von Willebrand factor (vWF) levels when given in its intravenous or intranasal forms, but its use in patients with type 2B vWD is controversial because desmopressin can worsen thrombocytopenia associated with this type. In general, for other types of vWD, a trial of desmopressin is performed during a non-bleeding state to document adequate and sustained increase in vWF levels. However, before considering desmopressin for type 2B vWD, a monitored study demonstrating effect on the platelet count in addition to the vWF level needs to be documented. There is a higher concentration of vWF in cryoprecipitate than in fresh frozen plasma, therefore fresh frozen plasma is not the preferred blood product for replacing vWF. Epistaxis is a common presenting symptom in children with or without an underlying bleeding diathesis. Most episodes of epistaxis can be controlled with direct pressure applied to the Kiesselbach plexus for a minimum of 5 minutes. Referral to a hematologist should be considered for patients with prolonged nasal bleeding, longer than 30 minutes or sufficient to cause anemia, recurrent bleeding, or a family history of a bleeding disorder, as seen in this vignette. Prevention of recurrent epistaxis includes humidification of the nasal mucosa with air humidifier, saline nasal spray, and/or intranasal petroleum jelly at bedtime, avoiding nasal trauma (eg, nose picking), cauterization, topical therapies (eg, antibiotic ointment), and treatment of allergic rhinitis. Nasal corticosteroids are to be avoided in patients with allergic rhinitis, and alternative treatments, such as antihistamines, could be used instead. If patients report swallowing blood, there may be posterior bleeding, and a referral to an otolaryngologist is warranted. The estimated prevalence of vWD in the United States is 0.6% to 1.3%, making it the most common congenital bleeding disorder. Von Willebrand disease is caused by a deficiency or defect of vWF, a protein essential for platelet adhesion to the injured vascular endothelium and as a carrier protein for factor VIII. Type 1 vWD, resulting from a partial quantitative deficiency in vWF, accounts for 70% to 80% of cases. Type 3 vWD, characterized by the absence of vWF, accounts for less than 5% of cases. The remaining 20% of vWD cases are type 2 variants, caused by qualitative defects in vWF. Type 2B vWD is caused by a gain-of-function mutation in the A1 region of VWF, resulting in increased binding between vWF and platelets and leading to increased clearance of both platelets and high-molecular-weight vWF multimers. Desmopressin can increase the release of the abnormal vWF protein and worsen the thrombocytopenia. Thrombocytopenia is not present in all cases of type 2B vWD; diagnosis can also be made by sequencing exon 28 of VWF. Platelet-type vWD presents with similar laboratory findings as seen in type 2B vWD (ie, low vWF, thrombocytopenia, decreased high-molecular-weight vWF multimers). However, platelet-type vWD is caused by mutation in the GP1b receptor on platelets, not in VWF. This mutation in the vWF receptor on platelets results in increased binding to and clearance of vWF as well as thrombocytopenia and is treated with platelet transfusions. In 2007, the National, Heart, Lung, and Blood Institute (NHLBI) vWD Expert Panel published clinical practice guidelines for making the diagnosis of vWD based on the presence of both clinical criteria consisting of personal and/or family history and/or physical evidence of mucocutaneous bleeding, and laboratory criteria including avWF level activity or antigen < 30 IU/dL. These recommendations allow for the diagnosis and treatment of vWD in individuals with vWF levels of 30 to 50 IU/dL in the presence of personal and/or family evidence for VWD. Individuals with vWF levels between 30 and 50 IU/dL with or without an extensive personal or family history of bleeding symptoms may be categorized as having "low vWF" instead of type 1 vWD. Lower mean vWF levels are also observed in individuals with blood type O. Regardless of the etiology, any individual with a significant clinical or family history of bleeding and low vWF level may benefit from treatment to increase the vWF level. The goal of treatment for vWD patients is to stop or prevent bleeding by increasing vWF concentration and/or promoting hemostasis. Desmopressin stimulates the release of vWF from the Weibel-palade bodies in endothelial cells and is approved for use in vWD in intravenous and intranasal formulations. The NHLBI recommends performing a desmopressin trial during a nonbleeding state to document adequate and sustained response. Desmopressin can be contraindicated in patients with type 2B vWD because it may worsen thrombocytopenia caused by increased platelet clearance. Human plasma-derived or recombinant vWF concentrates can replace vWF, especially in cases of urgent bleeding or when desmopressin is contraindicated. Antifibrinolytics, such as epsilon-aminocaproic acid and tranexamic acid, inhibit plasminogen and plasmin from lysing the fibrin clot. They are taken orally or administered intravenously to control mucosal bleeding, such as epistaxis and menorrhagia. PREP Pearls Antifibrinolytics, such as oral aminocaproic acid and tranexamic acid, can be a first-line treatment directed at improving hemostasis to control mucosal bleeding symptoms such as epistaxis in patients with von Willebrand disease. Desmopressin can be used to increase von Willebrand factor levels when given in its intravenous or intranasal forms, but its use in patients with type 2B von Willebrand disease is limited because desmopressin can worsen thrombocytopenia associated with this type. Before considering desmopressin for type 2B von Willebrand disease, a monitored study demonstrating the effect of administered desmopressin on the platelet count in addition to the von Willebrand factor level is highly recommended. ABP Content Specifications(s)/Content Area Von Willebrand: New topic of LOW vWD vs diagnosis of disease, management of epistaxis in same

A previously healthy, 15-year-old, African American girl receives intravenous vancomycin for methicillin-resistant Staphylococcus aureus osteomyelitis. Three weeks later, after she has completed therapy, she develops a sore throat with low-grade fever and body aches. Laboratory data are shown: Laboratory Test Result White blood cell count. 4,000/µL (4.0 × 109/L) Neutrophils. 10% Lymphocytes. 75% Monocytes. 10% Eosinophils. 5% Hemoglobin. 12.8 g/dL (128 g/L). Platelet count. 395 × 103/µL (395 × 109/L) Of the following, the MOST likely cause of her condition is A.aplastic anemia B.benign ethnic neutropenia C.cyclic neutropenia D.idiosyncratic drug reaction

The patient in the vignette most likely has idiosyncratic drug-induced neutropenia (IDIN) resulting from vancomycin exposure. The diagnosis of IDIN is based largely on the presentation of clinical-onset fever, chills, sore throat, oral ulcers, myalgia, arthralgia, and severe neutropenia weeks to months after exposure to the causative drug, as definitive laboratory testing for IDIN is not currently available for all suspected medications. Aplastic anemia is less likely, because there are no other types of cytopenia to suggest bone marrow hypoproduction. Benign ethnic neutropenia, while common in the African American population, is usually associated with a mild neutropenia, with the absolute neutrophil count (ANC) typically above 1.0 × 109/L and no symptoms. Recurrent neutropenia on a 21-day cycle is seen in cyclic neutropenia, a rare condition associated with mutations in the neutrophil elastase gene, ELANE, which leads to a shortened neutrophil lifespan requiring approximately 21 days for the body to replenish its supply. These patients also often have recurrent infections. Neutropenia is generally defined as an ANC of less than 5.0 × 109/L in neonates in the first month of life, less than 1.0 × 109/L in infants, and less than 1.5 × 109/L in children and adults. Severe neutropenia is defined as an ANC of less than 0.5 × 109/L and associated with a significant risk for infection. Medications can cause neutropenia through decreased production of neutrophils, as seen with chemotherapy drugs, or increased neutrophil destruction via idiosyncratic reactions to other drug types. The annual incidence of IDIN in the pediatric population is estimated to be 3.92 cases per 10,000 pediatric patients and 2.4 to 15.4 cases per million in the general population. It is usually associated with severe neutropenia (ANC of less than 0.5 × 109/L) and a mortality rate of approximately 5% because of septicemia. Although the definitive mechanism of IDIN is unknown, an immune process is supported by the delay in onset of neutropenia ranging from 1 week to 6 months after initial exposure to the drug, and shorter onset of reaction upon re-challenge. Thus, if re-exposure to the offending medication is necessary, it should be undertaken with caution and close monitoring. In the hapten hypothesis, the drug or a reactive metabolite of the drug acts as a hapten (ie, a low-molecular-weight molecule that cannot independently elicit an immune response, but that when bound to a carrier protein forms a complex capable of stimulating T cell or antibody production) and binds irreversibly to the neutrophil membrane, leading to neutrophil destruction. This mechanism is implicated in IDIN related to drugs such as propylthiouracil, amodiaquine, flecainide, and clozapine. There may also be a component of direct damage to neutrophils and myeloid precursors as described with clozapine, chlorpromazine, procainamide, and dapsone. Based on numerous case series, there is a vast array of medications implicated in IDIN, with medications such as clozapine (antipsychotic drug) and thionamides (antithyroid drugs) having the greatest risk of severe neutropenia. Diagnosis of IDIN may be challenging in neutropenic patients with concurrent infection, as the infection may be the cause or the result of the neutropenia. Poor prognostic indicators in IDIN include older age (older than 65 years), severely low ANC (less than 0.1 × 109/L), severe infection or sepsis, and comorbidities (renal, cardiac, pulmonary, or autoimmune disease). In cases of severe neutropenia with suspected IDIN, a thorough examination is needed to determine the cause of infection, and broad-spectrum antibiotics need to be initiated promptly. Granulocyte colony-stimulating factor administration has been shown to shorten recovery time, antibiotic course, and length of hospitalization. If IDIN is suspected, withdrawal of the offending drug is warranted, even in the absence of any clinical symptoms. Neutropenia typically resolves 1 to 3 weeks after the drug is discontinued, with a mean recovery time of 12 to 15 days and a range of 3 to 56 days. PREP Pearls Idiosyncratic drug-induced neutropenia is associated with severe neutropenia (absolute neutrophil count less than 0.5 × 109/L), a mortality rate of approximately 5% owing to septicemia, and can be caused by numerous medications. Presenting signs and symptoms of idiosyncratic drug-induced neutropenia are fever, chills, sore throat, oral ulcers, myalgia, arthralgia, and onset of severe neutropenia weeks to months after exposure to the drug. Granulocyte colony-stimulating factor administration has been shown to shorten recovery time, antibiotic course, and length of hospitalization in idiosyncratic drug-induced neutropenia. If idiosyncratic drug-induced neutropenia is suspected, withdrawal of the offending drug is warranted, even in the absence of any clinical symptoms with resolution of neutropenia 1 to 3 weeks after the drug is discontinued, and a mean recovery time of 12 to 15 days with a range of 3 to 56 days.

A previously healthy 6-year-old boy (weight 20 kg) diagnosed with acute immune thrombocytopenia presents with severe epistaxis. He has normal coagulation screening results and a normal complete blood count except for a platelet count of 5 × 103/µL [5 × 109/L]). He has no prior bleeding history and no family history of bleeding disorders. Despite treatment with intravenous methylprednisolone, immunoglobulin, and platelet transfusion, his epistaxis worsens with significant blood volume loss and hematemesis. He becomes hypotensive and receives 4 units of packed red blood cells. Laboratory results before and after transfusions reveal: Laboratory Test. Initial Result. Result After Receipt of 4 Units of Packed Red Blood Cells Platelet count. 5× 103/µL (5 × 109/L). 6 × 103/µL (5 × 109/L) Hemoglobin. 12 g/dL (70 g/L). 7 g/dL (70 g/L) Prothrombin time. 12sec. 23 sec Activated partial thromboplastin time. 30 sec. 70 sec Fibrinogen. 250 mg/dL (0.5 g/L). 50 mg/dL (0.1 g/L) D-dimer. 0.5 µg/mL (2.73 nmol/L 0.55 µg/mL (3.01 nmol/L) Of the following, the MOST likely explanation of this patient's coagulopathy is A.acquired inhibitor of coagulation B.consumptive coagulopathy C.dilutional coagulopathy D.excessive fibrinolysis

The patient in this vignette had ongoing, significant, rapid blood loss. He was transfused with approximately 1,200 mL of packed red blood cells (PRBC) based on 1 unit of PRBC= 300 ml. His estimated blood volume is 65 mL × body weight (20 kg) = 1,300 mL. These amounts meet criteria for 'massive blood transfusion' defined in children as the replacement of > 30 mL of blood/kg of body weight with evidence of ongoing blood loss. (In adults, massive blood transfusion is defined as the replacement of at least one blood volume in 24 hours or one-half of the blood volume in 3 hours.) The resulting hypocoagulable state in this vignette arises due to the transfusion of a large volume of PRBC with ongoing massive blood loss without replacement of coagulation factors by transfusion of fresh frozen plasma and cryoprecipitate. Substantial blood loss of platelets and coagulation factors combined with massive transfusion of PRBC lead to extensive hemodilution with defective hemostasis called dilutional coagulopathy. Dilutional coagulopathy is complex and involves deficiency in all elements of coagulation, including procoagulant, anticoagulant, fibrinolytic, and antifibrinolytic proteins. Hypofibrinogenemia is a major component of dilutional coagulopathy. Replacement of coagulation factors and fibrinogen with transfusions of fresh frozen plasma and cryoprecipitate are the mainstay of prevention and treatment. Fresh frozen plasma contains many procoagulant, anticoagulant, and antifibrinolytic factors but due to its low fibrinogen concentration, a large volume of fresh frozen plasma is needed to achieve an adequate increase in fibrinogen. Therefore, cryoprecipitate which contains a higher concentration of fibrinogen, is necessary for adequate fibrinogen replacement. Platelet transfusion is also indicated for treatment of dilutional thrombocytopenia in massive transfusions. In children, an acquired inhibitor of coagulation—often a lupus anticoagulant—is typically transient, asymptomatic, and does not cause bleeding. It poses a risk of thrombosis in adults or in adolescents with autoimmune disease. It manifests with marked prolongation of activated partial thromboplastin time (aPTT), normal fibrinogen levels, and rarely a prolonged prothrombin time (PT). Consumptive coagulopathy, also known as disseminated intravascular coagulopathy, may result from extensive tissue damage from substantial surgery, severe trauma, infection, vascular malformations, drugs, massive gastrointestinal or mucosal hemorrhage without or without an underlying bleeding diathesis. Such large-scale endothelial injury results in tissue factor activation, thrombin activation, and excessive fibrin deposition. The diagnosis of consumptive coagulopathy is made on the basis of a supportive clinical history such as severe trauma, surgery or infection, with prolongation of PT and aPTT, a decrease in fibrinogen level, presence of schistocytes or helmet cells, and an elevated D-dimer level. Hyperfibrinolysis is often considered in non-traumatic bleeding when coagulation factors and platelet counts are normal. Excessive fibrinolysis can result from massive transfusions with ongoing severe blood loss due to a rapid decrease in antifibrinolytic proteins, as a result from disseminated intravascular coagulopathy or from excess tissue plasmin activator or plasmin due to thrombolytic therapy. In this vignette, the excessive bleeding precedes the coagulation abnormalities which arise after dilution from massive transfusion. In excessive fibrinolysis, depletion of procoagulant factors result in prolongation of PT and aPTT, low fibrinogen level, and elevated D-dimer levels. PREP Pearls Massive blood loss and massive transfusion lead to extensive hemodilution with resulting dilutional coagulopathy. Dilutional coagulopathy is complex and involves deficiency in all elements of coagulation, including procoagulant, anticoagulant, fibrinolytic, and antifibrinolytic proteins. The mainstay of prevention and treatment of dilutional coagulopathy is the transfusion of fresh frozen plasma, cryoprecipitate, and platelets.

A 5½-month-old male infant is admitted for receipt of intravenous antibiotics for a suspected staphylococcal superinfection of eczematous lesions and new onset moderate neutropenia. Physical examination reveals crusted, erythematous skin lesions and a lack of tonsillar tissue. Family history reveals that a maternal uncle died of complications related to bronchiectasis and another maternal uncle died of a chronic enteroviral infection. Laboratory data are shown: Laboratory Test Result White blood cell count 1,800/μL (1.8 × 109/L) Absolute neutrophil count 600/μL (0.60 × 109/L) Hemoglobin 10.7 g/dL (107 g/L) Platelet count 345 × 103/μL (345 × 109/L) IgG 53 mg/dL (0.53 g/L) IgM 3 mg/dL (30 mg/L) IgA < 6 mg/dL (< 60 mg/L) Of the following, the MOST likely underlying mechanism for these findings is A.absence of somatic hypermutation in B-cell lymphocytes B.failure to produce mature CD19+ B-cell lymphocytes C.lack of immunoglobulin isotype class switching D.physiologic, age-appropriate immunoglobulin nadir

The patient in this vignette has a clinical history concerning for X-linked agammaglobulinemia (XLA), which is caused by a mutation in the Bruton tyrosine kinase gene (BTK) and results in a failure to produce mature CD19+ B-cells. His male sex, the emergence of infections as maternally transferred IgG wanes between 4 and 6 months of age, and low serum immunoglobulin levels point toward a significant defect in humoral immunity. Importantly, tonsillar tissue, which consists largely of B-cells in healthy individuals, was not seen on examination. The family history of maternal male relatives with complicated bronchiectasis and chronic enteroviral infection is consistent with an X-linked immunodeficiency. Both the lack of tonsillar tissue and the family history argue against transient hypogammaglobulinemia of infancy. Neutropenia is also seen in some patients with XLA, often in association with infection. The incidence of XLA is estimated to be 1 in 190,000 live male births. Patients are at risk for recurrent, life-threatening infections, particularly those that require opsonization such as encapsulated organisms (eg, Streptococcus pneumoniae and Haemophilus influenzae). Patients with XLA are also susceptible to giardiasis and enteroviral infections. Treatment is regular infusions of intravenous immunoglobulin. However, despite these treatments, patients with XLA continue to have recurrent infections on mucosal surfaces with high rates of chronic sinusitis and bronchiectasis, which is a leading cause of morbidity and mortality. The target supplemented IgG level is 800 g/L, which has helped to mitigate the infectious complications. Patients are also at risk of developing lymphoproliferative disorders and malignancies of the gastrointestinal tract. Immunodeficiency also results from other defects in B-cell development. In particular, patients who are unable to complete class switch recombination (ie, transition from a μ heavy chain [IgM]-expressing B-cell to an alternate heavy chain [IgG, IgA, or IgE]) have hyper-IgM syndrome. Hyper-IgM syndrome can result from several genetic mutations, the most common of which is a defect in CD40 ligand (CD40L), or can be acquired, as in the setting of congenital rubella infections and phenytoin exposure. Patients with hyper-IgM syndrome are not required to have elevated levels of IgM for diagnosis. Because mutations in CD40L and CD40 affect interactions between T-cells, which express CD40L, and antigen-presenting cells (eg, B-cells and macrophages that express CD40), they result in a combined immunodeficiency placing patients at risk for opportunistic infections such as Pneumocystis jiroveci. Other mutations resulting in the hyper-IgM phenotype include mutations in the genes encoding activation-induced cytidine deaminase and uracil N-glycosylase; these enzymes are important for optimized somatic hypermutation, which can occur only in mature B-cells. Infants less than 6 months of age typically are too young to have a sufficient number of mature B-cells to be protective or are too young to be symptomatic from the absence of somatic hypermutation in B-cells, unlike this vignette. Defective class switching has also been noted in patients with defects in the NF-kB pathway (eg, mutations in IKGKB/NEMO) and patients with activated PI3-kinase delta syndromes type 1 and 2, which are caused by mutations in PIK3CD and PIK3R1, respectively. These conditions are also diagnosed by elevated IgM levels, and low IgG and IgA levels, unlike this vignette. The normal physiologic, age-appropriate immunoglobulin nadir, between 3 to 6 months of age typically reveals normal IgG levels reflecting the additive contribution of waning placentally transferred maternal IgG, slightly low IgM levels ranging between 50-75% of normal adult values, and moderately low IgA levels ranging between 10-20% of normal adult values, unlike the patient described in this vignette. PREP Pearls X-linked agammaglobulinemia is caused by mutations in BTK that result in an inability to produce mature CD19+ B cells. Presenting features associated with X-linked agammaglobulinemia may include low serum immunoglobulin levels, lack of tonsillar tissue, recurrent infections, intermittent neutropenia, and a family history of bronchiectasis or chronic infections. Defective class switch recombination results in an ability to produce IgM but not IgG, IgA, or IgE and should raise concern for hyper-IgM syndrome. ABP Content Specifications(s)/Content Area Know the genetics of immunoglobulin production Know the biological properties of human immunoglobulins

A 3-month-old male infant has a history of mild eczema and new-onset petechiae. He has scattered petechiae and multiple 1-cm dry, scaling maculopapular lesions on his extremities without adenopathy or hepatosplenomegaly. Laboratory data are shown: Laboratory Test Result White blood cell count 11,200/µL (11.2 × 103/L) Hemoglobin 12.1 g/dL (121 g/L) Platelet count 10 × 103/µL (10 × 109/L) Absolute lymphocyte count 6,900/µL (6.9 × 109/L) Absolute neutrophil count 2,800/µL (2.8 × 109/L) Peripheral blood smear Decreased platelet count, small platelets Absolute CD3+ cells (reference range, 2,700-3,508/µL) 3,060/µL Absolute CD4+ cells (reference range, 1,919-2,472/µL) 2,754/µL Absolute CD8+ cells (reference range, 351-2,479/µL) 100/µL Absolute CD16+CD56+ cells (reference range, 15-300/µL) 290/µL Absolute CD19+ cells (reference range, 432-3,345/µL) 1,275/µL WAS protein ratio (reference range, 0.71-1.31) 0.1 Immunoglobulin levels are normal. Of the following, the MOST accurate statement regarding allogeneic hematopoietic stem cell transplantation for this infant is A.confirmatory genetic testing is needed prior to transplant B.an HLA-matched sibling donor is not an option C.a reduced-intensity conditioning regimen is standard D.younger age is a favorable prognostic factor

The patient in this vignette has typical manifestations of Wiskott-Aldrich syndrome (WAS), an X-linked recessive syndrome. Classic WAS findings may include microthrombocytopenia, eczema, lymphopenia, T-cell dysfunction, susceptibility to infections, and increased risk of autoimmune diseases and malignancy. Wiskott-Aldrich syndrome comprises approximately 1% to 1.5% of all primary immunodeficiency diagnoses in the United States. Historically, patients with severe WAS had a life expectancy of 2 decades without hematopoietic stem cell transplant (HSCT). Clinical findings are heterogeneous and dependent on the genetic mutation within WAS, which is located on the short arm of the X chromosome. Mutations in WAS affect production of the WAS protein (WASp), which impacts the survival and function of terminally differentiated cells. Mutations that cause a minor decrease in WASp expression have milder phenotypes, leading to the condition of X-linked thrombocytopenia. X-linked neutropenia is a separate clinical entity in which there is a mutation in the WAS binding domain. Larger deletions or nonsense mutations that truncate the mRNA construct early lead to more significantly decreased or absent WASp production. Patients with severely decreased or absent WASp will typically develop classic WAS. Therefore, confirmatory genetic testing is not necessary for a diagnosis of classic WAS, rather the phenotype including symptoms and degree of WASp expression, determines the severity of WAS. To assist in determining the need for HSCT, a scoring system was created to differentiate amongst X-linked thrombocytopenia, X-linked neutropenia, and classic WAS (Table). The score is based on the severity of clinical manifestations and degree of the loss of WASp expression. Patients with mutations in WAS that had a lower risk score (< 3 risk factors) demonstrated fewer complications after HSCT compared to patients with higher risk scores. In addition, improved outcomes have been observed in patients who received HSCT before the age of 5 years. A trend toward better outcomes before 2 years of age is potentially attributable to the increasing risk over time of developing infection and autoimmunity. Thus, young age is a favorable prognostic factor for better HSCT outcomes. With current HLA-typing search strategy and supportive care, allogeneic HSCT is recommended for patients diagnosed with clinically significant WAS. Pretransplant morbidity, donor stem cell source, and age of the recipient at HSCT remain significant determinants of post-transplant morbidity and mortality. Hematopoietic stem cell transplant should be considered when an HLA-matched donor is identified, given the risk of malignancy, autoimmune diseases, and bleeding problems. Options for HSCT include haploidentical donors, HLA-matched siblings, HLA-matched unrelated donors, and HLA-matched cord blood units. Unlike other primary immunodeficiency conditions, choice of pre-HSCT conditioning regimen may play an important role for patients with WAS. Mixed donor chimerism increases risk of post-HSCT autoimmune disorders or persistent thrombocytopenia. Patients with WAS have a high rate of graft rejection, which may be caused by insufficient stem cell dose and low degree of myelosuppression due to the conditioning regimen. Therefore, a full myeloablative chemotherapy conditioning regimen consisting of busulfan for myeloablation and cyclophosphamide or fludarabine for improved immunosuppression is recommended. Reduced-intensity regimens are not considered standard for patients with WAS. PREP Pearls Wiskott-Aldrich syndrome is a heterogeneous condition due to WAS mutations affecting protein expression, leading to the varying clinical manifestations of the disease. Pretransplant risk factors including age play a significant role in overall survival of patients with Wiskott-Aldrich syndrome after hematopoietic stem cell transplant. Unlike some other primary immunodeficiency diseases, in Wiskott-Aldrich syndrome myeloablative chemotherapy regimens are important to prevent mixed donor chimerism and graft rejection.

A 14-year-old adolescent girl has a 1-week history of worsening headache, now associated with night awakening and vomiting. Magnetic resonance imaging reveals a tumor (Figure). A biopsy is performed and reveals poorly differentiated glial cells that diffusely infiltrate the thalamus. Of the following, the BEST next step in confirming the suspected histologic diagnosis is A.fluorescence in situ hybridization for rearrangement of RELA B.fluorescence in situ hybridization for rearrangement of YAP1 C.immunohistochemistry for H3K27M mutation D.immunohistochemistry for INI1 expression

The presentation and imaging of the adolescent in this vignette is consistent with diffuse midline glioma (DMG). Diffuse midline glioma is a World Health Organization (WHO) grade IV diffusely infiltrating astrocytoma named for its location in the midline of the brain or spinal cord and for the presence of a histone gene mutation, H3K27M. Most pediatric DMGs arise in the pons or thalamus. They occur less frequently in the cerebellum and spinal cord. Because they arise in eloquent locations, they are not amenable to complete surgical removal. Eighty percent of midline gliomas harbour a histone H3 mutation; these mutations correlate with an aggressive clinical behaviour and poor response to treatment. Detection of the H3K27M mutation in tissue can be done through genetic sequencing or immunohistochemistry (histopathologic staining) and is now considered diagnostic for DMG. Ependymomas are glial tumors derived from differentiated ependymal cells lining the ventricles of the brain and the central canal of the spinal cord. They represent 6% to 12% of all pediatric brain tumors. Forty percent of ependymomas are supratentorial, while 60% are infratentorial. Supratentorial ependymomas (ST-EPN) most commonly occur in the cerebral parenchyma arising from embryonic rests of ependymal tissue trapped in the developing cerebral hemispheres. Molecular subgroups of ST-EPN have characteristic fusion genes involving RELA and YAP1 (ST-EPN-RELA and ST-EPN-YAP1). Fusion genes involving RELA and YAP1 can be assessed with fluorescent in situ hybridization. Atypical teratoid/rhabdoid tumor (AT/RT) is a rare childhood tumor which may develop throughout the central nervous system. Sixty percent of AT/RTs occur in the posterior fossa. Although there are rare case reports of AT/RT arising in the thalamus, most thalamic tumors in children are astrocytomas. Histologically AT/RT resembles rhabdomyosarcoma. INI1 (hSNF5/SMARCB1), located on 22q11.2, is deleted or mutated in AT/RT. Immunostaining for INI1 can be used to demonstrate the loss of INI1 expression, which is characteristic for AT/RT. For the first time in 2016, WHO combined both histological and molecular features in the classification of pediatric brain tumors. Diffuse midline glioma, H3K27M mutant, denotes a histologic diagnosis of pediatric high-grade astrocytoma that arises in the midline location of the brain and spinal cord and includes a molecular diagnosis that indicates the presence of a mutation at position K27 of H3 histone coding genes. The H3K27M mutation may occur in one of several histone-related genes, H3F3A or HIS1H3B/C. Lysine (K) is substituted for methionine (M) at position 27. The K27 residue may be acetylated or methylated, resulting in posttranslational changes that contribute to tumorigenesis. Unfortunately, DMG is uniformly fatal and remains the number one cause of brain cancer death in children. The median survival ranges from 4 to 17 months. Radiation therapy results in clinical improvement but has no effect on overall survival. Fusion genes are useful as diagnostic tools and may serve as therapeutic targets for more effective treatment in the future. PREP Pearls Diffuse midline glioma is a World Health Organization grade IV diffusely infiltrating astrocytoma named for its location in the midline of the brain or spinal cord and for the presence of a histone gene mutation, H3K27M. H3K27M mutation, detected by genetic sequencing or immunohistochemistry, correlates with an aggressive clinical behaviour and poor response to treatment in diffuse midline glioma.

A previously healthy 1-year-old boy has a 2-week history of daily fevers, fatigue, irritability, and refusing to walk. He has pallor and extensive ecchymoses without adenopathy or hepatosplenomegaly. A complete blood count reveals: Laboratory Test Result White blood cell count 2.4 × 103/µL (2.4 × 109/L) Hemoglobin 7.2 g/dL Platelet count 8 × 103/µL (8.0 × 109/L) Segmented neutrophils 10% Lymphocytes 81% Eosinophils 4% Blasts 5% A bone marrow aspiration results in an inadequate marrow for flow cytometry, but cytogenetics reveals a RBM15-MKL1 [t(1;22)(p13;q13)] fusion transcript. Bone marrow biopsy reveals medium to large blasts with blue vacuolated, eosinophilic cytoplasm containing fine granules and irregular cytoplasmic borders or pseudopod projections; nuclei with reticular, dense chromatin; two to three nucleoli; and increased reticulin staining. Of the following, the MOST likely blast immunophenotype is A.CD45+, CD13-, CD33+, CD34-, CD41+, CD42+, CD61+, HLA-DR- B.MPO++, CD13+, CD15+, CD33+, CD117+, CD34-, HLA-DR- C.CD45+, TdT+, CD10+, cyCD3+, sCD3+, CD4+, CD5+, CD8+, HLA-DR- D.TdT+, CD10+, CD19+, CD20-, CD22+

The presentation of symptoms, peripheral blood, marrow morphology and cytogenetics is most consistent with acute megakaryoblastic leukemia (AMKL), formerly known as French-American-British system typing (FAB) M7. Immunophenotyping, cellular morphology (the basis for FAB classification), and cytogenetics are necessary for diagnosis of most hematological neoplasms. T-cell acute lymphocytic leukemia (T-ALL), acute promyelocytic leukemia (APL) or FAB M3, and AMKL are three of the most commonly HLA-DR negative-associated acute leukemias, although rare HLA-DR positive cases have been reported. They can be differentiated by the expression of other lineage-specific markers. Acute megakaryoblastic leukemia is characterized by the presence of megakaryocytic surface glycoproteins (CD41, CD42, CD61) on blasts (response choice A). Whereas APL is associated with a strong myeloperoxidase expression and presence of myeloid molecular markers (CD13, CD15, CD33, CD117) (response choice B), T-ALL is characterized by presence of T cell markers (cyCD3, sCD3, CD4, CD5, CD8) (response choice C). Classical B-cell markers (CD10, CD19, CD20, CD22) are associated with B-cell lymphoblastic leukemia (response choice D). Acute megakaryoblastic leukemia is a rare subtype of acute myelogenous leukemia (AML) associated with an abnormal maturation of the megakaryocyte lineage. Whereas AMKL accounts for about 5% to 15% of all pediatric cases of AML, it is the predominant subtype in infants and children with Down syndrome younger than 3 years. Acute megakaryoblastic leukemia is often associated with myelofibrosis, possibly owing to the expression of platelet-derived growth factor and transforming growth factor β, which can contribute to increased reticulin staining and inadequate bone marrow aspiration. In this case, bone marrow biopsy is necessary to aid in diagnosis. Non-DS AMKL is characterized by the following chimeric or fusion oncogenes, in order of frequency: CBFA2T3-GLIS2 [inv(16)(p13.3q24.3)], RBM15-MKL1 [t(1;22)(p13;q13)], and NUP98-KDM5A [t(11;12)(p15;p13)], yet up to 35% of cases have no identifiable mutations. In children with Down syndrome (DS) or trisomy 21, AMKL has been associated with somatic GATA1 mutations. About 10% to 30% of infants with DS who develop transient abnormal myelopoiesis (TAM), a temporary proliferation of blasts, will eventually develop AML, which in the majority is of the megakaryoblastic subtype or myelodysplastic syndrome, or both. Acute megakaryoblastic leukemia associated with DS has demonstrated better outcomes relative to children with non-DS-associated AMKL. PREP Pearls Acute megakaryocytic leukemia, a subtype of acute myelogenous leukemia, is characterized by the presence of megakaryocytic surface glycoproteins CD41, CD42, and CD61. Non-Down syndrome acute megakaryocytic leukemia is characterized by the following chimeric or fusion oncogenes, in order of frequency: CBFA2T3-GLIS2 [inv(16)(p13.3q24.3)], RBM15-MKL1 [t(1;22)(p13;q13)], and NUP98-KDM5A [t(11;12)(p15;p13)]. Approximately 10% to 30% of patients with transient abnormal myelopoiesis associated with GATA1 mutations during infancy will develop acute myelogenous leukemia (often of acute megakaryocytic leukemia subtype), myelodysplastic syndrome, or both during childhood.

A 6-month-old male infant has gingival bleeding. On examination, he is also noted to have scattered petechiae and bruises on the face, trunk, and extremities. Prothrombin time and activated partial thromboplastin time are normal. The complete blood count shows normal white blood cell count and hemoglobin with a platelet count of 19 × 103/µL (19 x 109/L) and mean platelet volume of 10 fL (normal range, 7-12 fL). Immature platelet fraction is 2% (normal range, 1% to 9%). Of the following, the MOST likely cause of thrombocytopenia in this patient is A.absence of GP1b-IX-V complex B.antibodies against paternal platelet antigens C.mutation in c-Mpl D.mutation in MYH-9

The presentation of the infant in the vignette, who has severe thrombocytopenia with normal-sized platelets and a low-normal production of new platelets, is consistent with congenital amegakaryocytic thrombocytopenia (CAMT). Congenital amegakaryocytic thrombocytopenia is an autosomal recessive disorder associated with decreased or absent functional thrombopoietin receptors due to mutations in c-Mpl. This leads to decreased or absent production of megakaryocytes and platelets in the bone marrow. Patients with CAMT can progress to pancytopenia and myelodysplasia. Hematopoietic stem cell transplantation is recommended as definitive therapy. Defects of the GP1b-IX-V complex are pathognomonic for Bernard-Soulier syndrome, an autosomal recessive disorder characterized by large platelets as evidenced by an elevated mean platelet volume, and mild to moderate thrombocytopenia. The GP1b-IX-V complex is necessary for the binding of platelets to von Willebrand factor. Antibodies against paternal platelet antigens is the cause of neonatal alloimmune thrombocytopenia (NAIT), which manifests as severe thrombocytopenia in the newborn and a high risk of severe bleeding in the first few days after birth. Analogous to the placental transfer of antibodies from an Rh-negative mother to an Rh-positive fetus, in NAIT, maternal IgG antibodies against fetal platelet antigens inherited from the father can cross the placenta and cause platelet destruction in the fetus. After delivery, the neonate does not continue to make antibodies against his or her own platelets, and thus the thrombocytopenia resolves within several weeks as the maternal antibodies disappear. The late onset of symptoms in the child in the vignette is not consistent with NAIT. The MYH-9 related disorders are caused by inherited mutations in the MYH-9 gene, which lead to chronic, mild to severe macrothrombocytopenia increasing the risk for mild to severe bleeding. MYH-9 related thrombocytopenia is inherited in an autosomal dominant manner and associated with a constellation of other clinical features, including hearing loss, renal disease, cataracts, and/or Dohle-like inclusion in the cytoplasm of neutrophils. Treatment addresses bleeding from injury or surgery with platelet transfusions, desmopressin, and/or antifibrinolytics if mucosal bleeding is involved. Disorders with increased platelet turnover, such as immune or antibody-mediated conditions, are associated with an increased immature platelet fraction, as well as an elevated mean platelet volume because younger platelets are larger than older platelets. The immature platelet fraction (IPF) parameter on automated cell counters measures the percentage of circulating platelets that contain RNA, thus indicating the amount of young reticulated platelets in the peripheral blood. The IPF is increased in high turnover states such as immune thrombocytopenia and disseminated intravascular coagulation, and it is decreased in bone marrow failure. There is also emerging evidence that a low IPF may predict an increased risk of experiencing bleeding in thrombocytopenic disorders. Platelets are small anucleate cells derived from megakaryocytes in the bone marrow under the regulation of thrombopoietin. Transcription factors GATA-1 and NF-E2 found on megakaryocytes are involved in megakaryocyte proliferation and differentiation. Megakaryocytes and proplatelets (elongated large strands of megakaryocyte cytoplasm that eventually break off into platelets) are released by the bone marrow, travel to the lungs, and form platelets. Each megakaryocyte produces 1,000 to 3,000 platelets. Approximately 1 trillion platelets circulate in the blood and are involved in hemostasis, inflammation, atherosclerosis, autoimmunity, and tumor immunology. Approximately one-third of the total platelet supply is normally stored in the spleen. Although the normal lifespan of platelets in the circulation is 7 to 10 days, it can be shortened by immune-mediated destruction (eg, immune thrombocytopenia, drug-induced thrombocytopenia), consumption (eg, disseminated intravascular coagulation, thrombotic thrombocytopenic purpura, surgery, trauma), mechanical destruction (eg, cardiopulmonary bypass), or sequestration (eg, hypersplenism). PREP Pearls The immature platelet fraction parameter on automated cell counters measures the percentage of circulating platelets that contain RNA, thus indicating the amount of young reticulated platelets in the peripheral blood. The immature platelet fraction is increased in high turnover states, such as immune thrombocytopenia and disseminated intravascular coagulation, and decreased in bone marrow failure. Although the normal lifespan of platelets in the circulation is 7 to 10 days, it can be shortened by immune-mediated destruction (eg, immune thrombocytopenia, drug-induced thrombocytopenia), consumption (eg, disseminated intravascular coagulation, thrombotic thrombocytopenic purpura, surgery, trauma), mechanical destruction (eg, cardiopulmonary bypass), or sequestration (eg, hypersplenism). ABP Content Specifications(s)/Content Area

19-year-old man completed therapy for stage IIIA Hodgkin lymphoma 7 years ago. He was treated with 4 cycles of chemotherapy that consisted of doxorubicin, bleomycin, vincristine, etoposide, prednisone, and cyclophosphamide. He did not receive radiation therapy. He was evaluated last week for a 3-week history of worsening fatigue and bruising and was found to have pancytopenia. Results of a bone marrow biopsy show myelodysplasia. Of the following, the MOST likely cytogenetic mutation associated with this patient's presentation is A.5q deletion B.11q23 translocation C.21q22 translocation D.t(9;11)(p22;q23)

The risk of developing a secondary malignancy within 20 years after completion of Hodgkin lymphoma treatment is around 10% in males and slightly higher in females because of the increased risk of breast cancer. While the most common secondary malignancies after Hodgkin lymphoma therapy are breast cancers and sarcomas, leukemia often develops the soonest after completion of treatment. The most common treatment-associated leukemia is acute myelogenous leukemia, commonly referred to as t-AML. Whereas all of the mutations listed in the response choices are associated with t-AML, they are linked to different agents and have different latencies. Treatment-associated acute myeloid leukemia associated with alkylating agents typically manifests 3 to 5 years or longer after initiation of therapy or may present as myelodysplastic syndrome (MDS) before progressing to AML, as in this vignette. Alkylating agents associated with t-AML include cyclophosphamide, ifosfamide, melphalan, busulfan, dacarbazine, carmustine, lomustine, semustine, chlorambucil, and mechlorethamine. Chromosomal abnormalities associated with alkylating agents include deletions involving chromosomes 5 and/or 7. The latency of 7 years after therapy and presentation of MDS in this patient is likely due to his exposure to cyclophosphamide and to a deletion in chromosome 5 or 7. Topoisomerase II inhibitors such as the epipodophyllotoxins, etoposide and teniposide, in addition to anthracyclines, doxorubicin and idarubicin, have been associated with increased risk of developing t-AML. Risk varies depending on treatment doses, timing of exposure, and genetic predispositions. Treatment-associated AML arising after exposure to these agents are related to translocations involving 11q23, 21q22, or t(9;11)(p22;q23). Although these mutations are common in t-AML, they rarely present as MDS but instead as AML. These translocations often manifest as t-AML within the first 3 years after initiation of topoisomerase II inhibitor therapy. Cyclophosphamide has also been associated with acute, subacute, and late effects involving the urinary tract and renal dysfunction. Early in therapy, patients may experience hemorrhagic cystitis or dysuria. After completion of therapy, cyclophosphamide has been associated with long-term tubular and glomerular injury, hypertension, and bladder malignancies. Other alkylating agents have potential late effects as well. Busulfan has been associated with cataracts. Busulfan, carmustine, and lomustine have been associated with pulmonary fibrosis. Adult survivors of childhood cancer treatment that included alkylating agents are at risk for infertility. Although men have a higher incidence, women are at greater risk after treatment with higher cumulative doses. Evaluation for sexual dysfunction includes assessment of libido, maintenance of erections in men, menstrual patterns in women, and ability to conceive children. PREP Pearls Survivors of childhood Hodgkin lymphoma are at increased risk of developing a variety of secondary malignancies such as breast cancers, sarcomas, and leukemia. Treatment-associated acute myeloid leukemia (AML) due to alkylating agents typically manifests 3 to 5 years or longer after therapy initiation, may present as myelodysplastic syndrome before progressing to AML, and is often associated with deletions in chromosomes 5 and 7. Alkylating agents are associated with a range of other late effects in childhood cancer survivors, including renal dysfunction, urinary tract issues, and infertility.

An 18-month-old boy has been experiencing persistent morning vomiting for 1 week. Brain magnetic resonance imaging shows a large, heterogeneous, enhancing posterior fossa tumor with eccentric cystic components, calcifications, and intratumoral hemorrhage. Immunostaining of the biopsied tumor cells reveals loss of INI1 expression in the nucleus. His family history is significant for his older brother diagnosed with the same tumor at 3 years of age and no other cancers in other members. Of the following, the germline mutation MOST likely associated with this tumor is A.DICER1 B.SDHA C.SMARCB1 D.TP53

The tumor imaging and immunostaining findings in this vignette are most compatible with atypical teratoid/rhabdoid tumor. The young age of tumor development, the family history of a sibling also affected with the same tumor at a young age, and this rare type of central nervous system (CNS) tumor are suggestive of a familial cancer predisposition syndrome known as rhabdoid predisposition syndrome. Up to 95% of cases of rhabdoid predisposition syndrome are associated with germline mutations in SMARCB1. Children affected by this syndrome may develop rhabdoid tumors in the CNS, kidney, and extra-renal sites. Rhabdoid tumors of the CNS are called "atypical teratoid/rhabdoid tumors," while renal or extra-renal tumors are called "malignant rhabdoid tumors." Atypical teratoid/rhabdoid tumor associated with this syndrome typically develops in infants and children less than 4 years of age and resembles a primitive neuroectodermal tumor with rhabdomyoblastic-appearing cells, previously misdiagnosed as medulloblastoma. Other tumors affecting children with this syndrome include schwannomas. Approximately one-third of children with rhabdoid tumors have germline mutations in the INI1/SMARCB1 gene located on chromosome 22q11.2. This gene mutation affects chromatin modeling which impacts DNA replication and repair affecting cell cycle growth, division, and maturation. Because INI1/SMARCB1 is a tumor suppressor gene, both alleles must be inactivated for a tumor to arise. Although the majority of germline mutations are de novo, families with rhabdoid tumor predisposition have been identified. DICER1 mutations are found in two-thirds of pediatric cases of pleuropulmonary blastoma. The inheritance of DICER1 mutation is autosomal dominant with incomplete penetrance and variable expression. Since the de novo germline mutations are common in this rare tumor, a search for DICER1 mutation is recommended for affected individuals. Examples of other tumors associated with DICER1 mutations include Wilms, ocular tumors, pineoblastoma, rhabdomyosarcoma, and germ cell tumors. These tumors are typically not associated with loss of INI1 nuclear expression. Affected individuals may benefit from interval tumor surveillance. Hereditary paraganglioma/pheochromocytoma syndrome is a rare cancer predisposition syndrome associated with paragangliomas arising from the parasympathetic nervous system of the head and neck, and pheochromocytomas arising in the adrenal glands or sympathetic ganglia. If a child is diagnosed with either of these rare conditions, genetic testing for succinate dehydrogenase mutations is strongly suggested. These mutations occur in genes encoding subunits of the succinate dehydrogenase complex such as SDHA, SDHB, SDHC, SDHD, and SDHAF2. These tumors are typically not associated with loss of INI1 nuclear expression. Affected individuals may benefit from interval tumor surveillance. Patients with Li-Fraumeni syndrome have a mutation of TP53. TP53 is a tumor suppressor gene that plays an important role in cellular response to stress, in DNA repair, and in apoptosis. This is a highly penetrant cancer predisposition syndrome in which multiple family members may be affected. Patients with Li-Fraumeni syndrome are at increased risk of developing osteosarcoma, central nervous system tumors (gliomas, medulloblastomas, and choroid plexus carcinoma), breast cancer, and/or adrenocortical carcinoma. These tumors are typically not associated with loss of INI1 nuclear expression. Affected individuals may benefit from interval tumor surveillance. PREP Pearls Approximately one-third of children with atypical rhabdoid teratoid tumor have germline INI1/SMARCB1 alterations located on chromosome 22q11.2. SMARCB1 is a tumor suppressor gene, and both alleles must be inactivated for a tumor to arise. The majority of SMARCB1 germline mutations are de novo, but there are familial cases in which siblings with atypical rhabdoid teratoid tumor have been identified.

A 16-year-old adolescent girl was treated for bilateral retinoblastoma at 14 months of age with unilateral enucleation, systemic chemotherapy, and laser cryotherapy. She was found to have a germline mutation in the RB1 gene. She currently is reporting progressive swelling and worsening pain to her right knee for the past 3 months. On examination, she is afebrile, and her right lateral proximal shin demonstrates a 3-cm firm, fixed, tender mass. Of the following, the BEST next step in her plan of care is to A.obtain blood cultures B.obtain radiographs of the right knee and calf C.perform fine needle aspiration of the shin mass D.recommend rest, ice, compression, and elevation

This adolescent most likely has developed a soft-tissue sarcoma of her right lower extremity associated with the germline mutation in her RB1 gene. Radiographic imaging of her right lower extremity is a rapid way to distinguish possible tumor from trauma, infection, or benign bony lesion. Given the chronic nature of the symptoms and her underlying cancer predisposition syndrome, a regimen of rest, ice, compression, and elevation is not appropriate because this is likely not a sprain or strain. Although biopsy of the tissue is necessary, it would not be the first step in diagnosis. The girl has had no fevers or skin changes to suggest infection, so blood cultures are not indicated. Retinoblastoma (RB) may manifest as unilateral, bilateral, or trilateral (involving the pineal gland) disease. There are approximately 200 cases of RB diagnosed per year in the United States; 60% of cases are nonhereditary with unilateral disease, 25% of cases are hereditary with bilateral disease, and 15% of cases are hereditary with unilateral disease. Hereditary RB has been traditionally described as a malignancy arising as a result of Knudson's two-hit hypothesis, meaning both RB1 alleles undergo inactivation, one likely inherited and the other a somatic inactivation. Mutation in the RB1 gene (13q14) leads to hereditary retinoblastoma, which typically occurs in an autosomal-dominant fashion with incomplete penetrance. For example, a parent with RB1 gene mutation has an approximately 45% chance of passing on the RB1 gene mutation to each child. However, genetic testing for RB is complicated by test reliability, as well as by presence or absence of germline mosaicism in peripheral blood lymphocytes and tumor tissue, when available, in the affected child and parents. Genetic counseling is recommended for first-degree relatives depending on these various risk characteristics. The protein produced by RB1 helps with growth, differentiation, and apoptosis, and 95% of all retinoblastoma tumors are detected by 5 years of age. Children with germline RB1 mutation tend to develop bilateral retinoblastoma more commonly and at a younger age of 10 months, compared with the typical age of "sporadic" RB development, which is 24 months. For this reason, it is imperative to perform fundoscopic examinations on all siblings of children with any form of retinoblastoma. Children with hereditary RB are at risk of developing second cancers, especially if their treatment included radiation as part of upfront therapy. The cancers seen more commonly in children with hereditary retinoblastoma include osteosarcoma; soft-tissue sarcoma; breast cancer; melanoma; small cell lung carcinoma; lymphoma; and oral, nasal, brain, bladder, and uterine cancers. PREP Pearls Retinoblastoma is the most common intraocular malignancy, with approximately 200 cases per year in the United States. Retinoblastoma may occur as a sporadic or inherited condition, with the retinoblastoma gene mutation passed on in an autosomal-dominant pattern with incomplete penetrance. Children with inherited retinoblastoma are at risk of developing additional cancers, including bone and soft-tissue sarcomas, small cell lung cancer, melanoma, lymphoma, and oral, nasal, brain, bladder, uterine and breast cancers.

A previously healthy, 3-year-old Syrian male has a 2-day history of worsening pallor, fatigue, tachycardia, and port wine-colored urine. He has been taking oral trimethoprim sulfamethoxazole for the past 3 days for an otitis media. He has a history of neonatal jaundice requiring phototherapy. He has scleral icterus and no splenomegaly. Laboratory data are shown: Laboratory Test. Result Hemoglobin 6 g/dL (60 g/L). Mean corpuscular volume 75 fL Reticulocyte count. 6%. Direct antiglobulin test. Negative Of the following, the peripheral blood smear MOST consistent with this child's presentation is A. A. Response Choice A Choice A B. B. Response Choice B Choice B C. C. Response Choice C Choice C D. D. Response Choice D Choice D

This child has signs and symptoms of acute intravascular hemolysis, manifesting with new-onset severe anemia, hyperbilirubinemia, and hemoglobinuria. In a male of Middle Eastern or Mediterranean descent with hemolysis, the inheritance of glucose-6-phosphate dehydrogenase (G6PD) deficiency should be considered as an underlying condition especially after exposure to oxidative stressors. Oxidants associated with hemolysis in individuals with G6PD include infections, certain foods such as fava beans, certain chemicals such as naphthalene in mothballs, and medications such as antimalarial and sulfa drugs (Table). Glucose-6-phosphate dehydrogenase is the first enzyme of the hexose monophosphate shunt. The main function of G6PD is to reduce nicotinamide adenine dinucleotide phosphate, which is required for the prevention of oxidative damage through the reduction of glutathione. Normally, glutathione is oxidized by an offending chemical and then reduced by G6PD to prevent damage. If G6PD is deficient, the supply of reduced glutathione is depleted, which results in oxidation of the sulfhydryl groups of hemoglobin and, in turn, the denaturing and precipitation of hemoglobin with subsequent red blood cell (RBC) lysis. Denatured and precipitated hemoglobin on peripheral blood smear detected by supravital staining with methyl violet are called Heinz bodies, as shown in response choice B (Figure 1). Hemoglobin H bodies are unstable β chain tetramers that precipitate and form RBC inclusions called "golf balls" when stained with brilliant cresyl blue preparation (response choice A) (Figure 2). Hemoglobin H disease is an inherited, lifelong, symptomatically moderate form of α-thalassemia, with microcytic hypochromic anemia secondary to deficient α-globin chains. Basophilic stippling, as evidenced on hematoxylin and eosin staining and commonly seen in β-thalassemia trait and other forms of ineffective erythropoiesis, is caused by precipitation of fragments of ribosomal RNA (response choice C) (Figure 3). A Howell-Jolly body is a nuclear remnant or cluster of DNA that has not been removed from circulation owing to functional hyposplenism. Sickled RBCs and a nucleated RBC are also evident on the peripheral blood smear shown as response choice D (Figure 4). In 2019, the World Health Organization's grouped variants of G6PD deficiency into 5 classes according to levels of G6PD activity and clinical characteristics as follows: Class I: severely deficient (less than 2% residual activity), associated with chronic nonspherocytic hemolytic anemia Class II: severely deficient (less than 10% residual activity), associated with intermittent acute hemolytic episodes Class III: moderately deficient (10% to 60% residual activity), associated with rare acute hemolytic episodes Class IV: normal activity (60% to 150% activity), no clinical symptoms Class V: increased activity (> 150% activity), no clinical symptoms These classes are not distinct nor restricted to enzymatic activity ranges. Affected individuals may demonstrate similar G6PD activity levels with varying severity of hemolysis. Although there has been a gradual shift from biochemical analysis to mutation analysis, most individuals with G6PD deficiency are classified according to their clinical behaviors. Patients with class I G6PD deficiency suffer from chronic hemolysis with severe G6PD deficiency, although G6PD activity levels up to 10% have been reported. The 2 most common or classically recognized forms of deficiency in the United States are classes II and III. Class II variants typically demonstrate severe acute hemolytic episodes with exposure to oxidative stressors, and have traditionally included the Mediterranean form (G6PD Mediterranean), Middle Eastern, Indian, and Asian (G6PD Canton and G6PD Gaohe) variants. The largest subgroup affected by G6PD-deficiency in North America are African Americans, with a prevalence of approximately 10 % in African American males. The African American form (G6PDA-) falls mostly in Class III. In G6PD-deficient African American patients, most hemolytic episodes are mild with only a few severe symptomatic occurrences of hemolysis being reported. Glucose-6-phosphate dehydrogenase deficiency is an X-linked inherited disorder, associated with more than 400 mutations located on band Xq28. Affected males are hemizygous for the deficiency. Heterozygous females are usually clinically normal. However, depending on the random X chromosome inactivation and the degree of expression of the abnormal G6PD variant, the mean RBC enzyme activity in heterozygous females can vary from normal to severely deficient. Therefore, although it is a rare occurrence, some heterozygous females are as deficient in G6PD as are hemizygous males and experience severe hemolytic anemia. In populations with very high frequency of the G6PD-deficient allele, females can be homozygous or compound heterozygous. These females can have G6PD deficiency of the same clinical severity as that of hemizygous males. Individuals with G6PD deficiency are asymptomatic and hematologically normal in absence of exposure to oxidative agents or stressors. Acute onset of hemolysis in G6PD deficiency occurs hours or days from the exposure to the oxidative drug. Anemia varies in severity depending on the amount of red blood cell destruction. The severity of hemolysis is dose dependent, based on the amount of drug, chemical exposure (such as to naphthalene, the chemical in mothballs), or ingestion of foods (fava beans, other legumes) . Additional factors that affect the severity of hemolysis include the nature of the genetic enzyme defect, drug pharmacokinetics, and the presence of concurrent oxidative stressors such as infection. Examples of oxidative drugs associated with severe G6PD deficiency-associated hemolysis are: antimalarials such as primaquine, sulfonamide antibiotics such as sulfamethoxazole and sulfacetamide, antipyretics, urate oxidase, and analgesics such as acetanilide . Other antimicrobials include dapsone, quinolones and nitrofurantoin. Anemia is typically normocytic normochromic with compensatory reticulocytosis. Heinz bodies are often rapidly removed by the spleen, resulting in "bite" or "blister" cells commonly seen on hematoxylin-and-eosin-stained peripheral blood smear (Figure 5). Testing enzyme activity in the newborn period or during acute hemolysis can result in diagnostic difficulties. The diagnosis of G6PD deficiency should be suspected in severe or prolonged neonatal jaundice, particularly in newborns of Mediterranean or African descent. However, the diagnosis of G6PD deficiency may be missed when the enzyme assay is performed in the neonatal period, because neonates have a young red blood cell population that could result in a false-normal enzyme level. Similarly, in older children, during acute hemolysis, the destruction of large numbers of G6PD-deficient older red blood cells and the presence of a large number of reticulocytes and young red blood cells can result in false-normal G6PD levels. PREP Pearls Glucose-6-phosphate dehydrogenase deficiency is an X-linked metabolic disorder, most commonly found in hemizygous males, sometimes arising in heterozygous females owing to random X inactivation, and less commonly presenting in homozygous females. In a child of Middle Eastern, Asian, African, or Mediterranean descent with hemolysis, the inheritance of glucose-6-phosphate dehydrogenase deficiency should be considered as an underlying condition especially after exposure to oxidative stressors. Common oxidative stressors that precipitate hemolysis in people with glucose-6-phosphate dehydrogenase deficiency include antimalarials such as primaquine, sulfa drugs such as dapsone, food such as fava beans, chemicals such as henna and naphthalene (found in mothballs), and infection.

6-year-old girl with homozygous (hemoglobin SS) sickle cell anemia presents with acute-onset temperature of 39.1°C. She has a 2 day history of rhinorrhea. She appears well with stable vital signs and is not in respiratory distress. Her lungs are clear to auscultation. A blood culture is obtained and her complete blood count reveals: Laboratory Test Result White blood cell count 22,500/µL (25 × 109/L) Hemoglobin 8.5 g/dL (85 g/L) Platelets 604 × 103/µL (604 × 109/L) Neutrophils 72% Lymphocytes 17% Monocytes 7% Eosinophils 4% Absolute reticulocyte count 330 × 103/µL (330 × 109/L) Of the following, the BEST antibiotic for this patient is A.azithromycin B.ceftriaxone C.clindamycin D.vancomycin

This young child with sickle cell disease (SCD) and presumed functional asplenia is susceptible to bacterial infections, particularly encapsulated bacteria such as Haemophilus influenzae, Streptococcus pneumoniae, group B streptococci, Neisseria meningitidis, and Salmonella species. Because fever may be the first or only sign of invasive bacterial infection, children with SCD need prompt medical evaluation for body temperature greater than 38.5°C. Evaluation includes history and physical examination, complete blood count with differential, reticulocyte count, and blood culture, followed by administration of empiric antibiotics. In the United States, most patients receive broad-spectrum, gram-negative, and some gram-positive bacterial coverage with intravenous or intramuscular ceftriaxone. If the child is allergic to cephalosporins, intravenous clindamycin, which is active against Streptococcus spp, is an adequate alternative. However, in a well-appearing child with stable vital signs and normal respiratory status whose care can likely be managed on an outpatient basis, ceftriaxone will provide 24 hours of antibiotic treatment, whereas clindamycin is administered every 6 to 8 hours. Intravenous vancomycin, which is effective against staphylococci, enterococci, and streptococci viridans, is added if there are signs and symptoms of septic shock, meningitis, or both. Azithromycin, which treats Mycobacterium spp, Streptococcus spp, H influenzae, and Chlamydia spp, among others, is used in combination with ceftriaxone in suspected cases of acute chest syndrome. In addition to the evaluation discussed above, urinalysis and urine culture are obtained when dysuria, hesitancy, frequency, urgency, or flank or suprapubic pain are present. Chest radiograph is performed when there are signs or symptoms of lower respiratory tract infections such as tachypnea, hypoxemia, chest pain, and decreased breath sounds or crackles on lung auscultation. Osteomyelitis is considered in the setting of extremity pain and fever but can be challenging to diagnose, because symptoms are often similar to concurrent vaso-occlusive pain. Although criteria for hospital admission are institution specific, several adverse signs or symptoms warranting hospitalization include ill appearance, respiratory distress, hypotension, hypoxemia, or altered sensorium. Functional asplenia in SCD occurs secondary to recurrent vaso-occlusion in the spleen and subsequent autoinfarction. WIthout a spleen, there is dysfunctional antibody production and poor opsonophagocytosis, making individuals with SCD susceptible to encapsulated bacteria. In the absence of pneumococcal vaccination or penicillin prophylaxis, children with SCD have a 600-fold higher risk of developing invasive pneumococcal disease compared with children who do not have SCD. A 2019 systematic review of the literature in the current prophylactic era of oral penicillin prophylaxis and pneumococcal immunization (PCV13) found that the prevalence of invasive pneumococcal disease in children with SCD is 1.9% (95% CI; 1.7%-2.2%). In this study, the median age at diagnosis was 4.8 years, the majority of patients had hemoglobin SS disease, and the majority of pneumococcal serotypes identified were non-PCV13 serotypes. The Prophylactic Penicillin Study showed children treated with penicillin prophylaxis had 84% less infection than those who had not been so treated. These results led to universal newborn screening for SCD to allow early initiation of penicillin prophylaxis. Penicillin prophylaxis needs to be provided to all children with hemoglobin SS and sickle β0 thalassemia who are younger than 5 years. In children with other forms of SCD, need for penicillin prophylaxis is based on institution-specific guidelines. Children with SCD from birth to 3 years of age need penicillin VK 125 mg orally twice daily. At age 3, the oral dose is increased to 250 mg twice daily. If a child is allergic to penicillin, erythromycin may be substituted. Penicillin prophylaxis may be discontinued at age 5 years if the patient is fully vaccinated and has no history of invasive pneumococcal infection or splenectomy. Children with SCD need to be immunized against S pneumoniae, H influenzae type B, and N meningitidis as per the Centers for Disease Control and Prevention recommendations. This includes routine immunization during infancy as well as booster immunizations throughout childhood. All patients with SCD should receive annual influenza vaccination starting at 6 months of age. PREP Pearls Fever (≥ 38.5°C) in a patient with sickle cell disease is a medical emergency and requires evaluation with history and physical examination, complete blood count with differential, reticulocyte count, and blood culture, followed by administration of empiric antibiotics. Despite penicillin prophylaxis and pneumococcal vaccination, children with sickle cell disease continue to be at increased risk of developing invasive pneumococcal disease due to functional asplenia compared with children who do not have sickle cell disease.

A 17-year-old adolescent girl with a history of osteosarcoma of the right distal femur was treated successfully with chemotherapy and limb salvage surgery with placement of a megaprosthesis 3 years ago. She is seen for a routine off-therapy cancer survivorship visit. She notes that her health has been good, but her right upper thigh and hip are painful when she runs in softball practice. She is afebrile and has tenderness along the lateral aspect of her right thigh, without masses, redness, or swelling. Her complete blood count and C-reactive protein are normal. Radiography reveals normal femoral head and femoral neck with a 0.5 cm jagged edged area of lucency in the femur shaft at the proximal margin of the megaprosthesis. The surrounding soft tissue appears normal. Of the following, the MOST likely cause of her pain is: A.femoral decalcification B.infection in the soft tissue C.mechanical loosening of the prosthesis D.recurrent osteosarcoma

Total bone tumor resection of the lower extremity with megaprosthesis implantation has become a common treatment for certain patients with localized osteosarcoma and other bone tumors. A megaprosthesis is a mechanical device or endoprosthesis used to reconstruct a skeletal defect, manufactured from titanium or other metal alloy, which is often adjustable to allow for linear growth with a hinged knee to allow for preservation of motion. In the upper extremities, allograft-prosthetic composite reconstruction has been used after radical bone tumor resection providing greater ability to attach muscles and tendons to the allograft bone. Both of these mechanical devices have the potential for complications months to years after implantation including infection, tumor recurrence, mechanical loosening of the prosthesis from the bone, or mechanical failure of the device itself. Loosening of the bone around the megaprosthesis can be painful, particularly after brisk exercise. Tenderness to palpation without masses or swelling and lucency along the edge of the prosthesis on radiograph supports this complication in this vignette. Decalcification near the prosthesis is common particularly in a patient with little or no physical activity. While decalcification can be part of the mechanical loosening/failure process, the rest of the femur in this patient is normal, without osteopenia or osteoporosis. Infection can result in bone lucency near the megaprosthesis, but is typically associated with fever, redness, swelling, leukocytosis, or an elevated c-reactive protein, unlike in this vignette. Local recurrence of tumor manifests with a painful or painless mass with a mixture of lytic and sclerotic bony lesions on radiography. However, there is no mass palpable on examination and radiographic findings in this vignette are not consistent with tumor relapse. While we have no data as to this patient's smoking, vaping, or tobacco chewing habits, increased nicotine exposure has been associated with an increased risk of aseptic loosening of these prosthetic devices. Smoking, vaping, and tobacco chewing avoidance or cessation counseling is recommended for these patients. Many centers are using silver coated prosthetic devices to lower the risk of infection postimplantation. Thus far, these have had good results with little or no evidence of injury to surrounding tissue or accumulation of silver systemically. PREP Pearls Allograft-prosthetic composite grafts are often implanted in upper extremity limb salvage procedures while megaprosthesis are implanted in lower extremities after total bone tumor resection. Potential complications, months to years after implantation of endoprostheses, used to reconstruct skeletal deficits following total bone tumor resection include infection, tumor recurrence, mechanical loosening of the prosthesis from the bone, or mechanical failure of the device itself.

A previously transfused 17-year-old adolescent boy with sickle cell anemia was admitted to the hospital for the first time with acute chest syndrome. He required a packed red blood cell transfusion because of respiratory distress and hypoxia. No antibodies were detected, and he was transfused with an extended phenotype-matched packed red blood cell unit. His post- transfusion hemoglobin level was 10.3 g/dL (103 g/L). He returned to the emergency department 7 days after discharge with fever, low back pain, and jaundice. His hemoglobin level was 4.7 g/dL (47 g/L). The direct antiglobulin test had a positive result. The type and crossmatch revealed anti-Fya antibodies. The eluate revealed a pan-agglutinating autoantibody. The previous treating physician was contacted and confirmed that the patient had a history of anti-Fya antibodies. Of the following, the BEST precautionary step that would have prevented this complication is A.knowing patient's prior antibody status B.premedication with steroids C.providing sickle-negative packed red blood cells D.screening the unit for babesiosis

Transfusion of persons with sickle cell disease requires special precautions. In addition to being ABO compatible, the units should also be matched for D, C, c, E, e, and K antigens as well as any red cell alloantibodies. Some centers provide antigen matching for additional clinically significant antigens, such as Fya, Fyb, Jka, Jkb, M, N, and S, after a patient has developed antibodies. Alloantibody titers decrease over time and may become undetectable. Therefore, it is important to match for current as well as historic antibodies. Patients may not be aware of their antibody status. It is important to contact the blood bank at the previous treating institution to obtain the patient's previous antibody history prior to transfusion. In this patient who has developed delayed hemolytic transfusion reaction (DHTR) after a packed red blood cell transfusion, knowing the patient's previous antibody status and matching the units accordingly could have prevented the development of DHTR. Delayed hemolytic transfusion reaction occurs 1 to 21 days after a transfusion. Patients develop fever, a decreasing hemoglobin level, dark urine, and the new onset or worsening of pain. New alloantibodies may be identified along with a positive direct and/or indirect antiglobulin test. Serial hemoglobin electrophoresis may help identify if the hemolysis is affecting donor or recipient red blood cells. If the patient requires another transfusion, the most compatible unit is selected based on the patient's red blood cell phenotype/genotype and antibody status. For patients who have refractory hemolysis and do not get an increase in their hemoglobin level after the subsequent transfusion, corticosteroids, intravenous immune globulin, and rituximab have been used as well. Premedication of this patient prior to the transfusion in the vignette would not have affected the development of DHTR if the unit was not matched based on his historic antibody status. All packed red blood cell units given to sickle cell patients must be sickle negative. This would also not affect the development of DHTR. Transfusion-transmitted babesiosis may lead to a clinical picture similar to DHTR in patients with sickle cell disease. Screening for babesiosis is recommended in endemic areas. However, hemolysis related to babesiosis is not antibody mediated and should not lead to the development of alloantibodies or autoantibodies. PREP Pearls Delayed hemolytic transfusion reaction usually occurs 1 to 21 days after a packed red blood cell transfusion and is caused by alloantibody development. The clinical presentation may mimic a vaso-occlusive episode. Patients with sickle cell anemia require special precautions prior to a transfusion, including sickle-negative, extended phenotype matched blood. In addition, if the patient has a history of alloantibodies, it is critical to give packed red blood cell units that are negative for that antigen to prevent a delayed hemolytic transfusion reaction. Alloantibody titers decrease over time and may become undetectable. Therefore, it is important to obtain the patient's historic antibody status from previous treating institutions.

A 3-year-old girl with acute immune thrombocytopenia and epistaxis was treated with intravenous immunoglobulin 1 week ago. She is being evaluated in clinic for follow-up. Of the following, the MOST likely transient laboratory abnormality caused by her treatment is A.decreased plasma osmolality B.elevated serum sodium level C.increased blood lymphocyte count D.positive direct antiglobulin test (Coombs)

Treatment with intravenous immune globulin (IVIG) may cause transient laboratory abnormalities that are unlikely to be of clinical consequence. These abnormalities include: artifactual hyponatremia and hypomagnesemia; increased plasma osmolality; positive antibody tests including antinuclear antibody and rheumatoid factor; positive direct antiglobulin test (DAT, Coombs); and transient lymphopenia or neutropenia. Up to 30% of patients receiving high-dose IVIG treatment may have transient positive DAT in the absence of clinically significant hemolysis. This is thought to occur secondary to nonspecific binding of IgG to red blood cells during transient hypergammaglobulinemia after IVIG treatment. Patients may also rarely experience clinically significant hemolysis after IVIG treatment. This can occur secondary to the presence of red blood cell-specific antibodies in the pooled plasma that react with the patient's red blood cells. In a patient with newly positive DAT, it is important to rule out hemolysis prior to presumption of false-positive DAT. Hemolysis would be confirmed by the presence of anemia, increased reticulocytosis, decreased haptoglobin level, elevated indirect bilirubin level, and elevated lactate dehydrogenase level. If this evaluation is negative, no further investigation is needed to attribute the positive DAT to the IVIG treatment. Based on the half-life of IVIG, this false-positive DAT may persist up to 3 months after treatment. PREP Pearls Up to 30% of patients who receive intravenous immune globulin may have a transient positive direct antiglobulin test persisting up to 3 months without apparent hemolysis. A positive direct antiglobulin test 3 months after intravenous immune globulin treatment or associated with anemia at anytime should be investigated for possible hemolysis. Treatment with intravenous immune globulin can cause transient laboratory abnormalities that may be of no clinical consequence, such as increased plasma osmolality, positive antinuclear antibody, pseudohyponatremia, pseudohypomagnesemia, positive direct antiglobulin test, and transient lymphopenia or neutropenia.

A 1-year-old Asian-American girl is referred for microcytic anemia detected on routine screening. The girl's mother also has a history of chronic anemia. The infant's hemoglobin is 10.1 g/dL and mean corpuscular volume is 65 fL. Results of iron studies are normal. Hemoglobin (Hb) analysis shows Hb A 95%, Hb A2 2%, Hb F 3%. Of the following, the patient MOST likely has A.α-thalassemia trait B.ß-thalassemia trait C.hemoglobin E trait D.hereditary persistence of fetal hemoglobin

When an infant is about 6 months of age, the predominant hemoglobin switches from fetal hemoglobin (Hb F) to adult hemoglobin (Hb A). Fetal hemoglobin is composed of two ɑ chains and two ɣ chains, whereas Hb A is composed of two ɑ chains and two β chains. In ɑ-thalassemia, there is decreased production of the ɑ chains, leading to unpaired ɣ chains in the neonatal period (when Hb F is the predominant Hb), detected as Hb Barts (ɣ4) on the newborn screen. In patients with ɑ thalassemia trait (2 gene deletion), Hb Barts may be present on the newborn screening but is not detectable on hemoglobin electrophoresis after 6 months of age, once Hb A becomes the most abundant Hb. In contrast, disorders of β globin synthesis, including β-thalassemia, and Hb E would be detectable on Hb electrophoresis as elevated Hb A2 and presence of Hb E, respectively, at 1 year of age, when Hb A (a2b2) is the predominant hemoglobin, as seen in the patient in the vignette. In hereditary persistence of fetal hemoglobin, the expected Hb F should be higher (Table 1). A change in predominant hemoglobin occurs from embryonic to fetal to postnatal/adult stages (Table 2); therefore, ζ and έ globin chain disorders are clinically relevant only during embryonic development, γ chain disorders are present only in early infancy, and ɑ globin disorders present earlier than do β globin disorders. Fetal hemoglobin can be increased in disorders affecting globin gene expression (eg, hereditary persistence of fetal hemoglobin [HPFH], δɣ thalassemia), β globin gene disorders (eg, β-thalassemia and sickle cell anemia), as well as conditions of stress erythropoiesis (eg, malignancy, inherited bone marrow failure) (Table 1). Fetal hemoglobin can modulate the clinical and hematologic features of sickle cell anemia, and higher Hb F levels have been associated with decreased mortality. This is one of the mechanisms by which hydroxyurea works to ameliorate the severity of sickle cell disease. PREP Pearls In ɑ-thalassemia, there is decreased production of the ɑ chains, leading to unpaired ɣ chains in the neonatal period (when Hb F is the predominant hemoglobin), detected as Hb Barts (ɣ4) on the newborn screening. In patients with ɑ-thalassemia trait (two-gene deletion), hemoglobin Barts may be present on the newborn screening but usually is not detectable on hemoglobin electrophoresis after 6 months of age, once hemoglobin A becomes the most abundant hemoglobin. A change in predominant hemoglobin occurs from embryonic to fetal to postnatal/adult stages; therefore, ζ and έ globin chain disorders are clinically relevant only during embryonic development, ɣ chain disorders are present only in early infancy, and ɑ globin disorders present earlier than do β globin disorders. ABP Content Specifications(s)/Content Area Know the globin chain composition of embryonic, fetal, and adult hgbs Know the relative concentrations of embryonic, fetal, and adult hgbs in a newborn infant and variations in pathologic states

A 12-year-old boy has a non-tender, non-erythematous, 4 cm, firm, immobile, left anterior cervical lymph node which had been enlarging for the past 3 months. He is clinically well without fever, night sweats, or weight loss. A complete lymph node excision is performed, and pathology shows nodular lymphocyte-predominant Hodgkin lymphoma. Positron emission tomography-computed tomography shows no residual, nodal, or extranodal disease, consistent with stage 1A disease. Of the following, the BEST next step in management is A.involved-field radiation therapy B.multiagent chemotherapy C.observation D.rituximab

ll Hodgkin lymphomas (HLs) are of B-cell origin, but multiple unique subtypes have been identified. Classical Hodgkin lymphomas (cHLs) include nodular sclerosis HL, mixed-cellularity HL, and lymphocyte-depleted HL. Nodular lymphocyte-predominant HL (NLPHL) has clinicopathological features distinct from those of cHL and constitutes about 5% of all HLs. For stage 1 NLPHL, complete resection may be curative and observation is recommended as subsequent management. Although the peak incidence of NLPHL occurs in the fourth decade, NLPHL also occurs in children. Males are more commonly affected, and most patients present with stage I or II disease. The presentation is often long-standing isolated peripheral lymphadenopathy without systemic or constitutional symptoms. Mediastinal and axillary lymph nodes are less commonly involved than in cHL. B symptoms and extranodal disease are uncommon, as compared with cHL. Staging and response assessment in NLPHL is similar to that in cHL. The disease is fluorodeoxyglucose (FDG) avid, and FDG-positron emission tomography is used for NLPHL. However, standardized uptake values are usually lower than those in cHL. Histologically, the malignant cells in NLPHL are called "lymphocyte-predominant" (LP) cells. These are sometimes descriptively referred to as "popcorn cells" because of the presence of polylobed nuclei, as demonstrated in the Figure. The infiltrate generally appears to be vaguely nodular, but diffuse areas can also be seen. A diffuse architectural pattern is associated with disease relapse. The cells express CD45 and pan B-cell markers and are usually positive for BCL-6 and OCT2. In contrast to cHL, CD20 is often expressed whereas CD30 and CD15 are usually negative. The LP cells do not express programmed cell death ligand 1 or 2 (PD-L1 or PD-L2). The immunoglobulin genes are clonally rearranged, and the cells often demonstrate multiple chromosomal abnormalities. Low-stage disease has an excellent prognosis, and the treatment required is less intensive than in cHL. In pediatric patients, two studies have shown that surgery alone may be adequate for the majority of low-stage patients who are in imaging-proven, complete remission (CR) after excisional biopsy. In the Children's Oncology Group AHOD03P1 trial, 52 patients with completely resected stage 1A NLPHL had a 5-year event-free survival of 77% and overall survival of 100%. In an older European Network Group study, the progression-free survival at 43 months for patients in CR after surgery was 57%. All of the patients who were not in CR after surgery relapsed, but overall survival was still 100% because the salvage rate is excellent. The patient in this vignette has completely resected stage 1A disease and is in CR; therefore, he is a good candidate for observation, including serial examination and surveillance imaging. Chemotherapy, radiation, and rituximab are not needed for this patient but do have a role in NLPHL therapy. In pediatric patients with low-stage disease and incomplete resection, management consists of combination chemotherapy with or without radiation. Chemotherapy regimens have not been standardized but frequently involve 3 or 4 cycles of low-risk cHL chemotherapy. More intensive regimens such as ABVE-PC (doxorubicin, bleomycin, vincristine, etoposide, prednisone, cyclophosphamide) may be used in advanced-stage disease but are not necessary in low-stage disease. Involved-field radiation is quite effective in NLPHL and is a mainstay of therapy in adult disease. Although pediatric regimens have aimed to limit the radiation to selected groups who have a slow or inadequate response to chemotherapy, the optimal risk stratification is unclear. Rituximab, a monoclonal anti-CD20 antibody, has a very high response rate, but relapse is common when it is used as monotherapy. Therefore, rituximab is not considered standard of care for front-line management. It is often used in salvage regimens and may also be used in palliation. Advanced-stage disease occurs in 20% to 25% of patients and is harder to treat. The histologic features and presentation of advanced disease share some similarities with T-cell/histiocyte-rich large B-cell lymphoma (THRLBCL), such as a diffuse growth pattern with prominent histiocytes and T cells. Recurrent disease may also share some similarities with THRLBCL. For treatment of advanced disease, conventional cHL regimens are generally used, with preference going to alkylator-based regimens. In some situations, advanced disease may actually respond better to B-cell non-Hodgkin lymphoma therapy. Rarely, NLPHL can transform to diffuse large B-cell lymphoma (3%-14% of cases). Molecular and immunophenotypic studies suggest a clonal relationship between LP cells and cells of transformed diffuse large B-cell lymphoma, implying a common cell of origin. This transformation often occurs more than 5 years after NLPHL diagnosis but can occur as late as 20 years later. When it occurs, THRLBCL is the subtype of diffuse large B-cell lymphoma most commonly seen. Unfavorable risk factors associated with transformation in prior studies include splenic involvement and advanced-stage NLPHL at diagnosis. Prognosis is worse for transformed disease than for relapsed NLPHL. Long-term follow-up of patients with NLPHL and repeat biopsy in the event of presumed relapse are important. PREP Pearls Nodular lymphocyte-predominant Hodgkin lymphoma comprises approximately 5% of all Hodgkin lymphoma. Compared to classical Hodgkin lymphoma, nodular lymphocyte-predominant Hodgkin lymphoma is less likely to involve mediastinal and extranodal sites and is less likely to be associated with B symptoms. Low-stage nodular lymphocyte-predominant Hodgkin lymphoma has an excellent prognosis and requires less intensive therapy as compared to classical Hodgkin lymphoma. Nodular lymphocyte-predominant Hodgkin lymphoma has a risk of transformation to diffuse large B-cell lymphoma, usually more than 5 years after initial diagnosis.

A previously healthy 2-year-old girl is seen for follow-up of iron deficiency anemia with a hemoglobin of 7.2 g/dL (72 g/L) and mean corpuscular volume 70 µm3 (70 fL) discovered during a hospital admission for pneumonia after exhibiting dyspnea. Her chest radiograph revealed diffuse parenchymal infiltrates. Her diet is iron rich and includes minimal intake of cow's milk. She has had no evidence of abdominal pain, emesis, melena, diarrhea, constipation, or weight loss. After hospital discharge, she has taken a twice-daily oral iron supplement (4 mg/kg/day elemental iron) on an empty stomach with orange juice for the past month. Today's laboratory values are as follows: Laboratory Test Result Hemoglobin 7 g/dL (70 g/L) Mean corpuscular volume 68 µm3 (68 fL) Absolute reticulocyte count 320 ×103/µL (320 ×109/L) Ferritin 50 ng/mL (50 µg/L) Total iron binding capacity 454 μg/dL (81.27 µmol/L) Transferrin saturation 3% Of the following, the MOST likely cause of iron deficiency in this patient is A.Helicobacter pylori infection B.idiopathic pulmonary hemosiderosis C.sideroblastic anemia D.ulcerative colitis

n this child, a diagnosis of iron deficiency anemia with an incomplete response to adequate oral iron therapy, a history of dyspnea requiring hospitalization, and an abnormal chest radiograph, makes idiopathic pulmonary hemosiderosis (IPH) the most likely diagnosis. The differential diagnosis of microcytic anemia with low transferrin saturation also includes poor compliance with iron therapy; ongoing gastrointestinal bleeding, hematuria, severe epistaxis; malabsorption due to gastritis, antacid ingestion, celiac disease, inflammatory bowel disease; anemia of chronic inflammation; lead poisoning; underlying thalassemia traits; and sideroblastic anemia. In the absence of gastrointestinal symptoms, blood loss or malabsorption due to ulcerative colitis or Helicobacter pylori are not likely causes of iron deficiency. Sideroblastic anemia can manifest with microcytosis, normocytosis, or macrocytosis, and typically exhibit a normal to low reticulocyte count unlike this vignette. Idiopathic pulmonary hemosiderosis is a rare respiratory disease in children that leads to alveolar hemorrhage. Its prevalence and incidence are unknown.The underlying etiology is unknown, but it has been associated with cow's milk allergy, autoimmune diseases, and Trisomy 21. Hemosiderin accumulates in the pulmonary macrophages, leading to thickening of the alveolar basement membrane and irreversible interstitial fibrosis. The classic signs and symptoms of IPH include recurrent hemoptysis, diffuse parenchymal infiltrates evident on chest radiography, and iron-deficiency anemia. After an episode of pulmonary bleeding, chest radiography will reveal patchy opacities. On high-resolution computed tomographic scans of the chest, ground-glass opacities are evident. Presence of hemosiderin-laden macrophages (siderophages) in bronchoalveolar lavage confirms the diagnosis, although lung biopsy remains the reference standard for diagnosis. Most cases of IPH present before 10 years of age. In young children, respiratory symptoms range from mild intermittent cough to severe lower respiratory symptoms with tachypnea or dyspnea. Affected children can be concurrently or mistakenly diagnosed with pneumonia or reactive airway disease. Chest radiographic findings may improve during symptom-free periods. Diagnosis is often delayed because early hemoptysis is not always notable due to initial mild pulmonary bleeding and the tendency for young children to swallow their sputum. Iron deficiency anemia may be the first and only manifestation of IPH in young children. Typically, affected individuals of any age do not adequately respond to oral iron therapy. The ferritin may be normal or elevated owing to hemosiderin iron located in the lungs of patients with IPH. The elevated reticulocyte count indicates a response to oral iron supplement absorption without a corresponding increase in hemoglobin or mean corpuscular volume due to ongoing pulmonary blood loss. FIrst-line treatment is systemic corticosteroids and other immunosuppressants. If left untreated, IPH has a poor prognosis leading to interstitial fibrosis and subsequent restrictive lung disease. PREP Pearls Idiopathic pulmonary hemosiderosis can be considered in the differential diagnosis in young children with suspected iron deficiency anemia who fail to respond to adequate iron therapy as evidenced by reticulocytosis, normal ferritin levels, but no improvement in hemoglobin or mean corpuscular volume. Although hemoptysis is a classic symptom of idiopathic pulmonary hemosiderosis, this may not be notable in young children who swallow their sputum or perhaps due to initial episodes of mild pulmonary hemorrhage. Most children with idiopathic pulmonary hemosiderosis develop recurrent respiratory symptoms before 10 years of age ranging from mild intermittent cough to severe cough, wheezing, tachypnea or dyspnea, and can concurrently or mistakenly be diagnosed with pneumonia or reactive airway disease.


संबंधित स्टडी सेट्स

Croup, Epiglottis, & Bronchiolitis

View Set

Chapter 1-3 LearnSmart & Quizzes

View Set

Chapter 40: Management of Patients with Gastric and Duodenal Disorders

View Set

Intro to business final exam practice test 2

View Set

Psychological Testing: Chapter 5

View Set

5 - Life Insurance Premiums, Proceeds, and Beneficiaries - Exam

View Set

EBIO Test #3 Homework Questions & Practice Quiz & Clickers

View Set

Intro To Database Systems - Test 1

View Set